SlideShare a Scribd company logo
1 of 235
Download to read offline
Möc löc 
Líi nùi Šu 4 
C¥c th nh vi¶n tham gia bi¶n so€n 5 
1 CÂĄc b§t ÂŻng thĂčc kinh iÂșn 6 
1.1 B§t ÂŻng thĂčc giĂșa trung bÂŒnh cĂ«ng v  trung bÂŒnh nh„n (AM-GM). . . . . . . . . 6 
1.2 B§t ÂŻng thĂčc giĂșa trung bÂŒnh cĂ«ng v  trung bÂŒnh i·u ho  (AM-HM). . . . . . . 6 
1.3 B§t ÂŻng thĂčc Cauchy - Schwarz. . . . . . . . . . . . . . . . . . . . . . . . . . . . 6 
1.4 B§t ÂŻng thĂčc Holder. . . . . . . . . . . . . . . . . . . . . . . . . . . . . . . . . . 7 
1.5 B§t ÂŻng thĂčc Chebyshev. . . . . . . . . . . . . . . . . . . . . . . . . . . . . . . . 7 
1.6 B§t ÂŻng thĂčc Minkowski. . . . . . . . . . . . . . . . . . . . . . . . . . . . . . . . 7 
1.7 B§t ÂŻng thĂčc Schur. . . . . . . . . . . . . . . . . . . . . . . . . . . . . . . . . . . 7 
1.8 B§t ÂŻng thĂčc Vornicu - Schur. . . . . . . . . . . . . . . . . . . . . . . . . . . . . 8 
1.9 B§t ÂŻng thĂčc Bernoulli. . . . . . . . . . . . . . . . . . . . . . . . . . . . . . . . . 8 
1.10 Ba ti¶u chu©n SOS th÷íng g°p. . . . . . . . . . . . . . . . . . . . . . . . . . . . . 9 
2 Mët sÚ ¥nh gi¥ quen thuëc 9 
3 TuyÂșn tÂȘp b§t ÂŻng thĂčc 10 
3.1 B i 1.1 žn b i 1.40 . . . . . . . . . . . . . . . . . . . . . . . . . . . . . . . . . . . 10 
3.2 B i 2.1 žn b i 2.40 . . . . . . . . . . . . . . . . . . . . . . . . . . . . . . . . . . . 39 
3.3 B i 3.1 žn b i 3.40 . . . . . . . . . . . . . . . . . . . . . . . . . . . . . . . . . . . 59 
3.4 B i 4.1 žn b i 4.40 . . . . . . . . . . . . . . . . . . . . . . . . . . . . . . . . . . . 80 
3.5 B i 5.1 žn b i 5.40 . . . . . . . . . . . . . . . . . . . . . . . . . . . . . . . . . . . 104 
3.6 B i 6.1 žn b i 6.40 . . . . . . . . . . . . . . . . . . . . . . . . . . . . . . . . . . . 132 
3.7 B i 7.1 žn b i 7.40 . . . . . . . . . . . . . . . . . . . . . . . . . . . . . . . . . . . 148 
3.8 B i 8.1 žn b i 8.40 . . . . . . . . . . . . . . . . . . . . . . . . . . . . . . . . . . . 168 
3.9 B i 9.1 žn b i 9.40 . . . . . . . . . . . . . . . . . . . . . . . . . . . . . . . . . . . 193 
3.10 B i 10.1 žn b i 10.40 . . . . . . . . . . . . . . . . . . . . . . . . . . . . . . . . . 211 
3
Líi nùi Šu 
BiÂșn všn mÂąi nh§p nhĂŠ vĂźi nhĂșng con sĂąng d€t v o bĂ­, thuy·n všn mÂąi l¶nh ¶nh theo tĂžng con 
sĂąng i v o €i dĂ·ĂŹng, v  trong §t li·n cuĂ«c sĂšng všn cĂą nhi·u b§t cÂȘp cĂĄn ang xÂŁy ra,: : : , t§t 
cÂŁ nhĂșng i·u Ăą ·u l  cÂĄc b§t ÂŻng thĂčc trong ph€m trĂČ Â°c thĂČ cĂ”a tĂžng lŸnh vĂŒc. Trong toÂĄn 
hĂ„c cĂŽng vÂȘy nĂąi žn b§t ÂŻng thĂčc l  chĂłng ta nĂąi žn mĂ«t lĂźp b i toÂĄn khĂą m  ©n chĂča b¶n 
trong cĂą nhi·u lĂ­i giÂŁi ”p l€ kÂŒ l m say ­m bižt bao nhi¶u ngĂ·Ă­i. 
Trong thĂ­i €i cĂŠng ngh» thĂŠng tin vĂźi vi»c kžt nĂši internet b€n cĂą thÂș giao lĂ·u hĂ„c hĂ€i ֖c r§t 
nhi·u v· cÂĄc phĂ·ĂŹng phÂĄp l m b i b§t ÂŻng thĂčc, ho°c hĂ„c hĂ€i vĂźi nhi·u cuĂšn sÂĄch v· b§t ÂŻng 
thĂčc ang b y bÂĄn tr¶n thĂ  trĂ·Ă­ng nhĂ·ng Âș cĂą mĂ«t cuĂšn sÂĄch b§t ÂŻng thĂčc hay vĂźi sĂŒ hĂ«i tö 
tinh hoa kižn thĂčc cĂ”a nhi·u ngĂ·Ă­i thÂŒ i·u Ăą chÂœnh l  iÂșm m€nh cĂ”a cuĂšn sÂĄch b§t ÂŻng thĂčc 
m  c¥c b€n ang cŠm tr¶n tay. 
TuyÂșn TÂȘp B§t ÂŻng ThĂčc vĂźi khoÂŁng bĂšn tr«m b i toÂĄn b§t ÂŻng thĂčc chĂ„n lĂ„c ֖c gĂ»i tĂźi 
tĂž cÂĄc b€n trÂŽ, cÂĄc thŠy cĂŠ giÂĄo y¶u toÂĄn tr¶n mĂ„i mi·n cĂ”a tĂȘ quĂšc, Ă° Ăą bao gçm cÂĄc b i toÂĄn 
b§t ÂŻng thĂčc mĂźi sÂĄng t€o, cÂĄc b i toÂĄn b§t ÂŻng thĂčc khĂą, cÂĄc b i toÂĄn b§t ÂŻng thĂčc hay v  
thĂł vĂ  m  cÂĄc b€n trÂŽ muĂšn chia sÂŽ vĂźi mĂ„i ngĂ·Ă­i. i·u Ăą t€o n¶n sĂŒ h§p dšn, tÂœnh cÂȘp nhÂȘt v  
thíi €i cÔa cuÚn s¥ch n y. 
B€n Ă„c hÂąy nh„m nhi vĂźi nhĂșng lĂ­i giÂŁi hay, nhĂșng ĂŸ tĂ·Ă°ng Ă«c ÂĄo, nhĂșng sÂĄng kižn l€ kÂŒ trong 
cÂĄch giÂŁi tĂžng b i toÂĄn Âș tĂž Ăą rĂłt kinh nghi»m hĂ„c tÂȘp cho mÂŒnh, giĂłp cho b€n th¶m y¶u, th¶m 
tin v o vi»c giÂŁi nhi·u b i toÂĄn b§t ÂŻng thĂčc. 
VĂźi tinh thŠn l m vi»c nghi¶m tĂłc, ham hĂ„c hĂ€i nhĂąm bi¶n tÂȘp xin ֖c gĂ»i lĂ­i cÂŁm ĂŹn s„u 
s­c tĂźi t§t cÂŁ cÂĄc b€n Âą tham gia gĂ»i b i v  giÂŁi b i, çng thĂ­i cĂŽng xin b y tĂ€ sĂŒ cÂŁm 
ĂŹn v  kÂœnh trĂ„ng tĂźi thŠy giÂĄo Ch„u NgĂ„c HĂČng - THPT Ninh HÂŁi - Ninh ThuÂȘn Âą nhi»t 
tÂŒnh cĂš všn kŸ thuÂȘt latex. NhĂąm bi¶n tÂȘp cĂŽng xin gĂ»i lĂ­i cÂŁm ĂŹn tĂźi ban quÂŁn trĂ  diÂčn  n 
http://forum.mathscope.org/index.php Âą cĂȘ vĂŽ, Ă«ng vi¶n anh em trong quÂĄ trÂŒnh l m vi»c Âș 
ng y hĂŠm nay chĂłng ta cĂą mĂ«t cuĂšn sÂĄch hay, cĂą giÂĄ trĂ  cao v· kižn thĂčc chuy¶n mĂŠn m  l€i ho n 
to n miÂčn phÂœ v· t i chÂœnh. 
TUYĆĄN TĆ P B‡T NG THÙC chÂœnh thĂčc ֖c phÂĄt h nh tr¶n cĂ«ng çng m€ng nhĂșng 
ngĂ·Ă­i y¶u toÂĄn, Âș tĂž Ăą thĂȘi mĂ«t luçng giĂą mĂźi em l€i nhi·u i·u mĂźi l€ cho hĂ„c sinh, l  t i 
li»u tham khÂŁo hĂșu Âœch cho giÂĄo vi¶n trong vi»c giÂŁng d€y v  hĂ„c tÂȘp b§t ÂŻng thĂčc. 
Do thĂ­i gian g§p rĂłt v  trÂŒnh Ă« cĂą h€n, dĂČ r§t cĂš g­ng song nhĂșng sai sĂąt l  khĂą trÂĄnh khĂ€i r§t 
mong nhÂȘn ֖c sĂŒ thĂŠng cÂŁm, chia sÂŽ, gĂąp ĂŸ cĂ”a cÂĄc b€n Âș nhĂąm bi¶n tÂȘp ho n thi»n cuĂšn sÂĄch 
tĂšt hĂŹn. MĂ„i ĂŸ kižn Ăąng gĂąp xin gĂ»i v· Ă a chÂż hoangquan9@gmail. 
Thay m°t nhùm bi¶n so€n, tÊi xin ch„n th nh c£m Ïn! 
H  Nëi, ng y 10 th¥ng 8 n«m 2011 
€i di»n nhùm bi¶n so€n 
ChÔ bi¶n 
Ho ng Minh Qu„n-Batigoal 
4
C¥c th nh vi¶n tham gia bi¶n so€n 
NĂ«i dung 
 Ho ng Minh Qu„n - THPT NgÄc T£o - H  Nëi. 
 T«ng HÂŁi Tu„n - THPT NguyÂčn Ăčc CÂŁnh - TP. ThÂĄi BÂŒnh. 
 L¶ Ăčc CÂŁnh - THPT Chuy¶n L¶ Hçng Phong-Nam Ă nh. 
  o Th¥i Hi»p - PTNK - HQG HCM. 
 Ph€m Tu§n Huy - PTNK - HQG HCM. 
 Ph€m Quang H÷ng - THPT Cao B¥ Qu¥t - H  Nëi. 
 Ph€m Tižn Kha - THPT Chuy¶n L¶ Hçng Phong - TP. HCM. 
 NguyÂčn V«n KhÂĄnh - THPT Chuy¶n B­c Ninh - TP. B­c Ninh. 
 NguyÂčn ThĂ  Nguy¶n Khoa - THCS NguyÂčn Tri PhĂ·ĂŹng - TP. Huž. 
 M€c Ăčc TrÂœ - HÂŁi DĂ·ĂŹng. 
LATEX 
HĂ© trñ kŸ thuÂȘt Latex 
1. Ch„u NgĂ„c HĂČng - THPT Ninh HÂŁi -Ninh ThuÂȘn. 
2. C¥c th nh vi¶n trong nhùm bi¶n so€n. 
TrŒnh b y bŒa 
Ho ng Minh Qu„n - THPT NgÄc T£o - H  Nëi. 
5
1 CÂĄc b§t ÂŻng thĂčc kinh iÂșn 
1.1 B§t ÂŻng thĂčc giĂșa trung bÂŒnh cĂ«ng v  trung bÂŒnh nh„n (AM-GM). 
Nžu a1; a2; : : : ; an l  cÂĄc sĂš thĂŒc khĂŠng „m, thÂŒ 
a1 + a2 + : : : + an  n n p 
a1a2 : : : an: 
ÂŻng thĂčc xÂŁy ra khi v  chÂż khi a1 = a2 = : : : = an. 
1.2 B§t ÂŻng thĂčc giĂșa trung bÂŒnh cĂ«ng v  trung bÂŒnh i·u ho  (AM-HM). 
Nžu a1; a2; : : : ; an l  cÂĄc sĂš thĂŒc dĂ·ĂŹng, thÂŒ 
a1 + a2 + : : : + an 
n 
 
n 
1 
a1 
+ 1 
a2 
+ : : : + 1 
an 
: 
ÂŻng thĂčc xÂŁy ra khi v  chÂż khi a1 = a2 = : : : = an. 
ThĂŒc ch§t „y l  mĂ«t h» quÂŁ trĂŒc tižp cĂ”a b§t ÂŻng thĂčc Cauchy - Schwarz. Hai trĂ·Ă­ng hñp thĂ·Ă­ng 
֖c sĂ» döng nh§t cĂ”a b§t ÂŻng thĂčc n y l  khi n = 3 hay n = 4. 
VĂźi n = 3, ta cĂą 
a + b + c 
3 
 
3 
1 
a + 1 
b + 1 
c 
; 
1 
a 
+ 
1 
b 
+ 
1 
c 
 
9 
a + b + c 
: 
VĂźi n = 4, ta cĂą 
a + b + c + d 
4 
 
4 
1 
a + 1 
b + 1 
c + 1 
d 
; 
1 
a 
+ 
1 
b 
+ 
1 
c 
+ 
1 
d 
 
16 
a + b + c + d 
: 
1.3 B§t ÂŻng thĂčc Cauchy - Schwarz. 
D€ng sĂŹ c§p cĂ”a nĂą ֖c phÂĄt biÂșu nhĂ· sau: 
Nžu a1; a2; : : : ; an v  b1; b2; : : : ; bn l  cÂĄc sĂš thĂŒc tuĂœ ĂŸ, thÂŒ 
(a1b1 + a2b2 + : : : + anbn)2  (a21 
+ a22 
+ : : : + a2 
n)(b1 + b2 + : : : + b2 
n): 
ÂŻng thĂčc xÂŁy ra khi v  chÂż khi 
a1 
b1 
= 
a2 
b2 
= : : : = 
an 
bn 
, trong ù ta sû döng quy ÷ßc: nžu mšu 
bÂŹng 0 thÂŒ tĂ» cĂŽng bÂŹng 0. 
Trong ¥nh gi¥ tr¶n, chÄn ai = 
xi 
p 
yi 
,bi = 
p 
yi vĂźi xi; yi 2 R; yi  0, ta thu ֖c b§t ÂŻng thĂčc 
Cauchy - Schwarz d€ng ph„n thĂčc: 
Nžu x1; x2; : : : ; xn l  cÂĄc sĂš thĂŒc v  y1; y2; : : : ; yn, l  cÂĄc sĂš thĂŒc dĂ·ĂŹng, thÂŒ 
x21 
y1 
+ 
x22 
y2 
+ : : : + 
x2 
n 
yn 
 
(x1 + x2 + : : : + xn)2 
y1 + y2 + : : : + yn 
: 
ÂŻng thĂčc xÂŁy ra khi v  chÂż khi 
x1 
y1 
= 
x2 
y2 
= : : : = 
xn 
yn 
. 
6
1.4 B§t ÂŻng thĂčc Holder. 
Cho xij (i = 1; 2; : : : ;m; j = 1; 2; : : : ; n) l  cÂĄc sĂš thĂŒc khĂŠng „m. Khi Ăą ta cĂą 
Ym 
i=1 
  
Xn 
j=1 
xij 
! 1 
m 
 
Xn 
j=1 
  
Ym 
i=1 
x 
1 
m 
ij 
! 
: 
TĂȘng quÂĄt hĂŹn, nžu p1; p2; : : : ; pn l  cÂĄc sĂš thĂŒc dĂ·ĂŹng thoÂŁ mÂąn p1 + p2 + : : : + pn = 1, thÂŒ 
Ym 
i=1 
  
Xn 
j=1 
xij 
!pi 
 
Xn 
j=1 
  
Ym 
i=1 
xpi 
ij 
! 
: 
1.5 B§t ÂŻng thĂčc Chebyshev. 
Cho hai dÂąy sĂš thĂŒc a1  a2  : : :  an v  b1; b2; : : : ; bn. Khi Ăą 
1. Nžu b1  b2  : : :  bn thŒ n 
Xn 
i=1 
aibi  
  
Xn 
i=1 
ai 
!  
Xn 
i=1 
bi 
! 
; 
2. Nžu b1  b2  : : :  bn thŒ n 
Xn 
i=1 
aibi  
  
Xn 
i=1 
ai 
!  
Xn 
i=1 
bi 
! 
. 
1.6 B§t ÂŻng thĂčc Minkowski. 
Cho hai d¹y sÚ d÷Ïng a1; a2; : : : ; an v  b1; b2; : : : ; bn. Vßi mÄi r  1, ta cù  
Xn 
i=1 
(ai + bi)r 
#1 
r 
 
  
Xn 
i=1 
ari 
!1 
r 
+ 
  
Xn 
i=1 
bri 
!1 
r 
: 
vuut 
TrĂ·Ă­ng hñp r = 2 l  trĂ·Ă­ng hñp thĂ·Ă­ng ֖c sĂ» döng nh§t cĂ”a b§t ÂŻng thĂčc Minkowski. Khi Ăą 
ta cĂą Xn 
i=1 
(ai + bi)2  
vuut 
Xn 
i=1 
a2i 
+ 
vuut 
Xn 
i=1 
b2i 
: 
1.7 B§t ÂŻng thĂčc Schur. 
Cho cÂĄc sĂš thĂŒc khĂŠng „m a; b; c. Khi Ăą vĂźi mĂ„i sĂš thĂŒc dĂ·ĂŹng r, ta cĂą 
ar(a  b)(a  c) + br(b  a)(b  c) + cr(c  a)(c  b)  0: 
ÂŻng thĂčc xÂŁy ra khi v  chÂż khi a = b = c, ho°c a = 0 v  b = c, ho°c cÂĄc hoÂĄn vĂ  tĂ·ĂŹng Ăčng. 
Hai trĂ·Ă­ng hñp thĂ·Ă­ng ֖c sĂ» döng nh§t cĂ”a b§t ÂŻng thĂčc Schur l  r = 1 v  r = 2. 
VĂźi r = 1, ta cĂą b§t ÂŻng thĂčc Schur bÂȘc ba 
a3 + b3 + c3 + 3abc  ab(a + b) + bc(b + c) + ca(c + a); 
(a + b + c)3 + 9abc  4(a + b + c)(ab + bc + ca); 
(b  c)2(b + c  a) + (c  a)2(c + a  b) + (a  b)2(a + b  c)  0; 
7
a2 + b2 + c2 + 
9abc 
a + b + c 
 2(ab + bc + ca); 
a 
b + c 
+ 
b 
c + a 
+ 
c 
a + b 
+ 
4abc 
(a + b)(b + c)(c + a) 
 2: 
VĂźi r = 2, ta thu ֖c b§t ÂŻng thĂčc Schur bÂȘc bĂšn 
a4 + b4 + c4 + abc(a + b + c)  ab(a2 + b2) + bc(b2 + c2) + ca(c2 + a2): 
1.8 B§t ÂŻng thĂčc Vornicu - Schur. 
VĂźi mĂ„i sĂš thĂŒc a; b; c v  x; y; z  0, b§t ÂŻng thĂčc 
x(a  b)(a  b) + y(b  c)(b  a) + z(c  a)(c  b)  0 
óng nžu mët trong c¥c i·u ki»n sau ÷ñc tho£ m¹n 
1. a  b  c v  x  y; 
2. a  b  c v  z  y; 
3. a  b  c v  x + z  y; 
4. a  b  c  0 v  ax  by; 
5. a  b  c  0 v  cz  by; 
6. a  b  c  0 v  ax + cz  by; 
7. x; y; z l  ë d i ba c€nh cÔa mët tam gi¥c; 
8. x; y; z l  bÂŒnh phĂ·ĂŹng Ă« d i ba c€nh cĂ”a mĂ«t tam giÂĄc; 
9. ax; by; cz l  ë d i ba c€nh cÔa mët tam gi¥c; 
10. ax; by; cz l  bÂŒnh phĂ·ĂŹng Ă« d i ba c€nh cĂ”a mĂ«t tam giÂĄc; 
11. Tçn t€i mĂ«t h m lçi t : I ! R+, trong Ăą I l  tÂȘp xÂĄc Ă nh cĂ”a a; b; c, sao cho x = 
t(a); y = t(b); z = t(c). 
1.9 B§t ÂŻng thĂčc Bernoulli. 
Nžu   1 ho°c   0 thŒ (1 + x)  1 + x; 8x  1. 
Nžu 0    1 thŒ (1 + x)  1 + x; 8x  1. 
8
1.10 Ba ti¶u chu©n SOS th÷íng g°p. 
GiÂŁ sĂ» a  b  c v  cĂą: Sa(b  c)2 + Sb(c  a)2 + Sc(a  b)2  0(Sa; Sb; Sc l  cÂĄc h m chĂča 
bižn a; b; c). 
Khi Ăą b§t ÂŻng thĂčc Ăłng nžu thĂ€a mÂąn mĂ«t trong cÂĄc ti¶u chu©n. 
1.Sb  0; Sb + Sc  0; Sb + Sa  0. 
2.Vßi a; b; c  0 thÀa m¹n Sb  0; Sc  0; a2Sb + b2Sa  0. 
3.Sb  0; Sc  0; Sa(b  c) + Sb(a  c)  0 
2 Mët sÚ ¥nh gi¥ quen thuëc 
1 VĂźi mĂ„i sĂš thĂŒc a; b, ta luĂŠn cĂą 
2(a2 + b2)  (a + b)2 
ChĂčng minh. Âș ĂŸ rÂŹng 
2(a2 + b2)  (a + b)2 = (a  b)2  0; 
do Ăą ta cĂą i·u phÂŁi chĂčng minh. 
ÂŻng thĂčc xÂŁy ra khi v  chÂż khi a = b. 2 
2 VĂźi mĂ„i sĂš thĂŒc a; b; c, ta luĂŠn cĂą 
a2 + b2 + c2  ab + bc + ca 
ChĂčng minh. Âș ĂŸ rÂŹng 
a2 + b2 + c2  (ab + bc + ca) = 
1 
2 
[(a  b)2 + (b  c)2 + (c  a)2]  0; 
do vÂȘy ta cĂą i·u phÂŁi chĂčng minh. 
ÂŻng thĂčc xÂŁy ra khi v  chÂż khi a = b = c. 2 
LĂ·u ĂŸ. TĂž ÂĄnh giÂĄ n y ta suy ra 
(a + b + c)2  3(ab + bc + ca); 
v  
3(a2 + b2 + c2)  (a + b + c)2: 
3 VĂźi mĂ„i sĂš thĂŒc dĂ·ĂŹng a; b; c, ta luĂŠn cĂą 
1 
a 
+ 
1 
b 
+ 
1 
c 
 
9 
a + b + c 
ChĂčng minh. „y l  mĂ«t kžt quÂŁ Âą ֖c · cÂȘp Ă° tr¶n. LĂ­i giÂŁi cĂą thÂș sĂ» döng b§t ÂŻng thĂčc 
AM-HM ho°c Cauchy - Schwarz. ÂŻng thĂčc xÂŁy ra khi v  chÂż khi a = b = c. 2 
9
3 TuyÂșn tÂȘp b§t ÂŻng thĂčc 
3.1 B i 1.1 žn b i 1.40 
1.1 Cho x; y; z l  cÂĄc sĂš thĂŒc dĂ·ĂŹng thĂ€a mÂąn x + y + z = 1. ChĂčng minh rÂŹng: 
8x + 8y + 8z  4x+1 + 4y+1 + 4z+1 
Líi gi£i. °t a = 2x; b = 2y; c = 2z. Khi ù i·u ki»n ¹ cho ÷ñc vižt l€i th nh 
a; b; c  0; abc = 2x+y+z = 64; 
v  ta cŠn chĂčng minh 
a3 + b3 + c3  4(a2 + b2 + c2): 
Âș ĂŸ rÂŹng ta cĂą ÂŻng thĂčc 
a3 + 32  6a2 = (a  4)2(a + 2); 
tĂž Ăą sĂ» döng giÂŁ thižt a  0 ta suy ra a3 + 32  6a2. Thižt lÂȘp cÂĄc b§t ÂŻng thĂčc tĂ·ĂŹng tĂŒ cho 
b v  c v  cĂ«ng vž theo vž cÂĄc b§t ÂŻng thĂčc thu ֖c, ta cĂą 
a3 + b3 + c3 + 96  6(a2 + b2 + c2): 
NhĂ· vÂȘy Âș kžt thĂłc chĂčng minh ta cŠn chÂż ra rÂŹng 
6(a2 + b2 + c2)  4(a2 + b2 + c2) + 96; 
hay 2(a2 + b2 + c2)  96. Tuy nhi¶n b§t ÂŻng thĂčc n y Ăłng theo b§t ÂŻng thĂčc AM-GM cho ba 
sĂš: 
2(a2 + b2 + c2)  2:3 3 p 
a2b2c2 = 6 3 p 4096 = 96: 
NhĂ· vÂȘy phÂČp chĂčng minh žn „y ho n t§t.2 
1.2 Cho a; b; c l  cÂĄc sĂš thĂŒc thoÂŁ mÂąn a  4; b  5; c  6 v  a2 + b2 + c2 = 90. TÂŒm giÂĄ trĂ  
nhĂ€ nh§t cĂ”a biÂșu thĂčc: 
P = a + b + c 
Líi gi£i. °t a = m+ 4; b = n + 5; c = p + 6, khi ù m; n; p  0 v  tÞ gi£ thižt a2 + b2 + c2 = 90 
ta suy ra 
m2 + n2 + p2 + 8m + 10n + 12p = 13: 
Âș ĂŸ rÂŹng ta cĂą ÂŻng thĂčc sau 
(m+ n + p)2 + 12(m+ n + p) = (m2 + n2 + p2 + 8m+ 10n + 12p) + 2(mn + np + pm + 2m+ n): 
žn „y ta sĂ» döng cÂĄc giÂŁ thižt Âą cho Âș cĂą 
(m + n + p)2 + 12(m + n + p)  13; 
tĂž Ăą ta suy ra m+ n + p  1. Thay m = a  4; n = b  5; p = c  6 ta suy ra a + b + c  10 hay 
P  16. 
10
Cuçi cĂČng, vĂźi a = 4; b = 5; c = 7 (thoÂŁ mÂąn cÂĄc i·u ki»n Âą cho) ta cĂą P = 16 n¶n ta kžt luÂȘn 
16 l  giÂĄ trĂ  nhĂ€ nh§t cĂ”a biÂșu thĂčc P. 
PhÂČp chĂčng minh ho n t§t. 2 
1.3 Cho x; y; z l  cÂĄc sĂš thĂŒc thoÂŁ mÂąn xy + yz + 3zx = 1. TÂŒm giÂĄ trĂ  nhĂ€ nh§t cĂ”a biÂșu 
thĂčc: 
P = x2 + y2 + z2 
Líi gi£i. °t a = 
p 
17 
4 
9 + 3 
v  b = 
3 + 
p 
17 
4 
, khi ù a = 3b v  a+1 = 2b2 = c = 
p 
17 
4 
13 + 3 
. p 
döng b§t ÂŻng thĂčc AM-GM ta thu ֖c cÂĄc b§t ÂŻng thĂčc sau 
x2 + b2y2  2bxy; 
by2 + z2  2byz; 
a(z2 + x2)  2azx: 
žn „y ta cĂ«ng vž theo vž cÂĄc b§t ÂŻng thĂčc thu ֖c Âș cĂą 
(a + 1)(x2 + z2) + 2b2y2  2b(xy + yz) + 2azx; 
hay c(x2 + y2 + z2)  2b(xy + yz + 3zx). TĂž Ăą ta thay cÂĄc giÂĄ trĂ  cĂ”a xy + yz + 3zx, b v  c Âș 
֖c 
P = x2 + y2 + z2  
p 
17  3 
2 
: 
CuĂši cĂČng, vĂźi x = z = 
1 
4 p 
17 
v  y = 
r 
p 
17  51 
13 
34 
(thoÂŁ mÂąn giÂŁ thižt) thÂŒ P = 
p 
17  3 
2 
n¶n ta 
kžt luÂȘn 
p 
17  3 
2 
l  giÂĄ trĂ  nhĂ€ nh§t cĂ”a biÂșu thĂčc P. 
PhÂČp chĂčng minh ho n t§t.2 
1.4 Cho a; b; c l  cÂĄc sĂš thĂŒc dĂ·ĂŹng thoÂŁ mÂąn a + b + c = 1. ChĂčng minh rÂŹng: 
a7 + b7 
a5 + b5 + 
b7 + c7 
b5 + c5 + 
c7 + a7 
c5 + a5 
 
1 
3 
LĂ­i giÂŁi. TrĂ·Ăźc hžt ta cĂą ÂŻng thĂčc sau 
2(a7 + b7)  (a2 + b2)(a5 + b5) = (a  b)2(a + b)(a4 + a3b + a2b2 + ab3 + b4); 
do vÂȘy tĂž giÂŁ thižt a; b  0 ta suy ra 
a7 + b7 
a5 + b5 
 
a2 + b2 
2 
: 
Ho n to n tĂ·ĂŹng tĂŒ ta cĂŽng cĂą 
b7 + c7 
b5 + c5 
 
b2 + c2 
2 
v  
c7 + a7 
c5 + a5 
 
c2 + a2 
2 
. žn „y ta cëng vž theo 
vž ba b§t ÂŻng thĂčc thu ֖c Âș cĂą 
a7 + b7 
a5 + b5 + 
b7 + c7 
b5 + c5 + 
c7 + a7 
c5 + a5 
 a2 + b2 + c2: 
11
NhĂ· vÂȘy Âș kžt thĂłc chĂčng minh ta cŠn chÂż ra rÂŹng 
a2 + b2 + c2  
1 
3 
: 
Tuy nhi¶n b§t ÂŻng thĂčc tr¶n Ăłng do 
a2 + b2 + c2  
1 
3 
= a2 + b2 + c2  
(a + b + c)2 
3 
= 
(a  b)2 + (b  c)2 + (c  a)2 
3 
 0: 
NhĂ· vÂȘy phÂČp chĂčng minh žn „y ho n t§t.2 
1.5 Cho a; b; c l  cÂĄc sĂš thĂŒc dĂ·ĂŹng. ChĂčng minh rÂŹng: 
b2c 
a3(b + c) 
+ 
c2a 
b3(c + a) 
+ 
a2b 
c3(a + b) 
 
1 
2 
(a + b + c) 
Líi gi£i. Ta ¥p döng AM-GM cho ba sÚ nh÷ sau: 
b2c 
a3(b + c) 
+ 
b + c 
4bc 
+ 
1 
2b 
s 
 3 3 
b2c 
a3(b + c) 
: 
(b + c) 
4bc 
: 
1 
2b 
= 
3 
2a 
; 
tĂž Ăą ta suy ra 
b2c 
a3(b + c) 
 
3 
2a 
 
3 
4b 
 
1 
4c 
: 
Thižt lÂȘp hai b§t ÂŻng thĂčc tĂ·ĂŹng tĂŒ v  cĂ«ng l€i, ta suy ra 
b2c 
a3(b + c) 
+ 
c2a 
b3(c + a) 
+ 
a2b 
c3(a + b) 
 
 
3 
2 
 
3 
4 
 
1 
4 
 
(a + b + c) = 
1 
2 
(a + b + c): 
PhÂČp chĂčng minh ho n t§t.2 
1.6 Cho a; b; c l  cÂĄc sĂš thĂŒc khĂŠng „m. ChĂčng minh rÂŹng: 
p 
3(a  b)(b  c)(c  a) 
(a + b + c)3  6 
LĂ­i giÂŁi. B§t ÂŻng thĂčc ban Šu mang tÂœnh hoÂĄn vĂ  giĂșa cÂĄc bižn n¶n khĂŠng m§t tÂœnh tĂȘng quÂĄt, 
ta giÂŁ sĂ» a = max fa; b; cg. 
VĂźi a  b  c thÂŒ vž phÂŁi l  biÂșu thĂčc khĂŠng dĂ·ĂŹng, trong khi vž trÂĄi l  biÂșu thĂčc khĂŠng „m n¶n 
b§t ÂŻng thĂčc cŠn chĂčng minh hiÂșn nhi¶n Ăłng. Do vÂȘy ta xÂČt trĂ·Ă­ng hñp a  c  b. Khi Ăą bÂŒnh 
phĂ·ĂŹng hai vž ta thu ֖c b§t ÂŻng thĂčc tĂ·ĂŹng Ă·ĂŹng sau: 
(a + b + c)6  108[(a  b)(b  c)(c  a)]2: 
Âș ĂŸ rÂŹng cÂĄc bižn khĂŠng „m, v  vĂźi vi»c s­p thĂč tĂŒ nhĂ· tr¶n thÂŒ 
[(a  b)(b  c)(c  a)]2 = [(a  b)(c  b)(a  c)]2  (a  c)2a2c2: 
žn „y ta ÂĄp döng b§t ÂŻng thĂčc AM-GM Âș cĂą 
4(a  c)2a2c2 = (a  c)2:2ac:2ac  
[(a  c)2 + 2ac + 2ac]3 
27 
= 
(a + c)6 
27 
; 
tĂž Ăą ta suy ra 
[(a  b)(b  c)(c  a)]2  
(a + c)6 
108 
; 
12
v  nhĂ· vÂȘy ta Âą chĂčng minh ֖c b§t ÂŻng thĂčc ban Šu vÂŒ 
(a + b + c)6  (a + c)6  108[(a  b)(b  c)(c  a)]2: 
PhÂČp chĂčng minh ho n t§t.2 
1.7 Cho a; b; c l  cÂĄc sĂš thĂŒc dĂ·ĂŹng thoÂŁ mÂąn a + b + c = 
1 
a 
+ 
1 
b 
+ 
1 
c 
. ChĂčng minh rÂŹng: 
2(a + b + c)  
p 
a2 + 3 + 
p 
b2 + 3 + 
p 
c2 + 3 
LĂ­i giÂŁi. DÂč th§y b§t ÂŻng thĂčc cŠn chĂčng minh tĂ·ĂŹng Ă·ĂŹng vĂźi mĂ©i b§t ÂŻng thĂčc trong dÂąy 
sau 
(2a  
p 
a2 + 3) + (2b  
p 
b2 + 3) + (2c  
p 
c2 + 3)  0; 
a2  1 
2a + 
p 
a2 + 3 
+ 
b2  1 
2b + 
p 
b2 + 3 
+ 
c2  1 
2c + 
p 
c2 + 3 
 0; 
a2  1 
a 
2 + 
r 
1 + 
3 
a2 
+ 
b2  1 
b 
2 + 
r 
1 + 
3 
b2 
+ 
c2  1 
c 
2 + 
r 
1 + 
3 
c2 
 0: 
CÂĄc b§t ÂŻng thĂčc tr¶n ·u mang tÂœnh Ăši xĂčng giĂșa cÂĄc bižn n¶n khĂŠng m§t tÂœnh tĂȘng quÂĄt ta 
ho n to n cĂą thÂș giÂŁ sĂ» a  b  c. Khi Ăą khĂŠng khĂą Âș ta suy ra 
a2  1 
a 
 
b2  1 
b 
 
c2  1 
c 
v  
1 
2 + 
q 
1 + 3 
a2 
 
1 
2 + 
q 
1 + 3 
b2 
 
1 
2 + 
q 
1 + 3 
b2 
: 
NhĂ· vÂȘy theo b§t ÂŻng thĂčc Chebyshev ta ֖c 
a2  1 
a 
2 + 
q 
1 + 3 
a2 
+ 
b2  1 
b 
2 + 
r 
1 + 
3 
b2 
+ 
c21 
c 
2 + 
r 
1 + 
3 
c2 
 
1 
3 
Xa2  1 
a 
 
0 
BB@ 
X 1 
2 + 
r 
1 + 
3 
a2 
1 
CCA 
Nh÷ng theo gi£ thižt ta l€i cù 
Xa2  1 
a 
= (a + b + c)  
 
1 
a 
+ 
1 
b 
+ 
1 
c 
 
= 0 
n¶n ta suy ra 
a2  1 
a 
2 + 
q 
1 + 3 
a2 
+ 
b2  1 
b 
2 + 
r 
1 + 
3 
b2 
+ 
c2  1 
c 
2 + 
r 
1 + 
3 
c2 
 0, v  vÂŒ vÂȘy b§t ÂŻng thĂčc Âą cho 
cĂŽng Ăłng. 
PhÂČp chĂčng minh ho n t§t.2 
1.8 Cho a; b; c l  cÂĄc sĂš thĂŒc dĂ·ĂŹng thoÂŁ mÂąn a + b + c = 3. ChĂčng minh rÂŹng: 
ab 
p 
c2 + 3 
+ 
bc 
p 
a2 + 3 
+ 
ca 
p 
b2 + 3 
 
3 
2 
13
LĂ­i giÂŁi. TrĂ·Ăźc hžt Âș ĂŸ rÂŹng 
ab + bc + ca  
(a + b + c)2 
3 
=  
 
(a  b)2 + (b  c)2 + (c  a)2 
6 
 
 0; 
do Ăą tĂž giÂŁ thižt ta suy ra ab + bc + ca  3. NhĂ· vÂȘy 
ab 
p 
c2 + 3 
 
ab 
p 
c2 + ab + bc + ca 
= 
ab p 
(c + a)(b + c) 
: 
žn „y ta ÂĄp döng b§t ÂŻng thĂčc AM-GM Âș cĂą 
ab 
p 
c2 + 3 
 
1 
2 
 
ab 
c + a 
+ 
ab 
b + c 
 
: 
Thižt lÂȘp hai b§t ÂŻng thĂčc tĂ·ĂŹng tĂŒ v  cĂ«ng l€i, ta suy ra dÂąy cÂĄc ÂĄnh giÂĄ sau 
ab 
p 
c2 + 3 
+ 
bc 
p 
a2 + 3 
+ 
ca 
p 
b2 + 3 
 
1 
2 
 
ab 
c + a 
+ 
bc 
c + a 
 
+ 
 
bc 
a + b 
+ 
ca 
a + b 
 
+ 
 
ca 
b + c 
+ 
ab 
b + c 
 
; 
ab 
p 
c2 + 3 
+ 
bc 
p 
a2 + 3 
+ 
ca 
p 
b2 + 3 
 
a + b + c 
2 
; 
tĂž Ăą vĂźi lĂ·u ĂŸ a + b + c = 3 ta suy ra b§t ÂŻng thĂčc Âą cho l  Ăłng. 
PhÂČp chĂčng minh ho n t§t.2 
1.9 Cho a; b; c l  cÂĄc sĂš thĂŒc dĂ·ĂŹng thay ĂȘi b§t kÂŒ. ChĂčng minh rÂŹng: 
b + c 
a 
+ 
c + a 
b 
+ 
a + b 
c 
2 
 4(ab + bc + ca) 
 
1 
a2 + 
1 
b2 + 
1 
c2 
 
LĂ­i giÂŁi 1. DÂč th§y rÂŹng b§t ÂŻng thĂčc ban Šu tĂ·ĂŹng Ă·ĂŹng vĂźi mĂ©i b§t ÂŻng thĂčc trong dÂąy 
sau 
[ab(a + b) + bc(b + c) + ca(c + a)]2  4(a + b + c)(a2b2 + b2c2 + c2a2) 
X 
a2b2(a + b)2 + 2abc[ 
X 
a(a + b)(a + c)]  4 
nX 
a3b3 + abc[ 
X 
o 
ab(a + b)] 
Tuy nhi¶n Âș ĂŸ rÂŹng 
X 
X 
a2b2(a + b)2  4( 
a3b3) = 
X 
a2b2(a  b)2  0 
v  
2abc[ 
X 
a(a + b)(a + c)]  4 
n 
abc[ 
X 
o 
= 2abc[a3 + b3 + c3 + 3abc  
ab(a + b)] 
X 
ab(a + b)]  0; 
do Ăą b§t ÂŻng thĂčc ban Šu l  Ăłng. PhÂČp chĂčng minh žn „y ho n t§t.2 
LĂ­i giÂŁi 2. B§t ÂŻng thĂčc ban Šu mang tÂœnh hoÂĄn vĂ  giĂșa cÂĄc bižn, n¶n khĂŠng m§t tÂœnh tĂȘng 
quÂĄt, ta giÂŁ sĂ» b = max fa; b; cg. 
Ta ÂĄp döng b§t ÂŻng thĂčc AM-GM nhĂ· sau 
 
b + c 
a 
+ 
c + a 
b 
+ 
a + b 
c 
2 
= 
 
a 
b 
+ 
b 
a 
+ 
a 
c 
 
+ 
 
b 
c 
+ 
c 
b 
+ 
c 
a 
2 
 4 
 
a 
b 
+ 
b 
a 
+ 
a 
c 
 
b 
c 
+ 
c 
b 
+ 
c 
a 
 
: 
14
NhĂ· vÂȘy Âș kžt thĂłc chĂčng minh, ta cŠn chÂż ra rÂŹng 
 
a 
b 
+ 
b 
a 
+ 
a 
c 
 
b 
c 
+ 
c 
b 
+ 
c 
a 
 
 (ab + bc + ca) 
 
1 
a2 + 
1 
b2 + 
1 
c2 
 
: 
Tuy nhi¶n bÂŹng phÂČp bižn ĂȘi tĂ·ĂŹng Ă·ĂŹng ta ֖c 
(b  a)(b  c) 
ca 
 0; 
l  mët ¥nh gi¥ óng do ta ¹ gi£ sû b = max fa; b; cg. 
PhÂČp chĂčng minh žn „y ho n t§t.2 
LĂ­i giÂŁi 3. B§t ÂŻng thĂčc ban Šu mang tÂœnh Ăši xĂčng giĂșa cÂĄc bižn n¶n khĂŠng m§t tÂœnh tĂȘng 
quÂĄt, ta giÂŁ sĂ» b nÂŹm giĂșa a v  c. 
Ta ÂĄp döng b§t ÂŻng thĂčc AM-GM nhĂ· sau: 
4(ab + bc + ca) 
 
1 
a2 + 
1 
b2 + 
1 
c2 
 
 
 
ab + bc + ca 
ca 
+ ca 
 
1 
a2 + 
1 
b2 + 
1 
c2 
2 
: 
NhĂ· vÂȘy Âș kžt thĂłc chĂčng minh, ta cŠn chÂż ra rÂŹng 
b + c 
a 
+ 
c + a 
b 
+ 
a + b 
c 
 
ab + bc + ca 
ca 
+ ca 
 
1 
a2 + 
1 
b2 + 
1 
c2 
 
: 
ThĂŒc hi»n phÂČp bižn ĂȘi tĂ·ĂŹng Ă·ĂŹng ta ֖c b§t ÂŻng thĂčc 
(a  b)(b  c) 
b2 
 0; 
tuy nhi¶n „y l€i l  mĂ«t ÂĄnh giÂĄ Ăłng do ta Âą giÂŁ sĂ» b nÂŹm giĂșa a v  c. 
PhÂČp chĂčng minh žn „y ho n t§t.2 
NhÂȘn xÂČt. LĂ­i giÂŁi Šu ti¶n khĂŠng mang nhi·u ĂŸ nghŸa l­m, vÂŒ nĂą ĂŹn thuŠn chÂż l  bižn ĂȘi tĂ·ĂŹng 
Ă·ĂŹng k±m theo mĂ«t chĂłt tinh ĂŸ trong sĂ» döng cÂĄc ÂĄnh giÂĄ quen thuĂ«c v  cĂŹ bÂŁn. Ð „y ta b n 
th¶m v· hai líi gi£i bng AM-GM. 
Ta nh 
ÂȘn th§y rÂŹng phÂĄt biÂșu cĂ”a b i toÂĄn cĂą d€ng ChĂčng minh rÂŹng A2  4BC (Ă° „y 
b + c 
A = 
a 
+ 
c + a 
b 
+ 
a + b 
c 
2 
, B = ab + bc + ca v  C = 
1 
a2 + 
1 
b2 + 
1 
c2 . NhÂȘn xÂČt n y khÂĄ °c 
bi»t, nù gióp ta li¶n t÷ðng žn mët ¥nh gi¥ quen thuëc sau bng AM-GM: 
(x + y)2  4xy 8x; y  0: 
Do vÂȘy, mĂ«t cÂĄch tĂŒ nhi¶n ta nghŸ ra hai hĂ·Ăźng Âș giÂŁi quyžt b i toÂĄn tr¶n bÂŹng AM-GM: 
1. BiÂșu diÂčn A = X +Y , vĂźi X v  Y l  hai €i l֖ng thÂœch hñp, sau Ăą ÂĄp döng b§t ÂŻng thĂčc 
AM-GM Âș cĂą A2  4XY , tĂž Ăą i chĂčng minh XY  BC; ho°c 
15
2. BiÂșu diÂčn BC = 
B 
D 
:CD, vĂźi D l  mĂ«t €i l֖ng thÂœch hñp, sau Ăą ÂĄp döng b§t ÂŻng thĂčc 
AM-GM Âș cĂą 4BC  
 
B 
D 
+ CD 
2 
, tĂž Ăą i chĂčng minh A  
B 
D 
+ CD. 
Ð „y ta hiÂșu cöm tĂž thÂœch hñp l  nhĂ· thž n o? LĂ·u ĂŸ rÂŹng mĂ«t trong nhĂșng i·u cŠn Âș ĂŸ 
trong mĂ„i chĂčng minh b§t ÂŻng thĂčc l  cŠn phÂŁi ĂŹn giÂŁn hoÂĄ b§t ÂŻng thĂčc cŠn chĂčng minh. Ta 
cĂą thÂș tÂŒm cÂĄch giÂŁm bÂȘc, chu©n hoÂĄ i·u ki»n, : : :, nhĂ·ng tĂŒu chung l€i, ta luĂŠn muĂšn b§t ÂŻng 
thĂčc cŠn chĂčng minh trĂ° n¶n ĂŹn giÂŁn nh§t cĂą thÂș, Âș tĂž Ăą ÂĄp döng nh” nh ng cÂĄc ÂĄnh giÂĄ 
quen thuĂ«c ho°c bižn ĂȘi tĂ·ĂŹng Ă·ĂŹng. Ð „y ta tÂŒm cÂĄch thu gĂ„n ÂĄnh giÂĄ sau cĂČng theo kiÂșu 
tri»t ti¶u mĂ«t l֖ng ÂĄng kÂș cÂĄc phŠn tĂ» chung, tĂčc l  Ă° ÂĄnh giÂĄ XY  BC ho°c A  
B 
D 
+ CD, 
cÂĄc €i l֖ng X; Y;D ֖c chĂ„n sao cho Ă° hai vž cĂ”a b§t ÂŻng thĂčc cĂą nhi·u phŠn tĂ» chung Âș 
ta rĂłt gĂ„n. Cö thÂș: 
HĂ·Ăźng 1. TrĂ·Ăźc ti¶n ta vižt l€i A v  khai triÂșn tÂœch BC nhĂ· sau: 
A = 
b 
a 
+ 
c 
a 
+ 
c 
b 
+ 
a 
b 
+ 
a 
c 
+ 
b 
c 
= X + Y; 
BC = 
a 
c 
+ 
c 
b 
+ 
b 
a 
+ 
a 
b 
+ 
b 
c 
+ 
c 
a 
+ 
ca 
b2 + 
ab 
c2 + 
bc 
a2 : 
Âș ĂŸ rÂŹng trong BC cĂą phŠn tĂ» 
ca 
b2 , n¶n ta cŠn cù 
a 
b 
v  
c 
b 
Ă° X v  Y tĂ·ĂŹng Ăčng: 
X = 
a 
b 
+ : : : ; Y = 
c 
b 
+ : : : 
M°t kh¥c, trong BC cù phŠn tû 
a 
b 
, m  ð Y ¹ cù 
c 
b 
n¶n ta cŠn phŠn tû 
a 
c 
Ă° trong X: 
X = 
a 
b 
+ 
a 
c 
+ : : : ; Y = 
c 
b 
+ : : : 
Tižp töc, trong BC cù phŠn tû 
ab 
c2 , n¶n ta cŠn cù 
a 
c 
v  
b 
c 
Ă° X v  Y tĂ·ĂŹng Ăčng: 
X = 
a 
b 
+ 
a 
c 
+ : : : ; Y = 
c 
b 
+ 
b 
c 
+ : : : 
Tižp töc nhĂ· vÂȘy ta sÂł tÂŒm ֖c hai €i l֖ng X; Y chÂŻng h€n nhĂ· sau: 
X = 
a 
b 
+ 
b 
a 
+ 
a 
c 
; Y = 
b 
c 
+ 
c 
b 
+ 
c 
a 
; 
v  ta cĂą ֖c lĂ­i giÂŁi thĂč hai. CŠn lĂ·u ĂŸ rÂŹng „y khĂŠng phÂŁi l  cÂĄch chĂ„n duy nh§t. 
HĂ·Ăźng 2. XÂČt hi»u sau 
A  
B 
D 
 CD = 
b + c 
a 
+ 
c + a 
b 
+ 
a + b 
c 
 
ab + bc + ca 
D 
 D 
 
1 
a2 + 
1 
b2 + 
1 
c2 
 
: 
Âș ĂŸ rÂŹng trong hi»u tr¶n thÂŒ h» sĂš cĂ”a bižn b bÂŹng 
1 
c 
+ 
1 
a 
 
c + a 
D 
; 
nhĂ· vÂȘy Âș tÂŒm cÂĄch thu gĂ„n b§t ÂŻng thĂčc, t€i sao ta khĂŠng cho h» sĂš cĂ”a bižn b bÂŹng khĂŠng? 
Cö thÂș, nžu chĂ„n D = ca thÂŒ 
16
A  
B 
D 
 CD = 
b + c 
a 
+ 
c + a 
b 
+ 
a + b 
c 
 
ab + bc + ca 
ca 
 ca 
 
1 
a2 + 
1 
b2 + 
1 
c2 
 
= 
(a  b)(b  c) 
b2 ; 
v  nhĂ· vÂȘy ta Âą cĂą lĂ­i giÂŁi thĂč ba. 
1.10 Cho a; b; c l  cÂĄc sĂš thĂŒc dĂ·ĂŹng thoÂŁ mÂąn a + b + c = 1. TÂŒm giÂĄ trĂ  lĂźn nh§t cĂ”a biÂșu 
thĂčc: 
P = ab + bc + ca + 
5 
2 
p 
ab + (b + c) 
[(a + b) 
p 
bc + (c + a) 
p 
ca] 
LĂ­i giÂŁi. TrĂ·Ăźc hžt ta ÂĄp döng b§t ÂŻng thĂčc AM-GM nhĂ· sau: 
2(a + b)2 + 2ab = 
(a + b)2 
2 
+ 
(a + b)2 
2 
+ 
(a + b)2 
2 
+ 
(a + b)2 
2 
r 
+ 2ab  5 5 
ab(a + b)8 
8 
v  
(a + b)3  (2 
p 
ab)3 = 8( 
p 
ab)3; 
tĂž Ăą kžt hñp hai b§t ÂŻng thĂčc n y Âș cĂą 
2(a + b)2 + 2ab  5(a + b) 
p 
ab: 
Thižt lÂȘp hai b§t ÂŻng thĂčc tĂ·ĂŹng tĂŒ v  cĂ«ng l€i, ta suy ra 
p 
ab + (b + c) 
5[(a + b) 
p 
bc + (c + a) 
p 
ca]  4(a2 + b2 + c2) + 6(ab + bc + ca) 
žn „y ta cĂ«ng th¶m 2(ab + bc + ca) v o mĂ©i vž Âș cĂą 
p 
ab + (b + c) 
2(ab + bc + ca) + 5[(a + b) 
p 
bc + (c + a) 
p 
ca]  4(a + b + c)2; 
tĂž Ăą ta suy ra P  2(a + b + c)2 = 2. 
CuĂši cĂČng, vĂźi a = b = c = 
1 
3 
(thoÂŁ mÂąn i·u ki»n) thÂŒ P = 2 n¶n ta suy ra 2 l  giÂĄ trĂ  lĂźn nh§t 
cĂ”a biÂșu thĂčc P. 
PhÂČp chĂčng minh ho n t§t.2 
1.11 Cho a; b; c l  cÂĄc sĂš thĂŒc dĂ·ĂŹng thoÂŁ mÂąn 
1 
a 
+ 
1 
b 
+ 
1 
c 
 16(a+b+c). ChĂčng minh rÂŹng: 
1 
(a + b + 2 
p 
a + c)3 
+ 
1 
(b + c + 2 
p 
b + a)3 
+ 
1 
(c + a + 2 
p 
c + b)3 
 
8 
9 
LĂ­i giÂŁi. TrĂ·Ăźc hžt ta ÂĄp döng b§t ÂŻng thĂčc AM-GM nhĂ· sau: 
a + b + 
r 
a + c 
2 
+ 
r 
a + c 
2 
r 
 3 3 
(a + b)(a + c) 
2 
; 
tĂž Ăą ta suy ra 
1 
(a + b + 2 
p 
a + c)3 
 
2 
27(a + b)(a + c) 
: 
17
CĂ«ng vž theo vž b§t ÂŻng thĂčc n y vĂźi hai b§t ÂŻng thĂčc tĂ·ĂŹng tĂŒ cho ta 
1 
(a + b + 2 
p 
a + c)3 
+ 
1 
(b + c + 2 
p 
b + a)3 
+ 
1 
(c + a + 2 
p 
c + b)3 
 
4(a + b + c) 
27(a + b)(b + c)(c + a) 
: 
HĂŹn nĂșa, theo mĂ«t kžt quÂŁ quen thuĂ«c, ta l€i cĂą 
(a + b)(b + c)(c + a)  
8 
9 
(a + b + c)(ab + bc + ca); 
do vÂȘy 
1 
(a + b + 2 
p 
a + c)3 
+ 
1 
(b + c + 2 
p 
b + a)3 
+ 
1 
(c + a + 2 
p 
c + b)3 
 
1 
6(ab + bc + ca) 
:() 
žn „y ta sĂ» döng giÂŁ thižt v  ÂĄnh giÂĄ cĂŹ bÂŁn (ab + bc + ca)2  3abc(a + b + c) Âș cĂą 
16(a + b + c)  
1 
a 
+ 
1 
b 
+ 
1 
c 
 
3(a + b + c) 
ab + bc + ca 
; 
tĂž Ăą suy ra ab + bc + ca  
3 
16 
. Kžt hñp vßi () ta suy ra 
1 
(a + b + 2 
p 
a + c)3 
+ 
1 
(b + c + 2 
p 
b + a)3 
+ 
1 
(c + a + 2 
p 
c + b)3 
 
8 
9 
: 
PhÂČp chĂčng minh žn „y ho n t§t.2 
NhÂȘn xÂČt. 
1. CĂą thÂș th§y ÂĄnh giÂĄ ban Šu a+b+ 
r 
a + c 
2 
+ 
r 
a + c 
2 
r 
 3 3 
(a + b)(a + c) 
2 
chÂœnh l  iÂșm 
m§u chĂšt Âș giÂŁi quyžt b i toÂĄn. ThĂŒc ra ÂĄnh giÂĄ n y khĂŠng khĂą nghŸ tĂźi vÂŒ · b i Âą ngŠm 
gñi ĂŸ cho chĂłng ta phÂŁi ÂĄp döng b§t ÂŻng thĂčc AM-GM cho ba sĂš. 
2. Sau khi ÂĄnh giÂĄ bÂŹng AM-GM, ta cĂą thÂș sĂ» döng luĂŠn giÂŁ thižt Âș Ă·a v· b§t ÂŻng thĂčc 
thuŠn nh§t sau: 
(a + b + c) 
(a + b)(b + c)(c + a) 
 
3(ab + bc + ca) 
8abc(a + b + c) 
: 
B§t ÂŻng thĂčc n y cĂą thÂș ֖c chĂčng minh bÂŹng nhi·u cÂĄch khÂĄc nhau. 
1.12 Cho a; b; c l  cÂĄc sĂš thĂŒc dĂ·ĂŹng thoÂŁ mÂąn a + b + c = 
1 
a 
+ 
1 
b 
+ 
1 
c 
. ChĂčng minh rÂŹng: 
5(a + b + c)  7 + 8abc 
Líi gi£i. Tr÷ßc hžt tÞ gi£ thižt ta cù 
a + b + c = 
1 
a 
+ 
1 
b 
+ 
1 
c 
 
9 
a + b + c 
; 
tĂž Ăą suy ra a + b + c = 3. 
CĂŽng tĂž giÂŁ thižt ta cĂą ab+bc+ca = abc(a+b+c), tĂž „y ta suy ra b§t ÂŻng thĂčc sau l  tĂ·ĂŹng 
Ă·ĂŹng vĂźi b§t ÂŻng thĂčc cŠn chĂčng minh 
5(a + b + c)2  7(a + b + c) + 8(ab + bc + ca): 
18
Âș ĂŸ rÂŹng ta cĂą ÂĄnh giÂĄ cĂŹ bÂŁn sau: 
(a + b + c)2  3(ab + bc + ca); 
do vÂȘy Âș cĂą kžt luÂȘn cho b i toÂĄn ta cŠn chÂż ra rÂŹng 
5(a + b + c)2  7(a + b + c) + 
8(a + b + c)2 
3 
; 
hay a + b + c  3, l  mĂ«t ÂĄnh giÂĄ Ăłng do ta Âą chĂčng minh Ă° tr¶n. 
Do vÂȘy b§t ÂŻng thĂčc ban Šu ֖c chĂčng minh xong. B i toÂĄn kžt thĂłc.2 
1.13 Cho a; b; c l  cÂĄc sĂš thĂŒc dĂ·ĂŹng thoÂŁ mÂąn 
1 
a 
+ 
1 
b 
+ 
1 
c 
 16(a+b+c). ChĂčng minh rÂŹng: 
1 
2 + a2 + 
1 
2 + b2 + 
1 
2 + c2 
 1 
LĂ­i giÂŁi. B§t ÂŻng thĂčc cŠn chĂčng minh tĂ·ĂŹng Ă·ĂŹng vĂźi 
a2 
2 + a2 + 
b2 
2 + b2 + 
c2 
2 + c2 
 1: 
p döng b§t ÂŻng thĂčc Cauchy - Schwarz, ta cĂą 
a2 
2 + a2 + 
b2 
2 + b2 + 
c2 
2 + c2 
 
(a + b + c)2 
a2 + b2 + c2 + 6 
: 
NhĂ· vÂȘy Âș kžt thĂłc chĂčng minh ta cŠn chÂż ra rÂŹng 
(a + b + c)2 
a2 + b2 + c2 + 6 
 1: 
ThĂŒc hi»n phÂČp khai triÂșn tĂ·ĂŹng Ă·ĂŹng ta ֖c ab + bc + ca  3. Tuy nhi¶n b§t ÂŻng thĂčc n y 
Ăłng nhĂ­ v o giÂŁ thižt cĂ”a b i toÂĄn. LĂ·u ĂŸ rÂŹng tĂž giÂŁ thižt ta cĂą 
ab + bc + ca = abc(a + b + c); 
v  theo mĂ«t ÂĄnh giÂĄ quen thuĂ«c thÂŒ abc(a + b + c)  
(ab + bc + ca)2 
3 
, tĂž Ăą ta suy ra 
ab + bc + ca  
(ab + bc + ca)2 
3 
; 
hay ab + bc + ca  3. PhÂČp chĂčng minh žn „y ho n t§t.2 
1.14 Cho a; b; c; d l  cÂĄc sĂš thĂŒc dĂ·ĂŹng thoÂŁ mÂąn a+b+c+d = 1. TÂŒm giÂĄ trĂ  nhĂ€ nh§t cĂ”a 
biÂșu thĂčc: 
P = 
1 
a2 + b2 + c2 + d2 + 
1 
abc 
+ 
1 
bcd 
+ 
1 
cda 
+ 
1 
dab 
LĂ­i giÂŁi. KÂœ hi»u 
X 
l  tĂȘng hoÂĄn vĂ . TrĂ·Ăźc hžt ta sĂ» döng AM-GM v  giÂŁ thižt Âș cĂą cÂĄc ÂĄnh 
giÂĄ sau: 
abcd  
 
a + b + c + d 
4 
4 
= 
1 
256 
; 
ab + ac + ad + bc + bd + cd  
3(a + b + c + d)2 
8 
= 
3 
8 
: 
Kžt hñp cÂĄc ÂĄnh giÂĄ n y vĂźi b§t ÂŻng thĂčc Cauchy - Schwarz ta suy ra ֖c cÂĄc b§t ÂŻng thĂčc 
sau: 
19
1. 
1 
a2 + b2 + c2 + d2 + 
X 1 
4ab 
 
72 
a2 + b2 + c2 + d2 + 
X 
4ab 
= 
49 
(a + b + c + d)2 + 2 
X 
ab 
 
49 
1 + 2: 3 
8 
= 28; 
2. 7 
X 1 
4ab 
 
7:62 
X 
4ab 
 
7:36 
4: 3 
8 
= 168: 
M°t khÂĄc ÂĄp döng b§t ÂŻng thĂčc AM-GM cho bĂšn sĂš ta l€i cĂą 
X a 
bcd 
r 
 4 
1 
4abcd 
vuut 
 4 
1 
1 
256 
= 64: 
Kžt hñp ba b§t ÂŻng thĂčc vĂža chĂčng minh Ă° tr¶n, ta suy ra 
1 
a2 + b2 + c2 + d2 + 2 
X 1 
ab 
+ 
X a 
bcd 
 28 + 168 + 64 = 260: 
HĂŹn nĂșa, sĂ» döng giÂŁ thižt a + b + c + d = 1 ta suy ra 
P = 
1 
a2 + b2 + c2 + d2 + (a + b + c + d) 
 
1 
abc 
+ 
1 
bcd 
+ 
1 
cda 
+ 
1 
dab 
 
= 
1 
a2 + b2 + c2 + d2 + 2 
X 1 
ab 
+ 
X a 
bcd 
: 
Do vÂȘy P  260. 
CuĂši cĂČng, vĂźi a = b = c = d = 
1 
4 
(thoÂŁ mÂąn i·u ki»n) thÂŒ P = 260 n¶n ta suy ra 260 l  giÂĄ trĂ  
nhĂ€ nh§t cĂ”a biÂșu thĂčc P. 
PhÂČp chĂčng minh ho n t§t.2 
1.15 Cho x; y; z l  cÂĄc sĂš thĂŒc dĂ·ĂŹng thoÂŁ mÂąn xyz = 1. ChĂčng minh rÂŹng: 
18 
 
1 
x3 + 1 
+ 
1 
y3 + 1 
+ 
1 
z3 + 1 
 
 (x + y + z)3 
LĂ­i giÂŁi. SĂ» döng giÂŁ thižt, dÂč th§y b§t ÂŻng thĂčc cŠn chĂčng minh tĂ·ĂŹng Ă·ĂŹng vĂźi mĂ©i b§t ÂŻng 
thĂčc trong dÂąy sau: 
18 
 
3  
x3 
x3 + 1 
 
y3 
y3 + 1 
 
z3 
z3 + 1 
 
 (x + y + z)3; 
18 
 
x2 
x2 + yz 
+ 
y2 
y2 + zx 
+ 
z2 
z2 + xy 
 
+ (x + y + z)3  54: () 
p döng b§t ÂŻng thĂčc Cauchy - Schwarz, ta cĂą 
x2 
x2 + yz 
+ 
y2 
y2 + zx 
+ 
z2 
z2 + xy 
 
(x + y + z)2 
x2 + y2 + z2 + xy + yz + zx 
: 
20
NhĂ· vÂȘy nžu kÂœ hi»u V T() l  vž trÂĄi cĂ”a b§t ÂŻng thĂčc () thÂŒ ta cĂą 
V T()  
18(x + y + z)2 
x2 + y2 + z2 + xy + yz + zx 
+ (x + y + z)3: 
žn „y ta ÂĄp döng b§t ÂŻng thĂčc AM-GM Âș cĂą 
s 
V T()  2 
18(x + y + z)5 
x2 + y2 + z2 + xy + yz + zx 
: 
NhĂ· vÂȘy Âș kžt thĂłc chĂčng minh, ta cŠn chÂż ra rÂŹng 
(x + y + z)5  
81 
2 
(x2 + y2 + z2 + xy + yz + zx): 
TrĂ·Ăźc hžt ta ÂĄp döng b§t ÂŻng thĂčc AM-GM nhĂ· sau: 
(x+y+z)6 = [(x2+y2+z2)+(xy+yz +zx)+(xy+yz +zx)]3  27(x2+y2+z2)(xy+yz +zx)2: 
HĂŹn nĂșa, theo mĂ«t kžt quÂŁ quen thuĂ«c ta cĂą (xy + yz + zx)2  3xyz(x + y + z), do Ăą 
(x + y + z)6  81xyz(x2 + y2 + z2)(x + y + z); 
hay (x + y + z)5  81(x2 + y2 + z2) do xyz = 1. NhĂ· vÂȘy ta cŠn chÂż ra rÂŹng 
2(x2 + y2 + z2)  x2 + y2 + z2 + xy + yz + zx: 
Tuy nhi¶n bÂŹng phÂČp bižn ĂȘi tĂ·ĂŹng Ă·ĂŹng ta thu ֖c 
1 
2 
[(a  b)2 + (b  c)2 + (c  a)2]  0; 
l  mĂ«t b§t ÂŻng thĂčc hiÂșn nhi¶n Ăłng. Do vÂȘy b§t ÂŻng thĂčc ban Šu Âą ֖c chĂčng minh. 
B i to¥n kžt thóc.2 
1.16 Cho a; b; c l  cÂĄc sĂš thĂŒc dĂ·ĂŹng thoÂŁ mÂąn a4 + b4 + c4 = 3. ChĂčng minh rÂŹng: 
a2 
b + c 
+ 
b2 
c + a 
+ 
c2 
a + b 
 
3 
2 
LĂ­i giÂŁi. Ta sÂł i chĂčng minh 
a2 
b + c 
+ 
b2 
c + a 
+ 
c2 
a + b 
 
3 
2 
r 
a4 + b4 + c4 
4 
3 
; 
tĂž Ăą sĂ» döng giÂŁ thižt Âș suy ra kžt luÂȘn cho b i toÂĄn. ThÂȘt vÂȘy, ÂĄp döng b§t ÂŻng thĂčc Holder, 
ta cĂą 
 
a2 
b + c 
+ 
b2 
c + a 
+ 
c2 
a + b 
2 
[a2(b + c)2 + b2(c + a)2 + c2(a + b)2]  (a2 + b2 + c2)3: 
HĂŹn nĂșa, theo mĂ«t kžt quÂŁ quen thuĂ«c, ta cĂą 
2(a2 + b2)  (a + b)2; 
21
tĂž „y ta thižt lÂȘp hai ÂĄnh giÂĄ tĂ·ĂŹng tĂŒ Âș cĂą 
 
a2 
b + c 
+ 
b2 
c + a 
+ 
c2 
a + b 
2 
[2a2(b2 + c2) + 2b2(c2 + a2) + 2c2(a2 + b2)]  (a2 + b2 + c2)3; 
hay 
a2 
b + c 
+ 
b2 
c + a 
+ 
c2 
a + b 
 
1 
2 
s 
(a2 + b2 + c2)3 
a2b2 + b2c2 + c2a2 : 
NhĂ· vÂȘy Âș kžt thĂłc chĂčng minh ta cŠn chÂż ra rÂŹng 
s 
(a2 + b2 + c2)3 
a2b2 + b2c2 + c2a2 
r 
a4 + b4 + c4 
 3 4 
3 
: 
ThĂŒc hi»n phÂČp bižn ĂȘi tĂ·ĂŹng Ă·ĂŹng ta thu ֖c 
(a2 + b2 + c2)6  27(a4 + b4 + c4)(a2b2 + b2c2 + c2a2)2: 
Tuy nhi¶n b§t ÂŻng thĂčc tr¶n Ăłng nžu ta ÂĄp döng b§t ÂŻng thĂčc AM-GM nhĂ· sau: 
(a2 + b2 + c2)6 = [(a4 + b4 + c4) + (a2b2 + b2c2 + c2a2) + (a2b2 + b2c2 + c2a2)]3 
 27(a4 + b4 + c4)(a2b2 + b2c2 + c2a2)2 
PhÂČp chĂčng minh žn „y ho n t§t.2 
1.17 Cho a; b; c l  cÂĄc sĂš thĂŒc dĂ·ĂŹng thoÂŁ mÂąn a + b + c = 3. ChĂčng minh rÂŹng: 
a 
a + b + 1 
+ 
b 
b + c + 1 
+ 
c 
c + a + 1 
 1 
LĂ­i giÂŁi. SĂ» döng giÂŁ thižt, ta th§y rÂŹng cÂĄc b§t ÂŻng thĂčc sau l  tĂ·ĂŹng Ă·ĂŹng vĂźi b§t ÂŻng thĂčc 
cŠn chĂčng minh 
a 
4  c 
+ 
b 
4  a 
+ 
c 
4  b 
 1; 
a(4  a)(4  b) + b(4  b)(4  c) + c(4  c)(4  a)  (4  a)(4  b)(4  c); 
a2b + b2c + c2a + abc  4: 
B§t ÂŻng thĂčc tr¶n mang tÂœnh hoÂĄn vĂ  giĂșa cÂĄc bižn n¶n khĂŠng m§t tÂœnh tĂȘng quÂĄt, ta giÂŁ sĂ» c 
nÂŹm giĂșa a v  b. Khi Ăą 
a(a  c)(b  c)  0: 
ThĂŒc hi»n phÂČp khai triÂșn ta ֖c a2b+c2a  a2c+abc. TĂž „y ta cĂ«ng th¶m €i l֖ng (b2c+abc) 
v o hai vž Âș ֖c 
a2b + b2c + c2a + abc  a2c + b2c + 2abc = c(a + b)2: 
žn „y ta ¥p döng AM-GM nh÷ sau: 
c(a + b)2 = 
1 
2 
2c(a + b)(a + b)  
(2c + a + b + a + b)3 
2:27 
= 4; 
tĂž Ăą suy ra a2b + b2c + c2a + abc  4, tĂčc l  b§t ÂŻng thĂčc ban Šu Âą ֖c chĂčng minh. 
22
B i to¥n ho n t§t.2 
1.18 Cho a; b; c l  cÂĄc sĂš thĂŒc khĂŠng „m thoÂŁ mÂąn a + b + c = 1. ChĂčng minh rÂŹng: 
25 
27 
 (1  4ab)2 + (1  4bc)2 + (1  4ca)2  3 
LĂ­i giÂŁi. 
1. ChĂčng minh (1  4ab)2 + (1  4bc)2 + (1  4ca)2  3. 
Tr÷ßc hžt ta cù 
p 
ab; 
1 = a + b + c  a + b  2 
tĂž Ăą suy ra 1  4ab. žn „y ta sĂ» döng giÂŁ thižt cÂĄc bižn khĂŠng „m Âș cĂą 
0  1  4ab  1; 
tĂž Ăą m  (1  4ab)2  1. Thižt lÂȘp hai ÂĄnh giÂĄ tĂ·ĂŹng tĂŒ v  cĂ«ng l€i ta cĂą ngay i·u phÂŁi 
chĂčng minh. 
2. ChĂčng minh (1  4ab)2 + (1  4bc)2 + (1  4ca)2  
25 
27 
. 
DÂč th§y b§t ÂŻng thĂčc tr¶n tĂ·ĂŹng Ă·ĂŹng vĂźi mĂ©i b§t ÂŻng thĂčc trong dÂąy sau: 
3  8(ab + bc + ca) + 16(a2b2 + b2c2 + c2a2)  
25 
27 
; 
ab + bc + ca  2(a2b2 + b2c2 + c2a2)  
7 
27 
: 
Âș ĂŸ rÂŹng ta cĂą ÂŻng thĂčc sau 
ab  2a2b2  
5 
9 
 
ab  
1 
9 
 
 
7 
81 
= 2 
 
ab  
1 
9 
2 
; 
do Ăą ta suy ra ab  2a2b2  
5 
9 
 
ab  
1 
9 
 
+ 
7 
81 
. žn „y ta thižt lÂȘp hai ÂĄnh giÂĄ tĂ·ĂŹng tĂŒ 
v  cĂ«ng l€i Âș cĂą 
ab + bc + ca  2(a2b2 + b2c2 + c2a2)  
5 
9 
 
ab + bc + ca  
1 
3 
 
 
7 
27 
: 
HĂŹn nĂșa, theo mĂ«t kžt quÂŁ quen thuĂ«c ta cĂą ab+bc+ca  
(a + b + c)2 
3 
= 
1 
3 
, do vÂȘy ta suy 
ra 
ab + bc + ca  2(a2b2 + b2c2 + c2a2)  
7 
27 
; 
tĂčc l  b§t ÂŻng thĂčc ban Šu Âą ֖c chĂčng minh. 
TĂąm l€i ta Âą chĂčng minh ֖c 
25 
27 
 (1  4ab)2 + (1  4bc)2 + (1  4ca)2  3. PhÂČp chĂčng minh 
ho n t§t.2 
1.18 Cho x; y; z l  cÂĄc sĂš thĂŒc dĂ·ĂŹng thoÂŁ mÂąn xy + yz + zx = 1. ChĂčng minh rÂŹng: 
1 
1 + xy + z2 + 
1 
1 + yz + x2 + 
1 
1 + zx + y2 
 
9 
5 
Líi gi£i. °t x = 
1 
a 
; y = 
1 
b 
; z = 
1 
c 
. Khi ù sû döng gi£ thižt xy + yz + zx = 1, ta th§y rng 
23
1 
1 + xy + z2 = 
xy + yz + zx 
x2 + xy + xz + 2yz 
= 
1 
ab + 1 
bc + 1 
ca 
1 
a2 + 1 
ab + 1 
ac + 2 
bc 
= 
a(a + b + c) 
2a2 + ab + bc + ca 
; 
do Ăą b§t ÂŻng thĂčc Âą cho tĂ·ĂŹng Ă·ĂŹng vĂźi 
X a 
2a2 + ab + bc + ca 
 
9 
5(a + b + c) 
: 
Nh„n cÂŁ hai vž cĂ”a b§t ÂŻng thĂčc n y vĂźi ab + bc + ca v  chĂł ĂŸ rÂŹng 
a(ab + bc + ca) 
2a2 + ab + bc + ca 
= a  
2a3 
2a2 + ab + bc + ca 
; 
ta ֖c 
2 
X a3 
2a2 + ab + bc + ca 
+ 
9(ab + bc + ca) 
5(a + b + c) 
 a + b + c: 
p döng b§t ÂŻng thĂčc Cauchy - Schwarz, ta cĂą 
X a3 
2a2 + ab + bc + ca 
 
( 
X 
a2)2 
X 
a(2a2 + ab + bc + ca) 
= 
( 
X 
a2)2 
6abc + ( 
X 
a)(2 
X 
a2  
X 
ab) 
: 
(1) 
M°t khÂĄc, tĂž b§t ÂŻng thĂčc cĂŹ bÂŁn (ab + bc + ca)2  3abc(a + b + c), ta l€i cĂą 
3abc  
(ab + bc + ca)2 
a + b + c 
: (2) 
Kžt hñp (1) v  (2), ta suy ra 
X a3 
2a2 + ab + bc + ca 
 
X 
( 
a2)2( 
X 
a) 
X 
2( 
ab + bc + ca)2 + ( 
X 
a)2(2 
X 
a2  
X 
ab) 
: 
= 
X 
( 
X 
a2)( 
a) 
2 
X 
a2 + 3 
X 
ab 
: 
CuĂši cĂČng ta chÂż cŠn chĂčng minh 
2(a2 + b2 + c2)(a + b + c) 
2(a2 + b2 + c2) + 3(ab + bc + ca) 
+ 
9(ab + bc + ca) 
5(a + b + c) 
 a + b + c: 
Sau khi khai triÂșn v  rĂłt gĂ„n, ta ֖c b§t ÂŻng thĂčc hiÂșn nhi¶n Ăłng 
(ab + bc + ca)(a2 + b2 + c2  ab  bc  ca)  0: 
B i toÂĄn ֖c chĂčng minh xong.2 
24
1.19 Cho a; b; c l  cÂĄc sĂš thĂŒc dĂ·ĂŹng thoÂŁ mÂąn a + b + c = 
1 
a 
+ 
1 
b 
+ 
1 
c 
. ChĂčng minh rÂŹng: 
(b + c  a)(c + a  b)(a + b  c)  1 
LĂ­i giÂŁi 1. B§t ÂŻng thĂčc cŠn chĂčng minh mang tÂœnh Ăši xĂčng giĂșa cÂĄc bižn, do Ăą khĂŠng m§t 
tÂœnh tĂȘng quÂĄt, ta giÂŁ sĂ» a  b  c. Khi Ăą a + b  c  0 v  c + a  b  0. 
Nžu b+ca  0 thÂŒ b§t ÂŻng thĂčc hiÂșn nhi¶n Ăłng do (b+ca)(c+ab)(a+bc)  0  1. Do 
ù ta ch¿ cŠn gi£i quyžt b i to¥n trong tr÷íng hñp b+ca  0. Lóc n y ta °t x = b+ca; y = 
c + a  b; z = a + b  c. Khi ù ta vižt l€i i·u ki»n nh÷ sau 
x; y; z  0; x + y + z = 
2 
x + y 
+ 
2 
y + z 
+ 
2 
z + x 
; 
v  ta cŠn chĂčng minh 
xyz  1: 
Ta sÂł giÂŁi quyžt b i toÂĄn bÂŹng phĂ·ĂŹng phÂĄp phÂŁn chĂčng. ThÂȘt vÂȘy, giÂŁ sĂ» rÂŹng xyz  1. Khi Ăą 
sĂ» döng b§t ÂŻng thĂčc AM-GM, ta suy ra 
x + y + z = 
2 
x + y 
+ 
2 
y + z 
+ 
2 
z + x 
 
1 
p 
xy 
+ 
1 
p 
yz 
+ 
1 
p 
zx 
; 
hay 
p 
x + 
p 
y + 
p 
z  
p 
xyz(x + y + z). HĂŹn nĂșa, ta cĂŽng cĂą xyz  1 n¶n 
p 
x + 
p 
y + 
p 
z  x + y + z: 
Tuy nhi¶n theo b§t ÂŻng thĂčc AM-GM, ta l€i cĂą 
p 
x  
x + 1 
2 
. Ta thižt lÂȘp th¶m hai ÂĄnh giÂĄ 
tĂ·ĂŹng tĂŒ nĂșa Âș cĂą 
x + y + z + 3 
2 
 
p 
x + 
p 
y + 
p 
z  x + y + z; 
hay x + y + z  3. NhĂ·ng „y l  mĂ«t ÂĄnh giÂĄ sai vÂŒ theo mĂ«t kžt quÂŁ quen thuĂ«c, ta cĂą 
x + y + z = 
2 
x + y 
+ 
2 
y + z 
+ 
2 
z + x 
 
9 
x + y + z 
; 
dšn tĂźi x + y + z  3. M„u thušn n y chĂčng tĂ€ i·u giÂŁ sĂ» ban Šu l  sai, do vÂȘy xyz  1. 
PhÂČp chĂčng minh ho n t§t.2 
LĂ­i giÂŁi 2. B§t ÂŻng thĂčc cŠn chĂčng minh mang tÂœnh Ăši xĂčng giĂșa cÂĄc bižn, do Ăą khĂŠng m§t 
tÂœnh tĂȘng quÂĄt, ta giÂŁ sĂ» a  b  c. Khi Ăą a + b  c  0 v  c + a  b  0. 
Nžu b+ca  0 thÂŒ b§t ÂŻng thĂčc hiÂșn nhi¶n Ăłng do (b+ca)(c+ab)(a+bc)  0  1. Do 
ù ta ch¿ cŠn gi£i quyžt b i to¥n trong tr÷íng hñp b+ca  0. Lóc n y ta °t x = b+ca; y = 
c + a  b; z = a + b  c. Khi ù ta vižt l€i i·u ki»n nh÷ sau 
x; y; z  0; x + y + z = 
2 
x + y 
+ 
2 
y + z 
+ 
2 
z + x 
; 
v  ta cŠn chĂčng minh 
xyz  1: 
25
Ta sÂł giÂŁi quyžt b i toÂĄn bÂŹng phĂ·ĂŹng phÂĄp phÂŁn chĂčng. ThÂȘt vÂȘy, giÂŁ sĂ» rÂŹng xyz  1. Khi Ăą, 
tÞ gi£ thižt, ta suy ra 
(x + y + z)2(xy + yz + zx) = 2(x + y + z)2 + 2(xy + yz + zx) + xyz(x + y + z): () 
Tuy nhi¶n, theo b§t ÂŻng thĂčc AM-GM v  theo i·u giÂŁ sĂ» Ă° tr¶n, ta cĂą cÂĄc ÂĄnh giÂĄ 
xy + yz + zx  3 3 p 
x2y2z2  3; 
x + y + z  3 3 p 
xyz  3; 
do vÂȘy ta suy ra 
2(x + y + z)2(xy + yz + zx) 
3 
 2(x + y + z)2; 
2(x + y + z)2(xy + yz + zx) 
9 
 2(xy + yz + zx); 
(x + y + z)2(xy + yz + zx) 
9 
 xyz(x + y + z): 
Cëng vž theo vž c¥c ¥nh gi¥ tr¶n l€i, ta ÷ñc 
(x + y + z)2(xy + yz + zx)  2(x + y + z)2 + 2(xy + yz + zx) + xyz(x + y + z); 
trÂĄi vĂźi (). M„u thušn n y chĂčng tĂ€ i·u giÂŁ sĂ» ban Šu l  sai, do vÂȘy xyz  1. 
PhÂČp chĂčng minh ho n t§t.2 
1.20 Cho a; b; c l  cÂĄc sĂš thĂŒc dĂ·ĂŹng thoÂŁ mÂąn a + b + c = 3. ChĂčng minh rÂŹng: 
1 
5a2 + ab + bc 
+ 
1 
5b2 + bc + ca 
+ 
1 
5c2 + ca + ab 
 
3 
7 
LĂ­i giÂŁi. p döng b§t ÂŻng thĂčc Cauchy - Schwarz, ta cĂą 
1 
5a2 + ab + bc 
+ 
1 
5b2 + bc + ca 
+ 
1 
5c2 + ca + ab 
= 
X 
cyc 
(b + c)2 
(b + c)2(5a2 + ab + bc) 
 
4(a + b + c)2 
X 
cyc 
(b + c)2(5a2 + ab + bc) 
: 
Theo Ăą, ta cŠn chĂčng minh rÂŹng 
4(a + b + c)2 
X 
cyc 
(b + c)2(5a2 + ab + bc) 
 
3 
7 
: 
SĂ» döng giÂŁ thižt a + b + c = 3, ta th§y rÂŹng b§t ÂŻng thĂčc tr¶n tĂ·ĂŹng Ă·ĂŹng vĂźi 
X 
28(a + b + c)4  27[ 
cyc 
(b + c)2(5a2 + ab + bc)]: 
Sau khi khai triÂșn v  rĂłt gĂ„n, ta ֖c 
28 
X 
a4 + 58 
X 
cyc 
a3b + 85 
X 
cyc 
ab3  156 
X 
a2b2 + 15abc(a + b + c): 
26
Âș chĂčng minh b§t ÂŻng thĂčc n y, trĂ·Ăźc hžt ta chĂł ĂŸ žn cÂĄc ÂĄnh giÂĄ cĂŹ bÂŁn sau (thu ֖c bÂŹng 
b§t ÂŻng thĂčc AM-GM): X 
cyc 
a3b + 
X 
cyc 
ab3  2 
X 
a2b2; 
X 
a4 + 
X 
cyc 
ab3  
X 
cyc 
a3b + 
X 
cyc 
ab3  2 
X 
a2b2; 
X 
a4  
X 
a2b2  abc(a + b + c): 
TĂž Ăą ta suy ra 
58 
X 
cyc 
a3b + 58 
X 
cyc 
ab3  116 
X 
a2b2; 
27 
X 
a4 + 27 
X 
cyc 
ab3  54 
X 
a2b2; 
X 
a4 + 14 
X 
a2b2  15abc(a + b + c): 
CĂ«ng vž theo vž cÂĄc ÂĄnh giÂĄ tr¶n, ta thu ֖c b§t ÂŻng thĂčc cŠn chĂčng minh. 
B i to¥n kžt thóc.2 
1.21 Cho a; b; c l  cÂĄc sĂš thĂŒc dĂ·ĂŹng thay ĂȘi b§t kÂŒ. ChĂčng minh rÂŹng: 
b + c 
2a2 + bc 
+ 
c + a 
2b2 + ca 
+ 
a + b 
2c2 + ab 
 
6 
a + b + c 
LĂ­i giÂŁi. Nh„n cÂŁ hai vž cĂ”a b§t ÂŻng thĂčc cho 4(a + b + c), ta ֖c 
4(b + c)(a + b + c) 
2a2 + bc 
+ 
4(c + a)(a + b + c) 
2b2 + ca 
+ 
4(a + b)(a + b + c) 
2c2 + ab 
 24: 
Do 
4(b + c)(a + b + c) 
2a2 + bc 
= 
(a + 2b + 2c)2 
2a2 + bc 
 
a2 
2a2 + bc 
n¶n ta cù 
X(a + 2b + 2c)2 
2a2 + bc 
 24 + 
X a2 
2a2 + bc 
: 
B§t ÂŻng thĂčc n y ֖c suy ra bÂŹng cÂĄch cĂ«ng hai b§t ÂŻng thĂčc 
a2 
2a2 + bc 
+ 
b2 
2b2 + ca 
+ 
c2 
2c2 + ab 
 1; 
(a + 2b + 2c)2 
2a2 + bc 
+ 
(b + 2c + 2a)2 
2b2 + ca 
+ 
(c + 2c + 2b)2 
2c2 + ab 
 25: 
Do 
a2 
2a2 + bc 
= 
1 
2 
 
bc 
2(2a2 + bc) 
n¶n b§t ÂŻng thĂčc thĂč nh§t tĂ·ĂŹng Ă·ĂŹng vĂźi 
bc 
2a2 + bc 
+ 
ca 
2b2 + ca 
+ 
ab 
2c2 + ab 
 1; 
Ăłng vÂŒ theo b§t ÂŻng thĂčc Cauchy - Schwarz 
X bc 
2a2 + bc 
 
X 
bc 
2 
X 
bc(2a2 + bc) 
= 1: 
27
B„y giĂ­ ta sÂł chĂčng minh b§t ÂŻng thĂčc thĂč hai. „y l  b§t ÂŻng thĂčc Ăši xĂčng n¶n khĂŠng m§t 
tÂœnh tĂȘng quÂĄt, ta giÂŁ sĂ» c = minfa; b; cg. °t t = 
b + c 
2 
, ta sÂł chĂčng minh 
(a + 2b + 2c)2 
2a2 + bc 
+ 
(b + 2c + 2a)2 
2b2 + ca 
 
2(3t + 2c)2 
2t2 + tc 
: () 
SĂ» döng b§t ÂŻng thĂčc Cauchy - Schwarz, ta cĂą 
(a + 2b + 2c)2 
2a2 + bc 
+ 
(b + 2c + 2a)2 
2b2 + ca 
 
[b(a + 2b + 2c) + a(b + 2c + 2a)]2 
b2(2a2 + bc) + a2(2b2 + ca) 
= 
2(4t2  ab + 2tc)2 
2a2b2  3abtc + 4t3c 
: 
VŒ tc  ab  t2 n¶n 
2a2b2  3abtc  (2t4  3t3c) = (t2  ab)(2t2 + 2ab  3tc)  0; 
tÞ ù dšn žn 
(a + 2b + 2c)2 
2a2 + bc 
+ 
(b + 2c + 2a)2 
2b2 + ca 
 
2(4t2  ab + 2tc)2 
2a2b2  3abtc + 4t3c 
 
2(3t2 + 2tc)2 
2t4  3t3c + 4t3c 
= 
2(3t + 2c)2 
2t2 + tc 
: 
M°t kh¥c, ta l€i cù 
(c + 2c + 2b)2 
2c2 + ab 
 
(4t + c)2 
t2 + 2c2 : () 
Kžt hñp hai ÂĄnh giÂĄ () v  (), ta Ă·a b i toÂĄn v· vi»c chĂčng minh 
2(3t + 2c)2 
2t2 + tc 
+ 
(4t + c)2 
t2 + 2c2 
 25: 
Sau khi thu gĂ„n, ta ֖c b§t ÂŻng thĂčc hiÂșn nhi¶n Ăłng 
c(31t + 16c)(t  c)2 
t(2t + c)(t2 + 2c2) 
 0: 
B i toÂĄn ֖c chĂčng minh xong.2 
1.22 Cho a; b; c; d l  cÂĄc sĂš thĂŒc khĂŠng „m thĂ€a mÂąn a2+b2+c2+d2 = 1. ChĂčng minh rÂŹng: 
a 
b2 + 1 
+ 
b 
c2 + 1 
+ 
c 
d2 + 1 
+ 
d 
a2 + 1 
 
p 
a + b 
4(a 
p 
b + c 
p 
c + d 
p 
d)2 
5 
LĂ­i giÂŁi. p döng b§t ÂŻng thĂčc Cauchy - Schwarz, ta cĂą 
a 
b2 + 1 
+ 
b 
c2 + 1 
+ 
c 
d2 + 1 
+ 
d 
a2 + 1 
= 
a3 
a2b2 + a2 + 
b3 
b2c2 + b2 + 
c3 
c2d2 + c2 + 
d3 
d2a2 + d2 
 
p 
a + b 
(a 
p 
b + c 
p 
d)2 
p 
c + d 
a2 + b2 + c2 + d2 + a2b2 + b2c2 + c2d2 + a2d2 : 
28
NhĂ· vÂȘy, Âș kžt thĂłc chĂčng minh, ta cŠn chÂż ra rÂŹng 
a2 + b2 + c2 + d2 + a2b2 + b2c2 + c2d2 + a2d2  
5 
4 
; 
hay (a2 + c2)(b2 + d2)  
1 
4 
. Tuy nhi¶n „y l€i l  ÂĄnh giÂĄ Ăłng vÂŒ theo b§t ÂŻng thĂčc AM-GM: 
(a2 + c2)(b2 + d2)  
(a2 + c2 + b2 + d2)2 
4 
= 
1 
4 
; 
do vÂȘy b§t ÂŻng thĂčc ban Šu ֖c chĂčng minh xong. 
B i to¥n kžt thóc.2 
1.23 Cho x,y,z l  cÂĄc sĂš thĂŒc thuĂ«c o€n [0; 1]. ChĂčng minh rÂŹng: 
x 
3 p 
1 + y3 
+ 
y 
3 p 
1 + z3 
+ 
z 
3 p 
1 + x3 
 
3 
3 p 
1 + xyz 
Líi gi£i. Do x; y; z 2 [0; 1] n¶n ta cù 
x 
3 p 
1 + y3 
+ 
y 
3 p 
1 + z3 
+ 
z 
3 p 
1 + x3 
 
1 
3 p 
1 + y3 
+ 
1 
3 p 
1 + z3 
+ 
1 
3 p 
1 + x3 
: 
Âș ĂŸ rÂŹng theo b§t ÂŻng thĂčc Holder, ta ֖c ÂĄnh giÂĄ sau vĂźi mĂ„i sĂš thĂŒc dĂ·ĂŹng a; b; c: 
(a + b + c)3  9(a3 + b3 + c3); 
hay (a + b + c)  3 p 
9(a3 + b3 + c3). Sû döng ¥nh gi¥ n y, ta cù 
1 
3 p 
1 + y3 
+ 
1 
3 p 
1 + z3 
+ 
1 
3 p 
1 + x3 
s 
9 
 3 
 
1 
1 + y3 + 
1 
1 + x3 + 
1 
1 + z3 
 
: 
NhĂ· vÂȘy, Âș kžt thĂłc chĂčng minh, ta cŠn chÂż ra rÂŹng 
1 
1 + y3 + 
1 
1 + x3 + 
1 
1 + z3 
 
3 
1 + xyz 
: () 
Âș ĂŸ rÂŹng vĂźi hai sĂš thĂŒc a; b thay ĂȘi trong o€n [0; 1] ta luĂŠn cĂą 
1 
1 + a2 + 
1 
1 + b2 
 
2 
1 + ab 
= 
(ab  1)(a  b)2 
(1 + a2)(1 + b2)(1 + ab) 
 0: 
Sû döng ¥nh gi¥ n y, ta ÷ñc 
1 
1 + x3 + 
1 
1 + y3 + 
1 
1 + z3 + 
1 
1 + xyz 
 
2 
1 + 
p 
x3y3 
+ 
2 
1 + 
p 
z4xy 
 
4 
1 + xyz 
: 
Do vÂȘy ÂĄnh giÂĄ () ֖c chĂčng minh, dšn žn b§t ÂŻng thĂčc ban Šu Ăłng. 
PhÂČp chĂčng minh ho n t§t.2 
1.24 Cho a; b; c l  cÂĄc sĂš thĂŒc dĂ·ĂŹng thay ĂȘi b§t kÂŒ. ChĂčng minh rÂŹng: 
a2 
b + c 
+ 
b2 
a + c 
+ 
c2 
a + b 
 
a + b + c 
2 
29
LĂ­i giÂŁi 1. p döng b§t ÂŻng thĂčc Cauchy-Schwartz, ta cĂą 
a2 
b + c 
+ 
b2 
a + c 
+ 
c2 
a + b 
 
(a + b + c)2 
2(a + b + c) 
= 
a + b + c 
2 
: 
PhÂČp chĂčng minh ho n t§t. 2 
LĂ­i giÂŁi 2. p döng b§t ÂŻng thĂčc AM-GM cho hai sĂš dĂ·ĂŹng, ta cĂą 
a2 
b + c 
+ 
b + c 
4 
 a: 
CĂ«ng vž theo vž ÂĄnh giÂĄ n y vĂźi hai ÂĄnh giÂĄ tĂ·ĂŹng tĂŒ khÂĄc, ta ֖c: 
a2 
b + c 
+ 
b2 
a + c 
+ 
c2 
a + b 
+ 
a + b + c 
2 
 a + b + c; 
tĂž Ăą ta thu ֖c b§t ÂŻng thĂčc cŠn chĂčng minh. 
B i to¥n kžt thóc.2 
LĂ­i giÂŁi 3. B§t ÂŻng thĂčc ban Šu mang tÂœnh Ăši xĂčng giĂșa cÂĄc bižn, do Ăą khĂŠng m§t tÂœnh tĂȘng 
quÂĄt, ta giÂŁ sĂ» a  b  c. Khi Ăą ta cĂą 
1 
b + c 
 
1 
a + c 
 
1 
a + b 
: 
NhĂ· vÂȘy, theo b§t ÂŻng thĂčc Chebyshev, ta cĂą 
a2 
b + c 
+ 
b2 
a + c 
+ 
c2 
a + b 
 
1 
3 
:(a2 + b2 + c2):( 
1 
a + b 
+ 
1 
b + c 
+ 
1 
a + c 
): 
žn „y ta ¥p döng hai ¥nh gi¥ cÏ b£n x2 + y2 + z2  
(x + y + z)2 
3 
v  
1 
x 
+ 
1 
y 
+ 
1 
z 
 
9 
x + y + z 
Âș cĂą 
a2 
b + c 
+ 
b2 
a + c 
+ 
c2 
a + b 
 
1 
3 
: 
(a + b + c)2 
3 
: 
9 
2(a + b + c) 
= 
a + b + c 
2 
: 
PhÂČp chĂčng minh ho n t§t.2 
1.25 Cho a; b; c l  cÂĄc sĂš thĂŒc dĂ·ĂŹng thay ĂȘi b§t kÂŒ. ChĂčng minh rÂŹng: 
4 p 
3 + a4 + 4 p 
3 + b4 + 4 p 
3 + c4  4 p 
108(a + b + c) 
LĂ­i giÂŁi. p döng b§t ÂŻng thĂčc Holder, ta cĂą 
(1 + 3)(1 + 3)(1 + 3)(a4 + 3)  (a + 3)4; 
tĂž Ăą suy ra 4 p 
3 + a4  
3 + a 
4 p 
64 
. Thižt lÂȘp cÂĄc ÂĄnh giÂĄ tĂ·ĂŹng tĂŒ v  cĂ«ng l€i, ta ֖c 
4 p 
3 + a4 + 4 p 
3 + b4 + 4 p 
3 + c4  
9 + a + b + c 
4 p 
64 
: 
HĂŹn nĂșa, theo b§t ÂŻng thĂčc AM-GM, ta cĂą 
9 + a + b + c = 3 + 3 + 3 + (a + b + c)  4 4 p 
27(a + b + c); 
30
nhĂ· vÂȘy 
4 p 
3 + a4 + 4 p 
3 + b4 + 4 p 
3 + c4  
4 4 p 
27(a + b + c) 
4 p 
64 
= 4 p 
108(a + b + c): 
PhÂČp chĂčng minh ho n t§t.2 
1.26 Cho a; b l  cÂĄc sĂš thĂŒc dĂ·ĂŹng thoÂŁ mÂąn ab  1. ChĂčng minh rÂŹng: 
1 
1 + a2 + 
1 
1 + b2 
 
2 
1 + ab 
LĂ­i giÂŁi. ThĂŒc hi»n phÂČp bižn ĂȘi tĂ·ĂŹng Ă·ĂŹng, ta thu ֖c dÂąy cÂĄc ÂĄnh giÂĄ sau: 
2 + a2 + b2 
a2b2 + a2 + b2 + 1 
 
2 
1 + ab 
; 
2 + 2ab + a3b + b3a + a2 + b2  2a2b2  2a2  2b2  2  0; 
(ab  1)(a  b)2  0: 
ÂĄnh giÂĄ cuĂši cĂČng Ăłng do ab  1, do vÂȘy b§t ÂŻng thĂčc ban Šu ֖c chĂčng minh. 
B i to¥n kžt thóc.2 
1.27 Cho a; b; c l  cÂĄc sĂš thĂŒc dĂ·ĂŹng thoÂŁ mÂąn a + b + c = 1. ChĂčng minh rÂŹng: 
c + ab 
a + b 
+ 
a + bc 
b + c 
+ 
b + ac 
a + c 
 2 
LĂ­i giÂŁi. Âș ĂŸ rÂŹng ta cĂą 
c + ab = c(a + b + c) + ab = (c + a)(c + b); 
do vÂȘy b§t ÂŻng thĂčc cŠn chĂčng minh tĂ·ĂŹng Ă·ĂŹng vĂźi 
(c + a)(c + b) 
a + b 
+ 
(b + a)(b + c) 
a + c 
+ 
(a + b)(a + c) 
b + c 
 2: 
p döng ¥nh gi¥ cÏ b£n x2 + y2 + z2  xy + yz + zx, ta th§y ¥nh gi¥ tr¶n óng do 
(c + a)(c + b) 
a + b 
+ 
(b + a)(b + c) 
a + c 
+ 
(a + b)(a + c) 
b + c 
 b + c + a + b + c + a = 2: 
PhÂČp chĂčng minh ho n t§t.2 
1.28 Cho x; y; z l  cÂĄc sĂš thĂŒc dĂ·ĂŹng thoÂŁ mÂąn 2x + 3y + z = 1. TÂŒm giÂĄ trĂ  nhĂ€ nh§t cĂ”a 
biÂșu thĂčc: 
P = x3 + y3 + z3 
LĂ­i giÂŁi. p döng b§t ÂŻng thĂčc Holder, ta cĂą 
p 
2 + 3 
P(2 
p 
3 + 1)2 = (x3 + y3 + z3)(2 
p 
2 + 3 
p 
3 + 1)(2 
p 
2 + 3 
p 
3 + 1) 
 (2x + 3y + z)3 = 1: 
31
NhĂ· vÂȘy P  
1 
(2 
p 
2 + 3 
p 
3 + 1)2 
. 
CuĂši cĂČng, vĂźi x = 
p 
2 
p 
2 + 3 
2 
p 
3 + 1 
,y = 
p 
3 
p 
2 + 3 
2 
p 
3 + 1 
v  z = 
1 
p 
2 + 3 
2 
p 
3 + 1 
(tho£ m¹n i·u 
ki»n) thŒ P = 
1 
p 
2 + 3 
(2 
p 
3 + 1)2 
n¶n ta kžt luÂȘn 
1 
p 
2 + 3 
(2 
p 
3 + 1)2 
l  giÂĄ trĂ  nhĂ€ nh§t cĂ”a biÂșu 
thĂčc P. 
B i to¥n kžt thóc.2 
1.29 Cho a; b; c l  cÂĄc sĂš thĂŒc thoÂŁ mÂąn a2 +ab+b2 = 3. TÂŒm giÂĄ trĂ  nhĂ€ nh§t v  giÂĄ trĂ  lĂźn 
nh§t cĂ”a biÂșu thĂčc 
P = a2  ab  3b2 
LĂ­i giÂŁi. VĂźi b = 0 thÂŒ tĂž giÂŁ thižt ta suy ra a2 = 3, tĂž Ăą biÂșu thĂčc P cĂą giÂĄ trĂ  l  3. 
VĂźi b6= 0, xÂČt biÂșu thĂčc 
Q = 
P 
3 
= 
a2  ab  3b2 
a2 + ab + b2 = 
x2  x  3 
x2 + x + 1 
; 
trong Ăą x = 
a 
b 
. TÞ „y ta suy ra 
(Q  1)x2 + (Q + 1)x + Q + 3 = 0: 
Coi Ăą l  mĂ«t phĂ·ĂŹng trÂŒnh theo ©n x. XÂČt bi»t thĂčc cĂ”a phĂ·ĂŹng trÂŒnh tr¶n, ta th§y rÂŹng Âș 
phĂ·ĂŹng trÂŒnh tr¶n cĂą nghi»m thÂŒ 
(Q + 1)2  4(Q  1)(Q + 3)  0; 
tÞ „y ta suy ra 
p 
3 
3  4 
3 
 Q  
3 + 4 
p 
3 
3 
. HĂŹn nĂșa, do P = 3Q n¶n ta cĂą 
p 
3  P  3 + 4 
3  4 
p 
3: 
CuĂši cĂČng, vĂźi a =  
p 
2  
p 
3 v  b = 
p 
2 + 
p 
3 thΠP = 3  4 
p 
3; vĂźi a = 
p 
2 + 
p 
3 v  
b =  
p 
2  
p 
3 thΠP = 3 + 4 
p 
3 n¶n ta kžt luÂȘn 3  4 
p 
3 v  3 + 4 
p 
3 lŠn l÷ñt l  gi¥ trà nhÀ 
nh§t v  giÂĄ trĂ  lĂźn nh§t cĂ”a biÂșu thĂčc P. 
B i to¥n kžt thóc.2 
1.30 Cho a; b; c l  cÂĄc sĂš thĂŒc thoÂŁ mÂąn a2 + 2b2 = 3c2. ChĂčng minh rÂŹng: 
1 
a 
+ 
2 
b 
 
3 
c 
LĂ­i giÂŁi. TĂž giÂŁ thižt, ta suy ra (3c)2 = (a2 + 2b2)(1 + 2). TĂž „y ta ÂĄp döng b§t ÂŻng thĂčc 
Cauchy - Schwarz Âș cĂą 
(3c)2  (a + 2b)2; 
tĂž Ăą suy ra 3c  a + 2b. () 
HĂŹn nĂșa, cĂŽng theo b§t ÂŻng thĂčc Cauchy - Schwarz, ta cĂą 
1 
a 
+ 
1 
b 
+ 
1 
b 
 
9 
a + 2b 
: () 
32
Kžt hñp hai ÂĄnh giÂĄ () v  (), ta thu ֖c b§t ÂŻng thĂčc cŠn chĂčng minh. 
B i to¥n kžt thóc.2 
1.31 Cho x; y; z l  cÂĄc sĂš thĂŒc khĂŠng „m thoÂŁ mÂąn x2 + y2 + z2 = 3. TÂŒm giÂĄ trĂ  lĂźn nh§t 
cĂ”a biÂșu thĂčc: 
P = xy + yz + zx + 
5 
x + y + z 
LĂ­i giÂŁi. Âș ĂŸ rÂŹng 
P = 
(x + y + z)2  x2  y2  z2 
2 
+ 
5 
x + y + z 
= 
(x + y + z)2 
2 
+ 
5 
x + y + z 
 
3 
2 
; 
tĂž Ăą °t t = x + y + z, ta Ă·a b i toÂĄn v· vi»c tÂŒm giÂĄ trĂ  lĂźn nh§t cĂ”a biÂșu thĂčc 
Q = t2 + 
10 
t 
: 
Âș ĂŸ rÂŹng tĂž ÂĄnh giÂĄ x2 +y2 +z2  (x+y +z)2  3(x2 +y2 +z2), ta suy ra 
p 
3  t  3, do vÂȘy 
t2 + 
10 
t 
 
37 
3 
= 
(t  3)(3t2 + 9t  10) 
3t 
 0: 
NhĂ· vÂȘy Q  
37 
3 
, v  vŒ P = 
Q 
2 
 
3 
2 
n¶n 
P  
37 
6 
 
3 
2 
= 
14 
3 
: 
CuĂši cĂČng, vĂźi x = y = z = 1 (thoÂŁ mÂąn i·u ki»n) thÂŒ P = 
14 
3 
n¶n ta kžt luÂȘn 
14 
3 
l  gi¥ trà lßn 
nh§t cĂ”a biÂșu thĂčc P. 
B i to¥n kžt thóc.2 
1.32 Cho x; y l  cÂĄc sĂš thĂŒc dĂ·ĂŹng thoÂŁ mÂąn 2y  x. ChĂčng minh rÂŹng: 
1 
x3(2y  x) 
+ x2 + y2  3 
Líi gi£i. Ta th§y rng 
1 
x3(2y  x) 
+ x2 + y2 = 
1 
x2(2xy  x2) 
+ +x2 + (y2 + x2  x2); 
v  vÂŒ x2 + y2  2xy theo b§t ÂŻng thĂčc AM-GM n¶n 
1 
x3(2y  x) 
+ x2 + y2  
1 
x2(2xy  x2) 
+ x2 + (2xy  x2): 
žn „y ta ÂĄp döng b§t ÂŻng thĂčc AM-GM mĂ«t lŠn nĂșa Âș cĂą 
1 
x3(2y  x) 
s 
+ x2 + y2  3 3 
1 
x2(2xy  x2) 
:x2:(2xy  x2) = 3: 
PhÂČp chĂčng minh ho n t§t.2 
33
1.33 Cho a; b; c l  cÂĄc sĂš thĂŒc dĂ·ĂŹng thoÂŁ mÂąn ab + bc + ca = 2abc. ChĂčng minh rÂŹng: 
1 
a(2a  1)2 + 
1 
b(2b  1)2 + 
1 
c(2c  1)2 
 
1 
2 
Líi gi£i. °t m = 
1 
a 
; n = 
1 
b 
; p = 
1 
c 
. Khi Ăą i·u ki»n Âą cho tĂ·ĂŹng Ă·ĂŹng vĂźi m + n + p = 2 (Âș 
ĂŸ rÂŹng tĂž „y ta cĂą m; n; p  2), v  b§t ÂŻng thĂčc Âą cho ֖c vižt l€i th nh 
m3 
(2  m)2 + 
n3 
(2  n)2 + 
p3 
(2  p)2 
 
1 
2 
: 
p döng b§t ÂŻng thĂčc AM-GM, ta cĂą 
m3 
(2  m)2 + 
2  m 
8 
+ 
2  m 
8 
 
3m 
4 
; 
tĂž Ăą suy ra 
m3 
(2  m)2 
 m  
1 
2 
. Thižt lÂȘp hai ÂĄnh giÂĄ tĂ·ĂŹng tĂŒ cho n v  p v  cĂ«ng l€i, ta ֖c 
m3 
(2  m)2 + 
n3 
(2  n)2 + 
p3 
(2  p)2 
 m + n + p  
3 
2 
= 
1 
2 
: 
PhÂČp chĂčng minh ho n t§t.2 
1.34 Cho a; b; c l  cÂĄc sĂš thĂŒc khĂŠng „m thoÂŁ mÂąn a + 2b + 3c = 4. ChĂčng minh rÂŹng: 
(a2b + b2c + c2a + abc)(ab2 + bc2 + ca2 + abc)  8 
LĂ­i giÂŁi. p döng b§t ÂŻng thĂčc AM-GM, ta cĂą 
8(a2b + b2c + c2a + abc)(ab2 + bc2 + ca2 + abc) = 4(a2b + b2c + c2a + abc):2(ab2 + bc2 + ca2 + abc) 
 (a2b + b2c + c2a + 2ab2 + 2bc2 + 2ca2 + 3abc)2: 
HĂŹn nĂșa, ta cĂŽng cĂą 
(a + 2b)(b + 2c)(c + 2a) = 9abc + 2a2b + 2ac2 + 4a2c + 2b2c + 4b2a + 4c2b 
 2(a2b + b2c + c2a + 2ab2 + 2bc2 + 2ca2 + 3abc); 
do vÂȘy 8(a2b + b2c + c2a + abc)(ab2 + bc2 + ca2 + abc)  
 
(a + 2b)(b + 2c)(c + 2a) 
2 
2 
. M°t kh¥c, 
theo b§t ÂŻng thĂčc AM-GM, ta cĂą 
4(a + 2b)(b + 2c)(c + 2a) = (a + 2b)(4b + 8c)(c + 2a)  
 
3a + 6b + 9c 
3 
3 
= (a + 2b + 3c)3 = 64: 
NhĂ· vÂȘy, ta suy ra 
8(a2b + b2c + c2a + abc)(ab2 + bc2 + ca2 + abc)  
 
64 
4:2 
2 
= 64; 
hay (a2b + b2c + c2a + abc)(ab2 + bc2 + ca2 + abc)  8. 
34
PhÂČp chĂčng minh ho n t§t.2 
1.35 Cho a; b; c l  cÂĄc sĂš thĂŒc dĂ·ĂŹng thay ĂȘi b§t kÂŒ. ChĂčng minh rÂŹng: 
ab 
a + 3b + 2c 
+ 
bc 
b + 3c + 2a 
+ 
ac 
c + 3a + 2b 
 
a + b + c 
6 
Líi gi£i. Sû döng ¥nh gi¥ cÏ b£n 
9 
x + y + z 
 
1 
x 
+ 
1 
y 
+ 
1 
z 
, ta cĂą 
9 
a + 3b + 2c 
= 
9 
(a + c) + (b + c) + 2b 
 
1 
a + c 
+ 
1 
b + c 
+ 
1 
2b 
: 
TĂž Ăą ta suy ra 
9ab 
a + 3b + 2c 
 
ab 
a + c 
+ 
ab 
b + c 
+ 
a 
2 
. Ho n to n tĂ·ĂŹng tĂŒ, ta cĂŽng cĂą 
9bc 
b + 3c + 2a 
 
bc 
b + a 
+ 
bc 
c + a 
+ 
b 
2 
; 
v  
9ca 
c + 3a + 2b 
 
ca 
c + b 
+ 
ca 
a + b 
+ 
c 
2 
: 
Cëng vž theo vž c¥c ¥nh gi¥ tr¶n, ta thu ÷ñc 
9ab 
a + 3b + 2c 
+ 
9bc 
b + 3c + 2a 
+ 
9ca 
c + 3a + 2b 
 
ca + ab 
b + c 
+ 
ab + bc 
a + c 
+ 
bc + ca 
b + a 
+ 
a + b + c 
2 
= 
3(a + b + c) 
2 
; 
tĂž „y ta ֖c b§t ÂŻng thĂčc cŠn chĂčng minh. 
B i to¥n kžt thóc.2 
1.36 Cho a; b; c l  cÂĄc sĂš thĂŒc dĂ·ĂŹng thoÂŁ mÂąn abc = 1. ChĂčng minh rÂŹng: 
1 
a + b + 4 
+ 
1 
b + c + 4 
+ 
1 
c + a + 4 
 
1 
2 
Líi gi£i 1. °t x = 
p 
a; y = 
p 
b; z = 
p 
c. Khi Ăą ta phÂŁi chĂčng minh 
1 
x2 + y2 + 4 
+ 
1 
y2 + z2 + 4 
+ 
1 
z2 + x2 + 4 
 
1 
2 
vßi x; y; z  0 v  xyz = 1. 
Do 
1 
x2 + y2 + 4 
= 1  
x2 + y2 
x2 + y2 + 4 
= 1  
(x + y)2 + (x  y)2 
2(x2 + y2 + 4) 
n¶n b§t ÂŻng thĂčc n y cĂą thÂș ֖c 
vižt l€i th nh X (x + y)2 
x2 + y2 + 4 
+ 
X (x  y)2 
x2 + y2 + 4 
 2: 
KhĂŠng m§t tÂœnh tĂȘng quÂĄt, ta giÂŁ sĂ» x  y  z. SĂ» döng b§t ÂŻng thĂčc Cauchy - Schwarz, ta cĂą 
X (x + y)2 
x2 + y2 + 4 
 
[(x + y) + (y + z) + (z + x)]2 
X 
(x2 + y2 + 4) 
; 
v  X (x  y)2 
x2 + y2 + 4 
 
[x  y + y  z + x  z]2 
X 
(x2 + y2 + 4) 
: 
35
TĂž „y ta Ă·a b i toÂĄn v· chĂčng minh 
2(x + y + z)2 + 2(x  z)2  2(x2 + y2 + z2) + 12; 
hay 2(x  z)2 + 4(xy + yz + zx  3)  0. Tuy nhi¶n „y l€i l  ¥nh gi¥ óng do (x  z)2  0 v  
theo b§t ÂŻng thĂčc AM-GM thÂŒ 
xy + yz + zx  3 3 p 
x2y2z2 = 3; 
do vÂȘy b§t ÂŻng thĂčc ban Šu ֖c chĂčng minh xong. 
B i to¥n kžt thóc.2 
Líi gi£i 2. °t x = 3 p 
a; y = 3 p 
b; z = 3 p 
c. Khi Ăą x; y; z  0; xyz = 1 v  ta cŠn chĂčng minh 
1 
x3 + y3 + 4 
+ 
1 
y3 + z3 + 4 
+ 
1 
z3 + x3 + 4 
 
1 
2 
VĂźi chĂł ĂŸ ta cĂą ÂĄnh giÂĄ x3 + y3  xy(x + y), çng thĂ­i l€i cĂą 4 = 4xyz, ta Ă·a b i toÂĄn v· vi»c 
chĂčng minh 
1 
xy(x + y + 4z) 
+ 
1 
yz(y + z + 4x) 
+ 
1 
zx(z + x + 4y) 
 
1 
2 
; 
hay 
x + y 
x + y + 4z 
+ 
y + z 
y + z + 4x 
+ 
z + x 
z + x + 4y 
 1: 
p döng b§t ÂŻng thĂčc Cauchy-Schwartz, ta cĂą 
x + y 
x + y + 4z 
+ 
y + z 
y + z + 4x 
+ 
z + x 
z + x + 4y 
 
4(x + y + z)2 
X 
(x + y)(x + y + 4z) 
= 
4(x + y + z)2 
2(x2 + y2 + z2) + 10(xy + yz + zx) 
; 
nhĂ· vÂȘy, Âș kžt thĂłc chĂčng minh, ta cŠn chÂż ra rÂŹng 
4(x + y + z)2  2(x2 + y2 + z2) + 10(xy + yz + zx); 
hay x2 + y2 + z2  xy + yz + zx. Tuy nhi¶n „y l€i l  mĂ«t ÂĄnh giÂĄ Ăłng, do vÂȘy b§t ÂŻng thĂčc 
ban Šu ֖c chĂčng minh xong. 
B i to¥n kžt thóc.2 
1.37 Cho a; b; c l  cÂĄc sĂš thĂŒc dĂ·ĂŹng thoÂŁ mÂąn ab + bc + ca = 1. ChĂčng minh rÂŹng: 
r 
3 
1 
a 
r 
+ 6b + 3 
1 
b 
r 
+ 6c + 3 
1 
c 
+ 6a  
1 
abc 
LĂ­i giÂŁi. p döng b§t ÂŻng thĂčc Holder ta cĂą 
  
r 
3 
1 
a 
r 
+ 6b + 3 
1 
b 
r 
+ 6c + 3 
1 
c 
!3 
+ 6a 
 
 
1 
a 
+ 6b + 
1 
b 
+ 6c + 
1 
c 
 
+ 6a 
1 
p 
3 
 
:3 
1 
p 
3 
 
:3 
= 9 
 
1 
a 
+ 
1 
b 
+ 
1 
c 
+ 6a + 6b + 6c 
 
: () 
36
HĂŹn nĂșa, sĂ» döng ÂĄnh giÂĄ cĂŹ bÂŁn xy + yz + zx  
(x + y + z)2 
3 
, ta cĂą 
abc 
 
1 
a 
+ 
1 
b 
+ 
1 
c 
+ 6a + 6b + 6c 
 
= ab + bc + ca + 6abc(a + b + c) 
 ab + bc + ca + 2(ab + bc + ca)2 = 3; 
do vÂȘy 
1 
a 
+ 
1 
b 
+ 
1 
c 
+ 6a + 6b + 6c  
3 
abc 
. Kžt hñp vßi ¥nh gi¥ () ð tr¶n, ta ÷ñc 
  
r 
3 
1 
a 
r 
+ 6b + 3 
1 
b 
r 
+ 6c + 3 
1 
c 
!3 
+ 6a 
 
27 
abc 
; 
tĂž Ăą ta l§y c«n bÂȘc ba hai vž Âș thu ֖c b§t ÂŻng thĂčc cŠn chĂčng minh. 
B i to¥n kžt thóc.2 
1.38 Cho a; b; c l  cÂĄc sĂš thĂŒc dĂ·ĂŹng thoÂŁ mÂąn abc = 1. ChĂčng minh rÂŹng: 
1 
1 + a + b 
+ 
1 
1 + b + c 
+ 
1 
1 + a + c 
 1 
Líi gi£i. °t x = 
p 
a; y = 
p 
b; z = 
p 
c. Khi Ăą ta phÂŁi chĂčng minh 
1 
x2 + y2 + 1 
+ 
1 
y2 + z2 + 1 
+ 
1 
z2 + x2 + 1 
 1 
vßi x; y; z  0 v  xyz = 1. 
Do 
1 
x2 + y2 + 1 
= 1  
x2 + y2 
x2 + y2 + 1 
= 1  
(x + y)2 + (x  y)2 
2(x2 + y2 + 1) 
n¶n b§t ÂŻng thĂčc n y cĂą thÂș ֖c 
vižt l€i th nh X (x + y)2 
x2 + y2 + 1 
+ 
X (x  y)2 
x2 + y2 + 1 
 4: 
KhĂŠng m§t tÂœnh tĂȘng quÂĄt, ta giÂŁ sĂ» x  y  z. SĂ» döng b§t ÂŻng thĂčc Cauchy - Schwarz, ta cĂą 
X (x + y)2 
x2 + y2 + 1 
 
[(x + y) + (y + z) + (z + x)]2 
X 
(x2 + y2 + 1) 
; 
v  X (x  y)2 
x2 + y2 + 1 
 
[x  y + y  z + x  z]2 
X 
(x2 + y2 + 1) 
: 
TĂž „y ta Ă·a b i toÂĄn v· chĂčng minh 
(x + y + z)2 + (x  z)2  2(x2 + y2 + z2) + 3: 
M°t khÂĄc, theo b§t ÂŻng thĂčc AM-GM, ta l€i cĂą 
3 = 3 3 p 
x2y2z2  xy + yz + zx; 
do vÂȘy ta chÂż cĂĄn phÂŁi chĂčng minh 
(x + y + z)2 + (x  z)2  2(x2 + y2 + z2) + xy + yz + zx: 
37
Sau khi thu gĂ„n, ta ֖c b§t ÂŻng thĂčc hiÂșn nhi¶n Ăłng 
(x  y)(y  z)  0: 
B i toÂĄn do Ăą ֖c chĂčng minh xong.2 
1.39 Cho a; b; c l  cÂĄc sĂš thĂŒc dĂ·ĂŹng thoÂŁ mÂąn abc = 1. ChĂčng minh rÂŹng: 
a3 
b(c + 2) 
+ 
b3 
c(a + 2) 
+ 
c3 
a(b + 2) 
 1 
LĂ­i giÂŁi. p döng b§t ÂŻng thĂčc AM-GM, ta cĂą 
a3 
b(c + 2) 
+ 
b 
3 
+ 
c + 2 
9 
 a: 
LÂȘp hai b§t ÂŻng thĂčc tĂ·ĂŹng tĂŒ v  cĂ«ng l€i, ta ֖c 
a3 
b(c + 2) 
+ 
b3 
c(a + 2) 
+ 
c3 
a(b + 2) 
+ 
a + b + c 
3 
+ 
a + b + c + 6 
9 
 a + b + c; 
hay 
a3 
b(c + 2) 
+ 
b3 
c(a + 2) 
+ 
c3 
a(b + 2) 
 
5(a + b + c) 
9 
 
2 
3 
: 
M°t khÂĄc cĂŽng theo b§t ÂŻng thĂčc AM-GM thÂŒ a + b + c  3 3 p 
abc = 3, do vÂȘy 
a3 
b(c + 2) 
+ 
b3 
c(a + 2) 
+ 
c3 
a(b + 2) 
 
5 
3 
 
2 
3 
= 1: 
PhÂČp chĂčng minh ho n t§t.2 
1.40 Cho a; b; c l  cÂĄc sĂš thĂŒc khĂŠng „m thay ĂȘi b§t kÂŒ. ChĂčng minh rÂŹng: 
a3 + b3 + c3 + 3abc  ab(a + b) + bc(b + c) + ca(c + a) 
LĂ­i giÂŁi. B§t ÂŻng thĂčc ban Šu mang tÂœnh Ăši xĂčng giĂșa cÂĄc bižn, n¶n khĂŠng m§t tÂœnh tĂȘng 
quÂĄt, ta giÂŁ sĂ» a = maxfa; b; cg. Khi Ăą thĂŒc hi»n bižn ĂȘi tĂ·ĂŹng Ă·ĂŹng, ta thu ֖c dÂąy b§t 
ÂŻng thĂčc tĂ·ĂŹng Ă·ĂŹng vĂźi b§t ÂŻng thĂčc cŠn chĂčng minh 
a(a  b)(a  c) + b(b  a)(b  c) + c(c  a)(c  b)  0; 
(a  b)(a2  ac  b2 + bc) + c(a  c)(b  c)  0; 
(a  b)2(a + b  c) + c(a  c)(b  c)  0: 
ÂĄnh giÂĄ cuĂši cĂČng Ăłng do a = maxfa; b; cg, do vÂȘy b§t ÂŻng thĂčc ban Šu ֖c chĂčng minh 
xong. 
B i to¥n kžt thóc.2 
38
3.2 B i 2.1 žn b i 2.40 
2.1 Cho a; b; c l  cÂĄc sĂš thĂŒc dĂ·ĂŹng thay ĂȘi b§t kÂŒ. ChĂčng minh rÂŹng: 
a 
(b + c)2 + 
b 
(a + c)2 + 
c 
(a + b)2 
 
9 
4(a + b + c) 
LĂ­i giÂŁi. B§t ÂŻng thĂčc ban Šu tĂ·ĂŹng Ă·ĂŹng vĂźi 
(a + b + c) 
 
a 
(b + c)2 + 
b 
(a + c)2 + 
c 
(a + b)2 
 
 
9 
4 
: 
°t k = 
a 
b + c 
+ 
b 
a + c 
+ 
c 
a + b 
. Ta th§y rng 
(a+b+c) 
 
a 
(b + c)2 + 
b 
(a + c)2 + 
c 
(a + b)2 
 
= 
a2 
(b + c)2 + 
b2 
(a + c)2 + 
c2 
(a + b)2 + 
a 
b + c 
+ 
b 
a + c 
+ 
c 
a + b 
; 
v  theo mĂ«t ÂĄnh giÂĄ quen thuĂ«c thÂŒ 
a2 
(b + c)2 + 
b2 
(a + c)2 + 
c2 
(a + b)2 
 
k2 
3 
, do vÂȘy 
(a + b + c) 
 
a 
(b + c)2 + 
b 
(a + c)2 + 
c 
(a + b)2 
 
 
k2 
3 
+ k 
Ta l€i cĂą chĂł ĂŸ rÂŹng k  
3 
2 
theo b§t ÂŻng thĂčc Nesbitt, do Ăą 
(a + b + c) 
 
a 
(b + c)2 + 
b 
(a + c)2 + 
c 
(a + b)2 
 
 
9 
4:3 
+ 
3 
2 
= 
9 
4 
: 
PhÂČp chĂčng minh ho n t§t.2 
2.2 Cho a; b; c l  cÂĄc sĂš thĂŒc dĂ·ĂŹng thay ĂȘi b§t kÂŒ. ChĂčng minh rÂŹng: 
a 
p 
a2 + 8bc 
+ 
b 
p 
b2 + 8ac 
+ 
c 
p 
c2 + 8ab 
 1 
LĂ­i giÂŁi. p döng b§t ÂŻng thĂčc Cauchy-Schwartz ta cĂą 
a 
p 
a2 + 8bc 
+ 
b 
p 
b2 + 8ac 
+ 
c 
p 
c2 + 8ab 
 
(a + b + c)2 
p 
a2 + 8bc + b 
a 
p 
b2 + 8ac + c 
p 
c2 + 8ab 
: 
M°t khÂĄc, cĂŽng theo b§t ÂŻng thĂčc Cauchy - Schwarz, ta cĂą 
p 
a2 + 8bc + b 
a 
p 
b2 + 8ac + c 
p 
c2 + 8ab = 
p 
a3 + 8abc + 
p 
a 
p 
b 
p 
b3 + 8abc + 
p 
c 
p 
c3 + 8abc 
 
p 
(a + b + c)(a3 + b3 + c3 + 24abc); 
do vÂȘy 
a 
p 
a2 + 8bc 
+ 
b 
p 
b2 + 8ac 
+ 
c 
p 
c2 + 8ab 
 
(a + b + c)2 
p 
(a + b + c)(a3 + b3 + c3 + 24abc) 
= 
s 
(a + b + c)3 
a3 + b3 + c3 + 24abc 
: 
39
NhĂ· vÂȘy, Âș kžt thĂłc chĂčng minh, ta cŠn chÂż ra rÂŹng 
(a + b + c)3  a3 + b3 + c3 + 24abc; 
hay (a + b)(b + c)(c + a)  8abc. Tuy nhi¶n „y l  mĂ«t ÂĄnh giÂĄ Ăłng vÂŒ theo b§t ÂŻng thĂčc 
AM-GM, ta cĂą 
p 
ab:2 
(a + b)(b + c)(c + a)  2 
p 
bc:2 
p 
ca = 8abc; 
do vÂȘy b§t ÂŻng thĂčc ban Šu ֖c chĂčng minh xong. 
B i to¥n kžt thóc.2 
2.3 Cho a; b; c l  cÂĄc sĂš thĂŒc dĂ·ĂŹng thoÂŁ mÂąn çng thĂ­i c  a v  3a2+4b2+5c2 = 12. ChĂčng 
minh rÂŹng: 
1 
a 
+ 
1 
b 
+ 
1 
c 
 3 
Líi gi£i. TÞ gi£ thižt, ta cù 
4a2 + 4b2 + 4c2 = 12 + a2  c2  12; 
nhĂ· vÂȘy a2 + b2 + c2  3. TĂž „y ta cĂŽng cĂą 
a + b + c  
p 
3(a2 + b2 + c2)  3; 
v  vÂŒ vÂȘy ta chĂčng minh ֖c b§t ÂŻng thĂčc ban Šu vÂŒ 
1 
a 
+ 
1 
b 
+ 
1 
c 
 
9 
a + b + c 
 
9 
3 
= 3: 
B i to¥n kžt thóc.2 
2.4 Cho a; b; c l  cÂĄc sĂš thĂŒc dĂ·ĂŹng thay ĂȘi b§t kÂŒ. ChĂčng minh rÂŹng: 
a 
b 
+ 
b 
c 
+ 
c 
a 
 
a + c 
b + c 
+ 
b + a 
c + a 
+ 
c + b 
a + b 
Líi gi£i 1. °t 
X = 
a 
b 
2 
1 + 
; Y = 
1 + b 
c 
2 
; Z = 
1 + c 
a 
2 
: 
SĂ» döng b§t ÂŻng thĂčc Holder, ta thu ֖c 
 
1 + 
a 
b 
 
1 + 
b 
c 
 
1 + 
c 
a 
 
 
  
1 + 3 
r 
a 
b 
: 
b 
c 
: 
c 
a 
!3 
= 8; 
tĂž Ăą ta suy ra XY Z  1. 
B„y giĂ­ ta thĂŒc hi»n bižn ĂȘi b§t ÂŻng thĂčc Âą cho nhĂ· sau 
 
a 
b 
 
a + c 
b + c 
 
+ 
 
b 
c 
 
b + a 
c + a 
 
+ 
 
c 
a 
 
c + b 
a + b 
 
 0; 
c(a  b) 
b(b + c) 
+ 
a(b  c) 
c(c + a) 
+ 
b(c  a) 
a(a + b) 
 0; 
40
a 
b 
 1 
1 + b 
c 
+ 
b 
c 
 1 
1 + 
c 
a 
+ 
c 
a  1 
1 + 
a 
b 
 0: 
Âș ĂŸ rÂŹng 
a 
b 
 1 
1 + b 
c 
= 
2X  1  1 
2Y 
= 
X  1 
Y 
; 
do vÂȘy b§t ÂŻng thĂčc cuĂši cĂą thÂș vižt l€i th nh 
X  1 
Y 
+ 
Y  1 
Z 
+ 
Z  1 
X 
 0; 
tĂ·ĂŹng Ă·ĂŹng 
X 
Y 
+ 
Y 
Z 
+ 
Z 
X 
 
1 
X 
+ 
1 
Y 
+ 
1 
Z 
: 
SĂ» döng b§t ÂŻng thĂčc AM-GM, ta cĂą 
3 
XX 
Y 
= 
X 
X 
Y 
+ 
X 
Y 
+ 
Z 
X 
 
 3 
X 
r 
3 
ZX 
Y 2 = 3 3 p 
XY Z 
X 1 
Y 
= 3 
X 1 
Y 
: 
NhĂ· vÂȘy b§t ÂŻng thĂčc ban Šu ֖c chĂčng minh xong. 
B i to¥n kžt thóc.2 
LĂ­i giÂŁi 2. ThĂŒc hi»n bižn ĂȘi tĂ·ĂŹng tĂŒ nhĂ· cÂĄch 1, ta cŠn chĂčng minh 
c(a  b) 
b(b + c) 
+ 
a(b  c) 
c(c + a) 
+ 
b(c  a) 
a(a + b) 
 0: 
KhĂŠng m§t tÂœnh tĂȘng quÂĄt, ta giÂŁ sĂ» b l  sĂš nÂŹm giĂșa a v  c. Khi Ăą (ba)(bc)  0. Âș ĂŸ rÂŹng 
b(c  a) = c(a  b)  a(b  c); 
vÂŒ vÂȘy b§t ÂŻng thĂčc tr¶n cĂą thÂș vižt l€i th nh 
c(a  b) 
 
1 
b(b + c) 
 
1 
a(a + b) 
 
+ a(b  c) 
 
1 
c(c + a) 
 
1 
a(a + b) 
 
 0; 
tĂ·ĂŹng Ă·ĂŹng 
c[(a  b)2(a + b) + b(a  b)(a  c)] 
ab(a + b)(b + c) 
+ 
[(b  c)(a  c)(a + c) + a(b  c)2] 
c(c + a)(a + b) 
 0: 
B§t ÂŻng thĂčc cuĂši n y Ăłng do 
(a  b)(a  c) = (a  b)2  (b  a)(b  c)  0; 
v  
(b  c)(a  c) = (b  c)2  (b  a)(b  c)  0; 
do vÂȘy b§t ÂŻng thĂčc ban Šu ֖c chĂčng minh xong. 
B i to¥n kžt thóc.2 
NhÂȘn xÂČt. 
41
1. LĂ·u ĂŸ rÂŹng b§t ÂŻng thĂčc sau Ăłng vĂźi a; b; c v  k l  cÂĄc sĂš thĂŒc dĂ·ĂŹng: 
a 
b 
+ 
b 
c 
+ 
c 
a 
 
ka + c 
kb + c 
+ 
kb + c 
kc + a 
+ 
kc + b 
ka + b 
: 
Vßi k = 1, ta thu ÷ñc b i to¥n tr¶n. 
2. Ri¶ng vĂźi trĂ·Ă­ng hñp k = 1, ta cĂą thÂș chĂčng minh b i toÂĄn dĂŒa tr¶n b§t ÂŻng thĂčc sau („y 
l  mĂ«t b i trong Belarusian Mathematical Olympiad 1998): Cho a; b; c l  cÂĄc sĂš thĂŒc dĂ·ĂŹng. 
ChĂčng minh rÂŹng 
a 
b 
+ 
b 
c 
+ 
c 
a 
 
a + b 
b + c 
+ 
b + c 
a + b 
+ 1: 
Vi»c chĂčng minh cĂŽng nhĂ· ÂĄp döng xin Âș d nh cho b€n Ă„c. 
2.5 Cho a; b; c l  cÂĄc sĂš thĂŒc dĂ·ĂŹng thoÂŁ mÂąn a + b + c = 3. ChĂčng minh rÂŹng: 
p 
a + 
p 
b + 
p 
c  ab + bc + ca 
LĂ­i giÂŁi. B§t ÂŻng thĂčc Âą cho tĂ·ĂŹng Ă·ĂŹng vĂźi 
a2 + b2 + c2 + 2 
p 
a + 2 
p 
b + 2 
p 
c  a2 + b2 + c2 + 2(ab + bc + ca) = 9: 
p döng b§t ÂŻng thĂčc AM-GM, ta cĂą 
a2 + 2 
p 
a  3a: 
LÂȘp cÂĄc b§t ÂŻng thĂčc tĂ·ĂŹng tĂŒ v  cĂ«ng l€i, ta ֖c 
a2 + b2 + c2 + 2 
p 
a + 2 
p 
b + 2 
p 
c  3(a + b + c) = 9: 
PhÂČp chĂčng minh ho n t§t.2 
2.6 Cho a; b; c l  cÂĄc sĂš thĂŒc dĂ·ĂŹng thoÂŁ mÂąn ab + bc + ca = 1. ChĂčng minh rÂŹng: 
1 
abc 
+ 
4 
(a + b)(b + c)(c + a) 
 
p 
3 
2 
9 
LĂ­i giÂŁi. p döng b§t ÂŻng thĂčc AM-GM, ta cĂą 
1 
abc 
+ 
4 
(a + b)(b + c)(c + a) 
= 
1 
2abc 
+ 
1 
2abc 
+ 
4 
(a + b)(b + c)(c + a) 
s 
 3 3 
1 
a2b2c2(a + b)(b + c)(c + a) 
s 
= 3 3 
1 
abc(ab + ac)(bc + ba)(ca + cb) 
: 
M°t khÂĄc, cĂŽng theo b§t ÂŻng thĂčc AM-GM, ta cĂą hai ÂĄnh giÂĄ: 
a2b2c2  
(ab + bc + ca)3 
27 
; 
v  
(ab + bc)(bc + ca)(ca + ab)  
8(ab + bc + ca)3 
27 
; 
42
tÞ ù sû döng gi£ thižt ta suy ra abc  
1 
3 
p 
3 
v  (ab + bc)(bc + ca)(ca + ab)  
8 
27 
. Do vÂȘy 
1 
abc 
+ 
4 
(a + b)(b + c)(c + a) 
 3 
s 
3 
p 
3 
8 
27:3 
= 
p 
3 
2 
9 
: 
PhÂČp chĂčng minh ho n t§t.2 
2.7 Cho x; y; z l  cÂĄc sĂš thĂŒc thoÂŁ mÂąn x + y + z = 0, trong Ăą cĂą hai sĂš cĂČng d§u. ChĂčng 
minh rÂŹng: 
(x2 + y2 + z2)3 
(x3 + y3 + z3)2 
 6 
LĂ­i giÂŁi. KhĂŠng m§t tÂœnh tĂȘng quÂĄt, giÂŁ sĂ» x; y l  hai sĂš cĂČng d§u, tĂčc l  xy  0. VĂźi i·u ki»n 
z = x  y, ta cĂą 
(x2 + y2 + z2)3 
(x3 + y3 + z3)2 = 
8(x2 + y2 + xy)3 
9x2y2(x + y)2 : 
NhĂ· vÂȘy, nžu ta °t x2 + y2 = m v  xy = n (Âș ĂŸ rÂŹng m  2n) thÂŒ ta cŠn chĂčng minh 
8(m + n)3 
9n2(m + 2n) 
 6; 
hay 
4m3 + 4n3 + 12m2n + 12n2m  27n2m + 54n3: 
B§t ÂŻng thĂčc tr¶n mang tÂœnh thuŠn nh§t giĂșa cÂĄc bižn, do Ăą ta cho n = 1, lĂłc n y m  2 v  ta 
cŠn chĂčng minh 
4m3 + 12m2  15m  50  0: 
Tuy nhi¶n bÂŹng bižn ĂȘi tĂ·ĂŹng Ă·ĂŹng, ta ֖c (m2) 
 
m  
5 
2 
2 
 0. „y l  mët ¥nh gi¥ óng 
do m  2, do vÂȘy b§t ÂŻng thĂčc ban Šu ֖c chĂčng minh xong. 
B i to¥n kžt thóc.2 
2.8 Cho a; b; c l  cÂĄc sĂš thĂŒc dĂ·ĂŹng thay ĂȘi trong o€n [0; 1]. ChĂčng minh rÂŹng: 
p 
abc + 
p 
(1  a)(1  b)(1  c)  1 
LĂ­i giÂŁi. p döng b§t ÂŻng thĂčc Cauchy - Schwarz ta cĂą 
p 
abc + 
p 
(1  a)(1  b)(1  c)  
p 
(a + 1  a)[bc + (1  b)(1  c)] = 
p 
2bc  b  c + 1: 
NhĂ· vÂȘy, Âș kžt thĂłc chĂčng minh, ta cŠn chÂż ra rÂŹng 
2bc  b + c: 
Tuy nhi¶n „y l  mĂ«t ÂĄnh giÂĄ Ăłng vÂŒ theo giÂŁ thižt v  b§t ÂŻng thĂčc AM-GM thÂŒ 
p 
bc  b + c; 
2bc  2 
do Ăą b§t ÂŻng thĂčc ban Šu ֖c chĂčng minh xong. 
43
B i to¥n kžt thóc.2 
2.9 Cho a; b; c l  cÂĄc sĂš thĂŒc khĂŠng „m thay ĂȘi b§t kÂŒ. ChĂčng minh rÂŹng: 
p 
(ab + bc + ca)  
2 
p 
3: 3 p 
(a + b)(b + c)(c + a) 
LĂ­i giÂŁi. Âș ĂŸ rÂŹng ta cĂą ÂŻng thĂčc 
(a + b)(b + c)(c + a) = (a + b + c)(ab + bc + ca)  abc: 
M°t kh¥c, theo c¥c ¥nh gi¥ quen thuëc, ta cù 
a + b + c  
p 
3(ab + bc + ca); 
v  
abc  
r 
(ab + bc + ca)3 
27 
; 
do vÂȘy 
p 
3(ab + bc + ca)  
(a + b)(b + c)(c + a)  (ab + bc + ca) 
r 
(ab + bc + ca)3 
27 
= 
p 
(ab + bc + ca)3 
8 
p 
3 
3 
: 
TĂž „y, l§y c«n bÂȘc ba hai vž, ta thu ֖c b§t ÂŻng thĂčc cŠn chĂčng minh. 
B i to¥n kžt thóc.2 
2.10 Cho a; b; c l  cÂĄc sĂš thĂŒc ĂŠi mĂ«t ph„n bi»t. ChĂčng minh rÂŹng: 
a2 + b2 
a2  2ab + b2 + 
a2 + c2 
a2  2ac + c2 + 
b2 + c2 
b2  2bc + c2 
 
5 
2 
LĂ­i giÂŁi. DÂąy b§t ÂŻng thĂčc sau tĂ·ĂŹng Ă·ĂŹng vĂźi b§t ÂŻng thĂčc cŠn chĂčng minh 
(a + b)2 + (a  b)2 
(a  b)2 + 
(b + c)2 + (b  c)2 
(b  c)2 + 
(c + a)2 + (c  a)2 
(c  a)2 
 5; 
 
a + b 
a  b 
2 
+ 
 
b + c 
b  c 
2 
+ 
 
c + a 
c  a 
2 
 2: 
°t x = 
a + b 
a  b 
; y = 
b + c 
b  c 
; z = 
c + a 
c  a 
. Âș ĂŸ rÂŹng ta cĂą ÂŻng thĂčc 
xy + yz + zx = 
(a + b)(b + c) 
(a  b)(b  c) 
+ 
(b + c)(c + a) 
(b  c)(c  a) 
+ 
(c + a)(a + b) 
(c  a)(a  b) 
= 
(a + b)(b + c)(c  a) + (b + c)(c + a)(a  b) + (c + a)(a + b)(b  c) 
(a  b)(b  c)(c  a) 
= 1 
HĂŹn nĂșa, ta cĂŽng cĂą (x + y + z)2  0, do vÂȘy 
x2 + y2 + z2  2(xy + yz + zx) = 2: 
TĂž „y ta thu ֖c b§t ÂŻng thĂčc cŠn chĂčng minh. 
44
B i to¥n kžt thóc.2 
2.11 Cho a; b l  cÂĄc sĂš thĂŒc khĂŠng „m thoÂŁ mÂąn a + b  
4 
5 
. ChĂčng minh rÂŹng: 
r 
1  a 
1 + a 
+ 
r 
1  b 
1 + b 
 1  
r 
1  a  b 
1 + a + b 
LĂ­i giÂŁi. DÂąy b§t ÂŻng thĂčc sau l  tĂ·ĂŹng Ă·ĂŹng vĂźi b§t ÂŻng thĂčc cŠn chĂčng minh 
1  a 
1 + a 
+ 
1  b 
1 + b 
+ 2 
s 
(1  a)(1  b) 
(1 + a)(1 + b) 
 
1  a  b 
1 + a + b 
r 
+ 1 + 2 
1  a  b 
1 + a + b 
; 
2(1  ab) 
1 + ab + a + b 
+ 2 
r 
1 + ab  a  b 
1 + ab + a + b 
 
2 
1 + a + b 
+ 2 
r 
1  a  b 
1 + a + b 
: 
°t u = ab; v = a + b. Khi Ăą u; v  0 v  ta cŠn chĂčng minh 
2(1  u) 
1 + u + v 
+ 2 
r 
1 + u  v 
1 + u + v 
 
2 
1 + v 
+ 2 
r 
1  v 
1 + v 
: 
ThĂŒc hi»n bižn ĂȘi tĂ·ĂŹng Ă·ĂŹng, ta ֖c dÂąy b§t ÂŻng thĂčc sau 
1 + u  v 
1 + u + v 
 
1  v 
1 + v 
 
u(2 + v) 
(1 + v)(1 + v + u) 
 r 
1 + u  v 
1 + u + v 
+ 
r 
1  v 
1 + v 
! 
; 
2uv 
(1 + u + v)(1 + v) 
 
u(2 + v) 
(1 + v)(1 + v + u) 
 r 
1 + u  v 
1 + u + v 
+ 
r 
1  v 
1 + v 
! 
: 
Nžu u = 0 thÂŒ b§t ÂŻng thĂčc tr¶n hiÂșn nhi¶n Ăłng. Nžu u  0, b§t ÂŻng thĂčc tr¶n tĂ·ĂŹng Ă·ĂŹng 
vĂźi 
2v 
2 + v 
 
r 
1 + u  v 
1 + u + v 
+ 
r 
1  v 
1 + v 
: () 
Âș ĂŸ rÂŹng vĂźi u  0, ta cĂą ÂĄnh giÂĄ 
1 + u  v 
1 + u + v 
 
1  v 
1 + v 
; 
do vÂȘy r 
1 + u  v 
1 + u + v 
+ 
r 
1  v 
1 + v 
r 
 2 
1  v 
1 + v 
r 
1 + 
= 2 
2 
1 + v 
: 
HĂŹn nĂșa, ta l€i cĂą v  
4 
5 
theo gi£ thižt n¶n 
r 
1 + u  v 
1 + u + v 
+ 
r 
1  v 
1 + v 
s 
1 + 
 2 
2 
1 + 4 
5 
= 
2 
3 
: 
Ngo i ra cÎng do v  
4 
5 
 1 n¶n 
2v 
2 + v 
= 
2 
2 
v + 1 
 
2 
3 
; 
do vÂȘy ÂĄnh giÂĄ () Ăłng, cĂŽng cĂą nghŸa b§t ÂŻng thĂčc ban Šu ֖c chĂčng minh. 
45
B i to¥n ho n t§t.2 
2.12 Cho a; b; c l  cÂĄc sĂš thĂŒc dĂ·ĂŹng thay ĂȘi b§t kÂŒ. ChĂčng minh rÂŹng: 
a2 
b 
+ 
b2 
c 
+ 
c2 
a 
+ a + b + c  
6(a2 + b2 + c2) 
a + b + c 
LĂ­i giÂŁi. B§t ÂŻng thĂčc cŠn chĂčng minh mang tÂœnh hoÂĄn vĂ  giĂșa cÂĄc bižn, do Ăą khĂŠng m§t tÂœnh 
tĂȘng quÂĄt, ta giÂŁ sĂ» b l  sĂš h€ng nÂŹm giĂșa a v  c. Khi Ăą ta bižn ĂȘi b§t ÂŻng thĂčc nhĂ· sau 
X 
a2 
b 
 
+ b  2a 
 
6(a2 + b2 + c2) 
a + b + c 
 2(a + b + c); 
X(a  b)2 
b 
 
6(a2 + b2 + c2) 
a + b + c 
 2(a + b + c): 
p döng b§t ÂŻng thĂčc Cauchy - Schwarz, ta cĂą 
X(a  b)2 
b 
 
[(a  b) + (b  c) + (a  c)]2 
b + c + a 
= 
4(a  c)2 
a + b + c 
: 
Do Ăą ta chÂż cŠn chĂčng minh ֖c 
2(a  c)2  3(a2 + b2 + c2)  (a + b + c)2: 
Sau khi thu gĂ„n, ta ֖c b§t ÂŻng thĂčc hiÂșn nhi¶n Ăłng do b nÂŹm giĂșa a v  c 
2(b  c)(b  a)  0: 
B i to¥n ho n t§t.2 
2.13 Cho x; y; z l  cÂĄc sĂš thĂŒc thoÂŁ mÂąn x2 + y2 + z2 = 1. ChĂčng minh rÂŹng: 
1  x3 + y3 + z3  3xyz  1 
LĂ­i giÂŁi 1. ChĂł ĂŸ rÂŹng ta cĂą ÂŻng thĂčc 
(x3 + y3 + z3  3xyz)2 = (x + y + z)2(x2 + y2 + z2  xy  yz  zx)2 = (1 + 2t)(1  t)(1  t); 
trong Ăą t = xy + yz + zx. žn „y ta ÂĄp döng b§t ÂŻng thĂčc AM-GM Âș cĂą 
(x3 + y3 + z3  3xyz)2  
[(1 + 2t) + (1  t) + (1  t)]3 
27 
= 1; 
do vÂȘy 1  x3 + y3 + z3  3xyz  1. 
PhÂČp chĂčng minh ho n t§t.2 
LĂ­i giÂŁi 2. p döng b§t ÂŻng thĂčc Cauchy - Schwarz, ta cĂą 
(x3 + y3 + z3  3xyz)2 = [x(x2  yz) + y(y2  zx) + z(z2  xy)]2 
 (x2 + y2 + z2)[(x2  yz)2 + (y2  zx)2 + (z2  xy)2]: 
46
HĂŹn nĂșa, ta l€i cĂą 
(x2  yz)2 + (y2  zx)2 + (z2  xy)2 = (x2 + y2 + z2)2  (xy + yz + zx)2  (x2 + y2 + z2)2; 
do vÂȘy 
(x3 + y3 + z3  3xyz)2  (x2 + y2 + z2)3 = 1: 
TĂž Ăą ta suy ra 1  x3 + y3 + z3  3xyz  1. 
PhÂČp chĂčng minh ho n t§t.2 
2.14 Cho x; y; z l  cÂĄc sĂš thĂŒc dĂ·ĂŹng thay ĂȘi b§t kÂŒ. ChĂčng minh rÂŹng: 
xyz 
(1 + 3x)(z + 6)(x + 8y)(y + 9z) 
 
1 
74 
LĂ­i giÂŁi. p döng b§t ÂŻng thĂčc AM-GM, ta cĂą cÂĄc ÂĄnh giÂĄ sau: 
z + 6 = z + 1 + 1 + 1 + 1 + 1 + 1  7 7 p 
z; 
1 + 3x = 1 + 
x 
2 
+ 
x 
2 
+ 
x 
2 
+ 
x 
2 
+ 
x 
2 
+ 
x 
2 
r 
x6 
 7 7 
26 ; 
x + 8y = x + 
4y 
3 
+ 
4y 
3 
+ 
4y 
3 
+ 
4y 
3 
+ 
4y 
3 
+ 
4y 
3 
r 
xy6: 
 7 7 
46 
36 ; 
y + 9z = y + 
3z 
2 
+ 
3z 
2 
+ 
3z 
2 
+ 
3z 
2 
+ 
3z 
2 
+ 
3z 
2 
r 
yz6: 
 7 7 
36 
26 : 
Nh„n cÂĄc b§t ÂŻng thĂčc tr¶n vĂźi nhau, ta ֖c 
r 
z: 
(z + 6)(1 + 3x)(x + 8y)(y + 9z)  74 7 
x6 
26 :xy6: 
46 
36 :yz6: 
36 
26 = 74xyz; 
tĂž Ăą suy ra 
xyz 
(1 + 3x)(z + 6)(x + 8y)(y + 9z) 
 
1 
74 : 
PhÂČp chĂčng minh ho n t§t.2 
2.15 Cho a; b; c l  cÂĄc sĂš thĂŒc dĂ·ĂŹng thay ĂȘi b§t kÂŒ. ChĂčng minh rÂŹng: 
a + b 
ab + c2 + 
b + c 
bc + a2 + 
a + c 
ac + b2 
 
1 
a 
+ 
1 
b 
+ 
1 
c 
LĂ­i giÂŁi. p döng b§t ÂŻng thĂčc Cauchy - Schwarz, ta cĂą 
a2 
b(a2 + c2) 
+ 
b2 
a(b2 + c2) 
 
(a + b)2 
b(a2 + c2) + a(b2 + c2) 
= 
(a + b)2 
(a + b)(ab + c2) 
; 
tÞ „y ta suy ra 
a + b 
ab + c2 
 
a2 
b(a2 + c2) 
+ 
b2 
a(b2 + c2) 
. Thižt lÂȘp hai b§t ÂŻng thĂčc tĂ·ĂŹng tĂŒ rçi cĂ«ng 
l€i, ta ÷ñc 
a + b 
ab + c2 + 
b + c 
bc + a2 + 
a + c 
ac + b2 
 
a2 
b(a2 + c2) 
+ 
b2 
a(b2 + c2) 
+ 
b2 
c(b2 + a2) 
+ 
c2 
b(a2 + c2) 
+ 
a2 
c(a2 + b2) 
+ 
c2 
a(b2 + c2) 
= 
1 
a 
+ 
1 
b 
+ 
1 
c 
: 
47
PhÂČp chĂčng minh ho n t§t.2 
2.16 Cho a; b; c l  cÂĄc sĂš thĂŒc khĂŠng „m thoÂŁ mÂąn khĂŠng cĂą b§t kÂŒ hai sĂš n o çng thĂ­i 
bÂŹng 0. ChĂčng minh rÂŹng: 
a(b + c) 
b2 + bc + c2 + 
b(a + c) 
a2 + ac + c2 + 
c(a + b) 
a2 + ab + b2 
 2 
LĂ­i giÂŁi. p döng b§t ÂŻng thĂčc Cauchy - Schwarz, ta cĂą 
a(b + c) 
b2 + bc + c2 + 
b(a + c) 
a2 + ac + c2 + 
c(a + b) 
a2 + ab + b2 = 
a2 
a(b + c)  
abc 
b + c 
+ 
b2 
b(a + c)  abc 
a+c 
+ 
c2 
c(a + b)  abc 
a+b 
 
(a + b + c)2 
2(ab + bc + ca)  
abc 
b + c 
 
abc 
c + a 
 
abc 
a + b 
: 
NhĂ· vÂȘy, Âș kžt thĂłc chĂčng minh, ta cŠn chÂż ra rÂŹng 
(a + b + c)2  4(ab + bc + ca)  2abc:( 
1 
a + b 
+ 
1 
b + c 
+ 
1 
a + c 
); 
hay 
a2 + b2 + c2 + 2abc:( 
1 
a + b 
+ 
1 
b + c 
+ 
1 
a + c 
)  2(ab + bc + ca): 
p döng ¥nh gi¥ cÏ b£n 
1 
x 
+ 
1 
y 
+ 
1 
z 
 
9 
x + y + z 
, ta cĂą 
a2 + b2 + c2 + 2abc:( 
1 
a + b 
+ 
1 
b + c 
+ 
1 
a + c 
)  a2 + b2 + c2 + 
9abc 
a + b + c 
: 
CĂŠng vi»c cuĂši cĂČng chÂż cŠn chĂčng minh 
a2 + b2 + c2 + 
9abc 
a + b + c 
 2(ab + bc + ca); 
hay a3 + b3 + c3 + 3abc  ab(a + b) + bc(b + c) + ca(c + a). Tuy nhi¶n ¥nh gi¥ n y óng theo b§t 
ÂŻng thĂčc Schur bÂȘc ba n¶n b§t ÂŻng thĂčc ban Šu ֖c chĂčng minh xong. 
B i to¥n kžt thóc.2 
2.17 Cho a; b; c l  cÂĄc sĂš thĂŒc dĂ·ĂŹng thoÂŁ mÂąn a2 + b2 + c2 = 1. ChĂčng minh rÂŹng: 
a 
b2 + c2 + 
b 
a2 + c2 + 
c 
a2 + b2 
 
p 
3 
2 
3 
Líi gi£i. Sû döng gi£ thižt, ta cù 
a 
b2 + c2 + 
b 
a2 + c2 + 
c 
a2 + b2 = 
a 
1  a2 + 
b 
1  b2 + 
c 
1  c2 : 
Âș ĂŸ rÂŹng ta cĂą ÂĄnh giÂĄ 
a 
1  a2 
 
p 
3 
2 
3 
a2 = 
p 
3 + 2)(a 
a(a 
p 
3  1)2 
2(1  a2) 
 0; 
48
do vÂȘy 
a 
1  a2 
 
p 
3 
2 
3 
a2. Thižt lÂȘp hai ÂĄnh giÂĄ tĂ·ĂŹng tĂŒ v  cĂ«ng l€i, ta ֖c 
a 
1  a2 + 
b 
1  b2 + 
c 
1  c2 
 
p 
3 
2 
3 
(a2 + b2 + c2); 
do vÂȘy 
a 
b2 + c2 + 
b 
a2 + c2 + 
c 
a2 + b2 
 
p 
3 
2 
3 
: 
PhÂČp chĂčng minh ho n t§t.2 
2.18 Cho x; y; z l  cÂĄc sĂš thĂŒc dĂ·ĂŹng thay ĂȘi b§t kÂŒ. ChĂčng minh rÂŹng: 
p 
x + y + z 
 p 
x 
y + z 
+ 
p 
y 
x + z 
+ 
p 
z 
x + y 
 
 
p 
3 
2 
3 
LĂ­i giÂŁi. B§t ÂŻng thĂčc cŠn chĂčng minh mang tÂœnh thuŠn nh§t, do Ăą ta chu©n hĂąa x+y+z = 1. 
p 
p 
p 
çng thíi, ta °t a = 
x; b = 
y; c = 
z. NhĂ· vÂȘy ta cŠn chĂčng minh 
a 
1  a2 + 
b 
1  b2 + 
c 
1  c2 
 
p 
3 
2 
3 
: 
Tuy nhi¶n „y l  mĂ«t kžt quÂŁ Âą ֖c chĂčng minh Ă° b i 2.17 . 2 
2.19 Cho a; b; c l  cÂĄc sĂš thĂŒc khĂŠng „m thoÂŁ mÂąn khĂŠng cĂą b§t kÂŒ hai sĂš n o çng thĂ­i 
bÂŹng 0. ChĂčng minh rÂŹng: 
a3 
(2a2 + b2)(2a2 + c2) 
+ 
b3 
(2b2 + a2)(2b2 + c2) 
+ 
c3 
(2c2 + a2)(2c2 + b2) 
 
1 
a + b + c 
LĂ­i giÂŁi. p döng b§t ÂŻng thĂčc Cauchy-Schwartz, ta cĂą 
(2a2 + b2)(2a2 + c2) = (a2 + a2 + b2)(a2 + c2 + a2)  (a2 + ab + ac)2 = a2(a + b + c)2: 
NhĂ· vÂȘy 
a3 
(2a2 + b2)(2a2 + c2) 
 
a 
(a + b + c)2 . Thižt lÂȘp hai ÂĄnh giÂĄ tĂ·ĂŹng tĂŒ rçi cĂ«ng l€i, ta ֖c 
a3 
(2a2 + b2)(2a2 + c2) 
+ 
b3 
(2b2 + a2)(2b2 + c2) 
+ 
c3 
(2c2 + a2)(2c2 + b2) 
 
a + b + c 
(a + b + c)2 = 
1 
a + b + c 
: 
PhÂČp chĂčng minh ho n t§t.2 
2.20 Cho a; b; c l  cÂĄc sĂš thĂŒc khĂŠng „m thay ĂȘi b§t kÂŒ. ChĂčng minh rÂŹng: 
a2 + b2 
a + b 
+ 
b2 + c2 
b + c 
+ 
c2 + a2 
c + a 
 
3(a2 + b2 + c2) 
a + b + c 
LĂ­i giÂŁi. DÂąy b§t ÂŻng thĂčc sau tĂ·ĂŹng Ă·ĂŹng vĂźi b§t ÂŻng thĂčc cŠn chĂčng minh 
2(a2 + b2 + c2) + 
c(a2 + b2) 
a + b 
+ 
a(b2 + c2) 
b + c 
+ 
b(c2 + a2) 
c + a 
 3(a2 + b2 + c2); 
c[(a + b)2  2ab] 
a + b 
+ 
a[(b + c)2  2bc] 
b + c 
+ 
b[(c + a)2  2ca]) 
c + a 
 a2 + b2 + c2; 
2ab + 2bc + 2ca  a2 + b2 + c2 + 2abc 
 
1 
a + b 
+ 
1 
b + c 
+ 
1 
a + c 
 
: 
49
ÂĄnh giÂĄ cuĂši cĂČng l  mĂ«t kžt quÂŁ Âą ֖c chĂčng minh Ă° b i 2.16 , do vÂȘy ta kžt thĂłc chĂčng 
minh.2 
2.21 Cho x; y; z l  cÂĄc sĂš thĂŒc dĂ·ĂŹng thoÂŁ mÂąn x + y + z = 1. ChĂčng minh rÂŹng: 
xy 
1 + z 
+ 
yz 
1 + x 
+ 
xz 
1 + y 
 
1 
4 
LĂ­i giÂŁi. ChĂł ĂŸ rÂŹng 
xy 
1 + z 
= 
xy 
(x + z) + (y + z) 
; 
v  theo mĂ«t ÂĄnh giÂĄ quen thuĂ«c thÂŒ 
4 
(x + z) + (y + z) 
 
1 
x + z 
+ 
1 
y + z 
; 
do vÂȘy 
xy 
1 + z 
 
1 
4 
 
xy 
x + z 
+ 
xy 
y + z 
 
. Thižt lÂȘp hai ÂĄnh giÂĄ tĂ·ĂŹng tĂŒ rçi cĂ«ng l€i, ta ֖c 
xy 
1 + z 
+ 
yz 
1 + x 
+ 
xz 
1 + y 
 
1 
4 
 
xy + yz 
x + z 
+ 
yz + zx 
x + y 
+ 
zx + xy 
y + z 
 
= 
x + y + z 
4 
; 
tĂž „y ta thu ֖c b§t ÂŻng thĂčc cŠn chĂčng minh. 
B i to¥n kžt thóc.2 
2.22 Cho a; b; c l  cÂĄc sĂš thĂŒc dĂ·ĂŹng thay ĂȘi b§t kÂŒ. ChĂčng minh rÂŹng: 
( 
a 
b 
+ 
b 
c 
+ 
c 
a 
p 
3(a2 + b2 + c2) 
)(a + b + c)  3 
LĂ­i giÂŁi. p döng b§t ÂŻng thĂčc Cauchy-Schwartz, ta cĂą 
a 
b 
+ 
b 
c 
+ 
c 
a 
 
(a + b + c)2 
ab + bc + ca 
; 
do vÂȘy ( 
a 
b 
+ 
b 
c 
+ 
c 
a 
)(a+b+c)  
(a + b + c)3 
ab + bc + ca 
. NhĂ· vÂȘy, Âș kžt thĂłc chĂčng minh, ta cŠn chÂż ra rÂŹng 
(a + b + c)3  3(ab + bc + ca) 
p 
3(a2 + b2 + c2); 
hay (a+b+c)6  27(a2 +b2 +c2)(ab+bc+ca)2. Tuy nhi¶n „y l  mĂ«t ÂĄnh giÂĄ Ăłng vÂŒ theo b§t 
ÂŻng thĂčc AM-GM, ta cĂą 
(a + b + c)6 = [(a2 + b2 + c2) + (ab + bc + ca) + (ab + bc + ca)]3  27(a2 + b2 + c2)(ab + bc + ca)2; 
do vÂȘy b§t ÂŻng thĂčc ban Šu ֖c chĂčng minh xong. 
B i to¥n kžt thóc.2 
2.23 Cho a; b; c l  cÂĄc sĂš thĂŒc dĂ·ĂŹng thay ĂȘi b§t kÂŒ. ChĂčng minh rÂŹng: 
a2 
b 
+ 
b2 
c 
+ 
c2 
a 
p 
3(a2 + b2 + c2) 
+ a + b + c  2 
LĂ­i giÂŁi. p döng b§t ÂŻng thĂčc AM-GM ta ֖c 
a2 
b 
+ 
b2 
c 
+ 
c2 
a 
r 
( 
+ a + b + c  2 
a2 
b 
+ 
b2 
c 
+ 
c2 
a 
)(a + b + c): 
50
NhĂ· vÂȘy, Âș kžt thĂłc chĂčng minh, ta cŠn chÂż ra rÂŹng 
( 
a2 
b 
+ 
b2 
c 
+ 
c2 
a 
)(a + b + c)  3(a2 + b2 + c2): 
ThÂȘt vÂȘy, ÂĄp döng b§t ÂŻng thĂčc Cauchy-Schwartz, ta cĂą 
a2 
b 
+ 
b2 
c 
+ 
c2 
a 
 
(a2 + b2 + c2)2 
a2b + b2c + c2a 
: 
CĂŠng vi»c cuĂši cĂČng chÂż cŠn chĂčng minh 
(a + b + c)(a2 + b2 + c2)  3(a2b + b2c + c2a); 
hay (a3 + ab2) + (b3 + bc2) + (c3 + ca2)  2(a2b + b2c + c2a). Tuy nhi¶n „y l  mët ¥nh gi¥ óng 
theo b§t ÂŻng thĂčc AM-GM, do vÂȘy b§t ÂŻng thĂčc ban Šu ֖c chĂčng minh. 
B i to¥n kžt thóc.2 
2.24 Cho a; b; c l  cÂĄc sĂš thĂŒc dĂ·ĂŹng thuĂ«c khoÂŁng (0; 1) thoÂŁ mÂąn ab + bc + ca = 1. ChĂčng 
minh rÂŹng: 
a2 + b2 
(1  a2)(1  b2) 
+ 
b2 + c2 
(1  b2)(1  c2) 
+ 
c2 + a2 
(1  a2)(1  c2) 
 
9 
2 
LĂ­i giÂŁi. B§t ÂŻng thĂčc ban Šu tĂ·ĂŹng Ă·ĂŹng vĂźi mĂ©i b§t ÂŻng thĂčc trong dÂąy sau 
X 
a2 + b2 
(1  a2)(1  b2) 
+ 
1 
2 
 
 6; 
X(1 + a2)(1 + b2) 
(1  a2)(1  b2) 
 12: 
Âș ĂŸ rÂŹng ta cĂą 
(1  a2)(1  b2)  (1  ab)2 = (a  b)2; 
do vÂȘy (1  a2)(1  b2)  (1  ab)2. M°t khÂĄc, theo b§t ÂŻng thĂčc Cauchy - Schwarz, ta cĂą 
(1 + a2)(1 + b2)  (1 + ab)2; 
do vÂȘy ta suy ra 
(1 + a2)(1 + b2) 
(1  a2)(1  b2) 
 
(1 + ab)2 
(1  ab)2 : 
žn „y ta thižt lÂȘp hai ÂĄnh giÂĄ tĂ·ĂŹng tĂŒ v  cĂ«ng l€i Âș cĂą 
X(1 + a2)(1 + b2) 
(1  a2)(1  b2) 
 
X(1 + ab)2 
(1  ab)2 : 
Ta ÂĄp döng tižp b§t ÂŻng thĂčc AM-GM Âș suy ra 
X(1 + a2)(1 + b2) 
(1  a2)(1  b2) 
s 
 3 3 
(1 + ab)(1 + bc)(1 + ca) 
(1  ab)(1  bc)(1  ca) 
2 
: 
Do vÂȘy, Âș kžt thĂłc chĂčng minh, ta cŠn chÂż ra rÂŹng 
(1 + ab)(1 + bc)(1 + ca)  8(1  ab)(1  bc)(1  ca): 
51
°t x = ab; y = bc; z = ca. Khi Ăą x; y; z  0; x + y + z = 1 v  ta cŠn chĂčng minh 
(1 + x)(1 + y)(1 + z)  8(1  x)(1  y)(1  z); 
tĂ·ĂŹng Ă·ĂŹng 
9xyz  7(xy + yz + zx)  2: 
Theo mĂ«t kžt quÂŁ Âą ֖c chĂčng minh Ă° b i 2.35 , ta cĂą 
x2 + y2 + z2 + 
9xyz 
x + y + z 
 2(xy + yz + zx); 
tĂž Ăą sĂ» döng giÂŁ thižt x + y + z = 1 Âș suy ra 9xyz  4(xy + yz + zx)  1. CĂŠng vi»c cuĂši cĂČng 
l  chĂčng minh 
4(xy + yz + zx)  1  7(xy + yz + zx)  2; 
hay xy + yz + zx  
1 
3 
. Tuy nhi¶n „y l  mĂ«t ÂĄnh giÂĄ Ăłng vÂŒ 
xy + yz + zx  
(x + y + z)2 
3 
= 
1 
3 
; 
do vÂȘy b§t ÂŻng thĂčc ban Šu ֖c chĂčng minh xong. 
B i to¥n kžt thóc.2 
2.25 Cho a; b; c l  cÂĄc sĂš thĂŒc dĂ·ĂŹng thay ĂȘi b§t kÂŒ. ChĂčng minh rÂŹng: 
1 + a3 + b3 + 3 p 
1 + b3 + c3 + 3 p 
1 + a3 + c3  3 p 
27 + 2(a + b + c)3 
3 p 
LĂ­i giÂŁi. p döng b§t ÂŻng thĂčc Holder, ta cĂą 
(1 + a3 + b3)[27 + (a + b + c)3 + (a + b + c)3]2  [9 + a(a + b + c)2 + b(a + b + c)2]3; 
tĂž Ăą ta suy ra 
1 + a3 + b3: 3 p 
[27 + 2(a + b + c)3]2  9 + (a + b)(a + b + c)2: 
3 p 
Thižt lÂȘp hai b§t ÂŻng thĂčc tĂ·ĂŹng tĂŒ v  cĂ«ng l€i, ta ֖c 
3 p 
[27 + 2(a + b + c)3]2( 3 p 
1 + a3 + b3 + 3 p 
1 + b3 + c3 + 3 p 
1 + a3 + c3)  27 + 2(a + b + c)3; 
tĂž Ăą ta thu ֖c b§t ÂŻng thĂčc cŠn chĂčng minh. 
B i to¥n kžt thóc.2 
NhÂȘn xÂČt. B§t ÂŻng thĂčc tr¶n l  h» quÂŁ trĂŒc tižp cĂ”a b§t ÂŻng thĂčc Minkowsky mĂ° rĂ«ng: 
3 p 
a3 + b3 + c3 + 3 p 
d3 + e3 + f3 + 3 p 
g3 + h3 + k3  3 p 
(a + d + g)3 + (b + e + h)3 + (c + f + k)3: 
CÂĄch chĂčng minh tĂ·ĂŹng tĂŒ nhĂ· lĂ­i giÂŁi cĂ”a b i toÂĄn tr¶n. 
2.26 Cho a; b; c l  cÂĄc sĂš thĂŒc khĂŠng „m thay ĂȘi b§t kÂŒ. ChĂčng minh rÂŹng: 
bc 
(a + b)(a + c) 
+ 
ca 
(b + c)(b + a) 
+ 
ab 
(c + a)(c + b) 
 
2(a2 + b2 + c2) + ab + bc + ca 
2(a2 + b2 + c2) + 2(ab + bc + ca) 
52
LĂ­i giÂŁi. B§t ÂŻng thĂčc cŠn chĂčng minh tĂ·ĂŹng Ă·ĂŹng vĂźi 
1  
bc 
(a + b)(a + c) 
 
ca 
(b + c)(b + a) 
 
ab 
(c + a)(c + b) 
 1  
2(a2 + b2 + c2) + ab + bc + ca 
2(a2 + b2 + c2) + 2(ab + bc + ca) 
: 
M°t khÂĄc, Âș ĂŸ rÂŹng ta cĂą cÂĄc ÂŻng thĂčc sau: 
1  
bc 
(a + b)(a + c) 
 
ca 
(b + c)(b + a) 
 
ab 
(c + a)(c + b) 
= 
2abc 
(a + b)(b + c)(c + a) 
; 
1  
2(a2 + b2 + c2) + ab + bc + ca 
2(a2 + b2 + c2) + 2(ab + bc + ca) 
= 
ab + bc + ca 
(a + b)2 + (b + c)2 + (c + a)2 ; 
do Ăą ta cŠn chĂčng minh 
2abc 
(a + b)(b + c)(c + a) 
 
ab + bc + ca 
(a + b)2 + (b + c)2 + (c + a)2 ; 
hay 
2(a + b) 
(c + a)(c + b) 
+ 
2(b + c) 
(a + b)(a + c) 
+ 
2(c + a) 
(b + c)(b + a) 
 
1 
a 
+ 
1 
b 
+ 
1 
c 
: 
Âș ĂŸ rÂŹng 
1 
c 
 
2(a + b) 
(c + a)(c + b) 
= 
(c  a)(c  b) 
c(c + a)(c + b) 
; 
do Ăą b§t ÂŻng thĂčc cŠn chĂčng minh tĂ·ĂŹng Ă·ĂŹng vĂźi 
(a  b)(a  c) 
a(a + b)(a + c) 
+ 
(b  a)(b  c) 
b(b + a)(b + c) 
+ 
(c  a)(c  b) 
c(c + a)(c + b) 
 0: 
Tuy nhi¶n ÂĄnh giÂĄ n y Ăłng theo b§t ÂŻng thĂčc Vornicu - Schur, do vÂȘy b§t ÂŻng thĂčc ban Šu 
֖c chĂčng minh xong. 
B i to¥n kžt thóc.2 
2.27 Cho a; b; c l  cÂĄc sĂš thĂŒc dĂ·ĂŹng thoÂŁ mÂąn abc = 1. ChĂčng minh rÂŹng: 
4 p 
2a2 + bc + 4 p 
2b2 + ac + 4 p 
2c2 + ab  
ab + bc + ca 
4 p 
3 
: 
q 
p 
a + 
p 
b + 
p 
c 
Líi gi£i. °t x = 
1 
a 
; y = 
1 
b 
; z = 
1 
c 
. Khi ù x; y; z  0 v  xyz = 1. çng thíi ta cÎng cù 
4 p 
r 
2a2 + bc = 4 
2 
x2 + 
1 
yz 
r 
= 4 
2yz + x2 
x 
; 
v  ab + bc + ca = x + y + z. Theo Ăą, b§t ÂŻng thĂčc cŠn chĂčng minh trĂ° th nh 
r 
X 4 
2yz + x2 
x 
 
(x + y + z) 
4 p 
3 
s 
1 
p 
x 
+ 
1 
p 
y 
+ 
1 
p 
z 
; 
hay   
r 
X 4 
2yz + x2 
x 
!4 
 
(x + y + z)4 
3 
 
1 
p 
x 
+ 
1 
p 
y 
+ 
1 
p 
z 
2 
: 
53
p döng b§t ÂŻng thĂčc Holder, ta cĂą 
3(2yz + x2 + 2zx + y2 + 2xy + z2) 
 
1 
p 
x 
+ 
1 
p 
y 
+ 
1 
p 
z 
2 
 
  
X 4 
r 
2yz + x2 
x 
!4 
: 
NhĂ· vÂȘy, Âș kžt thĂłc chĂčng minh, ta cŠn chÂż ra rÂŹng 
3(2yz + x2 + 2zx + y2 + 2xy + z2)  
(x + y + z)4 
3 
; 
hay x + y + z  3. Tuy nhi¶n „y l  mĂ«t ÂĄnh giÂĄ Ăłng vÂŒ theo b§t ÂŻng thĂčc AM-GM 
x + y + z  3 3 p 
xyz = 3; 
do vÂȘy b§t ÂŻng thĂčc ban Šu ֖c chĂčng minh xong. 
B i to¥n kžt thóc.2 
2.28 Cho a; b; c l  cÂĄc sĂš thĂŒc khĂŠng „m ĂŠi mĂ«t ph„n bi»t. ChĂčng minh rÂŹng: 
(ab + bc + ca) 
 
1 
(a  b)2 + 
1 
(b  c)2 + 
1 
(c  a)2 
 
 4 
LĂ­i giÂŁi. B§t ÂŻng thĂčc ban Šu mang tÂœnh Ăši xĂčng giĂșa cÂĄc bižn, do Ăą khĂŠng m§t tÂœnh tĂȘng 
qu¥t, ta gi£ sû a  b  c  0. Khi ù ta °t a  b = x; b  c = y. TÞ „y ta suy ra x; y  0 v  
ab + bc + ca  ab = (c + y)(c + x + y)  y(x + y): 
çng thĂ­i, cĂŽng tĂž phÂČp °t tr¶n, ta cĂą 
1 
(a  b)2 + 
1 
(b  c)2 + 
1 
(c  a)2 = 
1 
x2 + 
1 
y2 + 
1 
(x + y)2 : 
NhĂ· vÂȘy, ta Ă·a b i toÂĄn v· vi»c chĂčng minh 
y(x + y) 
 
1 
x2 + 
1 
y2 + 
1 
(x + y)2 
 
 4; 
hay 
y(x + y) 
x2 + 
x 
y 
+ 
y 
x + y 
 3: 
°t t = 
x 
y 
. Khi Ăą t  0 v  ta cŠn chĂčng minh 
t + 1 
t2 + t + 
1 
t + 1 
 3: 
Sau khi bižn ĂȘi tĂ·ĂŹng Ă·ĂŹng, ta thu ֖c mĂ«t ÂĄnh giÂĄ hiÂșn nhi¶n Ăłng 
(t2  t  1)2  0; 
do vÂȘy b§t ÂŻng thĂčc ban Šu ֖c chĂčng minh xong. 
B i to¥n kžt thóc.2 
54
2.29 Cho a; b; c l  cÂĄc sĂš thĂŒc dĂ·ĂŹng thĂ€a mÂąn abc = 1. ChĂčng minh rÂŹng: 
a 
b 
+ 
b 
c 
+ 
c 
a 
+ 3  ab + bc + ca + a + b + c 
LĂ­i giÂŁi. Do abc = 1 n¶n tçn t€i cÂĄc sĂš thĂŒc dĂ·ĂŹng x; y; z sao cho 
a = 
x 
y 
; b = 
y 
z 
; c = 
z 
x 
: 
Khi Ăą b§t ÂŻng thĂčc cŠn chĂčng minh trĂ° th nh 
xz 
y2 + 
xy 
z2 + 
yz 
x2 + 3  
x 
z 
+ 
y 
x 
+ 
z 
y 
+ 
x 
y 
+ 
y 
z 
+ 
z 
x 
; 
tĂ·ĂŹng Ă·ĂŹng 
x3y3 + y3z3 + z3x3 + 3x2y2z2  xyz[xy(x + y) + yz(y + z) + zx(z + x)]: 
Tuy nhi¶n „y l  mĂ«t h» quÂŁ trĂŒc tižp cĂ”a b§t ÂŻng thĂčc Schur bÂȘc 3: 
m3 + n3 + p3 + 3mnp  mn(m + n) + np(n + p) + pm(p + m); 
Ă° „y m = xy,n = yz v  p = zx. Do vÂȘy b§t ÂŻng thĂčc ban Šu ֖c chĂčng minh xong. 
B i to¥n kžt thóc.2 
2.30 Cho a; b; c  0. ChĂčng minh rÂŹng: 
P a + b 
ab + c2 
 
P 1 
a 
LĂ­i giÂŁi. 
B§t ÂŻng thĂčc Âą cho tĂ·ĂŹng Ă·ĂŹng vĂźi: 
P 1 
a 
 
P a + b 
ab + c2 = 
(a  c)(b  c) 
abc + c3 + 
(b  a)(c  a) 
abc + a3 + 
(a  b)(c  b) 
abc + b3 
 0 
°t 
1 
abc + a3 = x; 
1 
abc + b3 = y; 
1 
abc + c3 = z: 
B§t ÂŻng thĂčc ֖c Ă·a v· d€ng Vornicu Schur: 
x(a  c)(b  c) + y(b  a)(c  a) + z(a  b)(c  b)  0() 
GiÂŁ sĂ» a  b  c, thž thÂŒ abc + c3  abc + b3. 
1 
Suy ra 
abc + c3 
 
1 
abc + b3 
hay z  y 
M°c khÂĄc, theo i·u giÂŁ sĂ» thÂŒ b  c, do Ăą a  b  a  c. 
Kžt hñp vßi z  y  0, suy ra z(a  c)  y(a  b). 
Vižt l€i b§t ÂŻng thĂčc () nhĂ· sau: 
x(a  b)(b  c) + (b  c)[z(a  c)  y(a  b)]  0 
B§t ÂŻng thĂčc n y Ăłng theo cÂĄc i·u giÂŁ sĂ». 
PhÂČp chĂčng minh ho n t§t. D§u bÂŹng xÂŁy ra khi a = b = c.2 
2.31 Cho x; y; z l  cÂĄc sĂš thĂŒc thĂ€a mÂąn x2 + y2 + z2 = 1. TÂŒm max cĂ”a biÂșu thĂčc: 
P = x3 + y3 + z3  3xyz 
LĂ­i giÂŁi. Ta cĂą: P = x3 + y3 + z3  3xyz = (x + y + z)(x2 + y2 + z2  xy  yz  zx). 
Suy ra P2 = (x + y + z)2(x2 + y2 + z2  xy  yz  zx)2 
p döng b§t ÂŻng thĂčc AM-GM cho 3 sĂš khĂŠng „m: 
55
P2 = (x + y + z)2(x2 + y2 + z2  xy  yz  zx)2 
= (x + y + z)2(x2 + y2 + z2  xy  yz  zx)(x2 + y2 + z2  xy  yz  zx) 
 
 
(x + y + z)2 + (x2 + y2 + z2  xy  yz  zx) + (x2 + y2 + z2  xy  yz  zx) 
3 
3 
= (x2 + y2 + z2)3 = 1 (theo gi£ thižt) . 
Suy ra P  1. 
VÂȘy maxP = 1 , (x+y +z)2 = x2 +y2 +z2 xy yz zx , x = 1; y = z = 0 v  cÂĄc hoÂĄn vĂ .2 
2.32 Cho a; b; c  0. ChĂčng minh rÂŹng: 
a2 
b + c 
+ 
b2 
a + c 
+ 
c2 
a + b 
 
a2 
a + b 
+ 
b2 
b + c 
+ 
c2 
c + a 
LĂ­i giÂŁi. 
CÂĄch 1 
Ta cĂą: 
P a2  b2 
a + b 
= a  b + b  c + c  a = 0 
Suy ra 
P 2a2 
a + b 
= 
P a2 + b2 
a + b 
Khi Ăą ta cŠn chĂčng minh: 
P 2c2 
a + b 
 
P a2 + b2 
a + b 
B§t ÂŻng thĂčc n y tĂ·ĂŹng Ă·ĂŹng vĂźi: 
P 2c2  a2  b2 
a + b 
 0 
hay 
P 
 
c2 
a + b 
 
a2 
a + b 
+ 
a2 
b + c 
 
c2 
b + c 
 
 0 
hay 
P (c  a)2(c + a) 
(a + b)(b + c) 
 0 (Ăłng) 
PhÂČp chĂčng minh ho n t§t. ÂŻng thĂčc xÂŁy ra khi v  chÂż khi a = b = c 
CÂĄch 2 
B§t ÂŻng thĂčc Âą cho tĂ·ĂŹng Ă·ĂŹng vĂźi: 
P 
a2 
 
1 
b + c 
 
1 
a + c 
 
 0 
hay 
(a2(a2  c2) + b2(b2  a2) + c2(c2  b2)  0 
hay 
1 
2 
[(a2  b2)2 + (b2  c2)2 + (c2  a2)2]  0 (Ăłng) . 
PhÂČp chĂčng minh ho n t§t. ÂŻng thĂčc xÂŁy ra khi v  chÂż khi a = b = c.2 
2.34 Cho x; y; z  0. ChĂčng minh rÂŹng: 
a2 + bc 
(b + c)2 + 
b2 + ac 
(a + c)2 + 
c2 + ab 
(a + b 
2 
 
3 
2 
LĂ­i giÂŁi. 
B§t ÂŻng thĂčc Âą cho tĂ·ĂŹng Ă·ĂŹng vĂźi: 
P 
 
a2 + bc 
(b + c)2 
 
1 
2 
 
 0 
hay 
56
P 2a2  b2  c2 
(b + c)2 
 0 
GiÂŁ sĂ» a  b  c. 
Khi Ăą ta cĂą hai dÂąy cĂČng chi·u: 8 
: 
2a2  b2  c2  2b2  a2  c2  2c2  a2  b2 
1 
(b + c)2 
 
1 
(a + c)2 
 
1 
(a + b)2 
p döng b§t ÂŻng thĂčc Chebychep cho hai dÂąy tr¶n: 
P 
 
(2a2  b2  c2): 
1 
(b + c)2 
 
 [ 
P 
(2a2  b2  c2)] : 
 
P 1 
(b + c)2 
 
= 0: 
 
P 1 
(b + c)2 
 
= 0 
PhÂČp chĂčng minh ho n t§t. ÂŻng thĂčc xÂŁy ra khi v  chÂż khi a = b = c.2 
2.35 Cho a; b  0. ChĂčng minh rÂŹng: 
(a2 + b2)(a + b)2 + (ab + 1)2  2(a + b)2 
LĂ­i giÂŁi. 
Âș ĂŸ rÂŹng a2 + b2 = (a + b)2  2ab. 
°t a2 + b2 = x; ab = y 
B§t ÂŻng thĂčc cŠn chĂčng minh tĂ·ĂŹng Ă·ĂŹng vĂźi: 
x(x + 2y) + (y + 1)2  2(x + 2y) 
Khai triÂșn v  rĂłt gĂ„n, ta ֖c: 
x2 + y2 + 1  2x  2y + 2xy  0 
hay 
(x + y  1)2  0 (Ăłng) 
PhÂČp chr 
Ăčng minh ho n t§t. ÂŻng thĂčc xÂŁy ra khi v  chÂż khi a2 + b2 + ab = 1 (chÂŻng h€n 
a = b = 
1 
3 
).2 
2.36 Cho a; b; c 2 R. ChĂčng minh rÂŹng: 
a3  b3 
(a  b)3 + 
b3  c3 
(b  c)3 + 
c3  a3 
(c  a)3 
 
9 
4 
LĂ­i giÂŁi. 
B§t ÂŻng thĂčc cŠn chĂčng minh tĂ·ĂŹng Ă·ĂŹng vĂźi: 
P a2 + b2 + ab 
(a  b)2 
 
9 
4 
NhÂȘn th§y rÂŹng: 
a2 + ab + b2 
(a  b)2 = 
1 
4 
(a + b)2 + 
3 
4 
(a  b)2 
(a  b)2 
 
3 
4 
(a  b)2 
(a  b)2 = 
3 
4 
Thöc hi»n tĂ·ĂŹng tĂŒ cho hai b§t ÂŻng thĂčc cĂĄn l€i. 
PhÂČp chĂčng minh ho n t§t. ÂŻng thĂčc xÂŁy ra khi v  chÂż khi a + b + c = 0.2 
2.37 Cho a; b  0. ChĂčng minh rÂŹng: 
1 
a2 + 
1 
b2 + 
4 
a2 + b2 
 
32(a2 + b2) 
(a + b)4 
LĂ­i giÂŁi. 
SĂ» döng b§t ÂŻng thĂčc AM-GM cho hai sĂš dĂ·ĂŹng: 
1 
a2 + 
1 
b2 + 
4 
a2 + b2 = 
a2 + b2 
a2b2 + 
4 
a2 + b2 
r 
a2 + b2 
 2 
a2b2 : 
4 
a2 + b2 = 
4 
ab 
57
Ta sÂł chĂčng minh: 
4 
ab 
 
32(a2 + b2) 
(a + b)4 
hay 
8ab(a2 + b2)  (a + b)4 
p döng b§t ÂŻng thĂčc 4xy  (x + y)2: 
8ab(a2 + b2) = 4:2ab:a2 + b2)  (a2 + b2 + 2ab)2 = (a + b)4 
PhÂČp chĂčng minh ho n t§t. ÂŻng thĂčc xÂŁy ra khi v  chÂż khi a = b.2 
2.38 Cho a; b  0 thĂ€a mÂąn a2 + b2 + c2 + abc = 4. ChĂčng minh rÂŹng: 
a + b + c  3 
LĂ­i giÂŁi. 
CÂĄch 1: 
Theo nguy¶n lÂœ Dirichlet, trong ba sĂš a; b; c ­t sÂł cĂą hai sĂš cĂČng phÂœa vĂźi 1 tr¶n tröc sĂš. G¿£ sĂ» hai 
sĂš Ăą l  a v  b. Thž thÂŒ: 
(a  1)(b  1)  0 
hay 
ab  a + b  1 . 
M°t khÂĄc, theo giÂŁ thižt v  b§t ÂŻng thĂčc AM-GM cho hai sĂš dĂ·ĂŹng: 
4  c2 = a2 + b2 + abc  2ab + abc = ab(2 + c) 
hay 
(2  c)(2 + c)  ab(2 + c) 
hay 
2  c  ab 
Kžt hñp vĂźi b§t ÂŻng thĂčc ab  a + b  1 (chĂčng minh tr¶n), suy ra: 
2  c  ab  a + b  1 
hay 
a + b + c  3 . 
PhÂČp chĂčng minh ho n t§t. ÂŻng thĂčc xÂŁy ra khi v  chÂż khi a = b = c = 1 
CÂĄch 2: 
°t a = 
2x p 
(x + y) (x + z) 
; b = 
2y p 
(y + z) (y + x) 
; c = 
2z p 
(z + y) (z + x) 
Suy ra: 
a + b + c = 
P 
p 
p 
2x 
y + z (x + y) (y + z) (z + x) 
VÂŒ thž b§t ÂŻng thĂčc a + b + c  3 sÂł tĂ·ĂŹng Ă·ĂŹng vĂźi: P 
2x 
p 
(x + y)(y + z)(z + x) 
p 
y + z  3 
„y chÂœnh l  b§t ÂŻng thĂčc Schur vĂźi cÂĄc bižn 
p 
y + z; 
p 
y + x; 
p 
z + x. 
PhÂČp chĂčng minh ho n t§t. ÂŻng thĂčc xÂŁy ra khi v  chÂż khi a = b = c = 1.2 
CÂĄch 3: 
GiÂŁ sĂ» tçn t€i mĂ«t sĂš (cho sĂš Ăą l  a) trong ba sĂš a; b; c lĂźn hĂŹn 2. Khi Ăą, vÂŒ a; b; c dĂ·ĂŹng n¶n: 
a2 + b2 + c2 + abc = 4  4 + b2 + c2 + abc  4 (vĂŠ lÂœ!) 
Do Ăą a; b; c 2 (0; 2] 
TÞ gi£ thižt suy ra: 
a2 + abc + 
b2c2 
4 
= 4 + 
b2c2 
4 
 b2  c2 
58
hay  
a + 
bc 
2 
2 
= 
(4  b2)(4  c2) 
4 
Do b; c  2 n¶n suy ra: 
a + b + c = 
r 
(4  b2) (4  c2) 
4 
 
bc 
2 
+ b + c 
p döng b§t ÂŻng thĂčc AM-GM, ta cĂą: 
r 
(4  b2) (4  c2) 
4 
 
bc 
2 
+ b + c  
1 
2 
(4  b2 + 4  c2)  bc 
2 
+ b + c 
= 3  
 
b + c 
2 
2 
 1 
 3 . 
PhÂČp chĂčng minh ho n t§t. ÂŻng thĂčc xÂŁy ra khi v  chÂż khi a = b = c = 1.2 
NhÂȘn xÂČt: 
B§t ÂŻng thĂčc a + b + c  3 cĂŽng Ăłng vĂźi i·u ki»n a2 + b2 + c2 + 
3 
2 
abc = 
9 
2 
. 
2.39 Cho x; y; z  0. ChĂčng minh rÂŹng: 
1 
x 
+ 
1 
y 
+ 
1 
z 
 
36 
9 + x2y2 + y2z2 + z2x2 
LĂ­i giÂŁi. 
°t xy = a; yz = b; xz = c, b§t ÂŻng thĂčc trĂ° th nh: 
p 
abc 
(a + b + c)(a2 + b2 + c2 + 9)  36 
p döng b§t ÂŻng thĂčc AM-GM: 
a + b + c  3 3 p 
p 
(abc)4 
abc = 3 12 
q 
3 3 p (abc)2:3:3:3 = 12 12 
a2 + b2 + c2 + 9  3 3 p 
(abc)2 + 9 = 3 3 p 
(abc)2 + 3 + 3 + 3  4 4 
p 
(abc)2 
Nh„n vž theo vž hai b§t ÂŻng thĂčc tr¶n: 
(a + b + c)(a2 + b2 + c2 + 9)  3 3 p 
p 
(abc)2 = 36: 
abc:12 12 
p 
abc . 
PhÂČp chĂčng minh ho n t§t. ÂŻng thĂčc xÂŁy ra khi v  chÂż khi a = b = c = 1.2 
2.40 Cho a; b; c 0 thĂ€a mÂąn a2 + b2 + c2 = 3. ChĂčng minh rÂŹng: 
4 
a2 + b2 + 1 
 
4 
b2 + c2 + 1 
 
4 
c2 + a2 + 1 
 
 3(a + b + c)2 
LĂ­i giÂŁi. 
p döng b§t ÂŻng thĂčc Holder: 
4 
a2 + b2 + 1 
 
4 
b2 + c2 + 1 
 
4 
c2 + a2 + 1 
 
 
 
r 
3 
64 
(a2 + b2)(b2 + c2)(c2 + a2) 
+ 1 
3 
p döng b§t ÂŻng thĂčc AM-GM v  b§t ÂŻng thĂčc 3(x2 + y2 + z2)  (x + y + z)2: 
r 
3 
64 
(a2 + b2)(b2 + c2)(c2 + a2) 
+ 1 
3 
 
 
12 
2(a2 + b2 + c2) 
+ 1 
3 
= 27 = 9(a2 + b2 + c2)  3(a + b + c)2 
PhÂČp chĂčng minh ho n t§t. ÂŻng thĂčc xÂŁy ra khi v  chÂż khi a = b = c = 
r 
1 
3 
.2 
3.3 B i 3.1 žn b i 3.40 
3.1 Cho a; b; c; x; y; z  0 thĂ€a mÂąn x + y + z = 1. ChĂčng minh rÂŹng: 
ax + by + cz + 2 
p 
(xy + yz + zx)(ab + bc + ca)  a + b + c 
LĂ­i giÂŁi. 
59
p döng b§t ÂŻng thĂčc Cauchy-Schwarz: 
ax + by + cz + 2 
p 
(xy + yz + zx)(ab + bc + ca)  
pP 
a2: 
P 
x2 + 
p 
2 
P 
xy:2 
P 
ab 
 
p 
( 
P 
a2 + 2 
P 
P 
ab)( 
x2 + 2 
P 
xy) 
= a + b + c (do x + y + z = 1) 
PhÂČp chĂčng minh ho n t§t. ÂŻng thĂčc xÂŁy ra khi v  chÂż khi 
a 
x 
= 
b 
y 
= 
c 
z 
= 
1 
a + b + c 
= 
a + b + c 
x + y + z 
= 
a + b + c hay a + b + c = 1. 2 
3.2 Cho a; b; c  0 thĂ€a mÂąn a3 + b3 + c3 = 3. TÂŒm giÂĄ trĂ  lĂźn nh§t cĂ”a: 
P = a4b4 + b4c4 + c4a4 
LĂ­i giÂŁi. 
Ta chĂčng minh giÂĄ trĂ  lĂźn nh§t cĂ”a biÂșu thĂčc l  3. 
°t a3 = x; b3 = y; c3 = z, suy ra x + y + z = 3. 
p döng AM-GM 
3a4b4  a3b3(a3 + b3 + 1) 
Khi Ăą, ta chÂż cŠn chĂčng minh: 
xy(x + y + 1) + yz(y + z + 1) + zx(z + x + 1)  9 
Ă·a v· d€ng çng bÂȘc, ta cŠn chĂčng minh 
3 
P 
xy(x + y) + (x + y + z)(xy + yz + zx)  (x + y + z)3 
Sau khi khai triÂșn, b§t ÂŻng thĂčc trĂ° th nh: 
x3 + y3 + z3 + 3xyz  
P 
xy(x + y) 
Ăłng theo b§t ÂŻng thĂčc Schur. 
PhÂČp chĂčng minh ho n t§t. ÂŻng thĂčc xÂŁy ra khi v  chÂż khi a = b = c = 1.2 
3.3 Cho a; b; c  0 thĂ€a mÂąn a2 + b2 + c2 = 1. TÂŒm giÂĄ trĂ  nhĂ€ nh§t cĂ”a: 
P = 
a2 
b + c 
+ 
b2 
c + a 
+ 
c2 
a + b 
LĂ­i giÂŁi. 
CÂĄch 1: 
GiÂŁ sĂ» a  b  c. 
Ta sÂł cĂą hai dÂąy cĂČng chi·u: 8 
a2  b2  c2 
1 
b + c 
: 
 
1 
a + c 
 
1 
a + b 
p döng lŠn l֖t b§t cÂĄc b§t ÂŻng thĂčc Chebychep, giÂŁ thižt a2 + b2 + c2 = 1 v  b§t ÂŻng thĂčc 
P 1 
x 
 
P9 
x 
, ta cĂą: 
P = 
a2 
b + c 
+ 
b2 
c + a 
+ 
c2 
a + b 
 
1 
3 
(a2 + b2 + c2) 
 
1 
b + c 
+ 
1 
c + a 
+ 
1 
a + b 
 
 
3 
2 (a + b + c) 
L€i theo b§t ÂŻng thĂčc: 
3 = 3(a2 + b2 + c2)  (a + b + c)2 
Suy ra: 
a + b + c  
p 
3 . 
Do Ăą: 
P  
3 
2(a + b + c) 
 
p 
3 
2 
. 
VÂȘy minP = 
p 
3 
2 
, a = b = c = 
1 
p 
3 
. 
60
Xuctu.com tuyen-tap-bat-dang-thuc
Xuctu.com tuyen-tap-bat-dang-thuc
Xuctu.com tuyen-tap-bat-dang-thuc
Xuctu.com tuyen-tap-bat-dang-thuc
Xuctu.com tuyen-tap-bat-dang-thuc
Xuctu.com tuyen-tap-bat-dang-thuc
Xuctu.com tuyen-tap-bat-dang-thuc
Xuctu.com tuyen-tap-bat-dang-thuc
Xuctu.com tuyen-tap-bat-dang-thuc
Xuctu.com tuyen-tap-bat-dang-thuc
Xuctu.com tuyen-tap-bat-dang-thuc
Xuctu.com tuyen-tap-bat-dang-thuc
Xuctu.com tuyen-tap-bat-dang-thuc
Xuctu.com tuyen-tap-bat-dang-thuc
Xuctu.com tuyen-tap-bat-dang-thuc
Xuctu.com tuyen-tap-bat-dang-thuc
Xuctu.com tuyen-tap-bat-dang-thuc
Xuctu.com tuyen-tap-bat-dang-thuc
Xuctu.com tuyen-tap-bat-dang-thuc
Xuctu.com tuyen-tap-bat-dang-thuc
Xuctu.com tuyen-tap-bat-dang-thuc
Xuctu.com tuyen-tap-bat-dang-thuc
Xuctu.com tuyen-tap-bat-dang-thuc
Xuctu.com tuyen-tap-bat-dang-thuc
Xuctu.com tuyen-tap-bat-dang-thuc
Xuctu.com tuyen-tap-bat-dang-thuc
Xuctu.com tuyen-tap-bat-dang-thuc
Xuctu.com tuyen-tap-bat-dang-thuc
Xuctu.com tuyen-tap-bat-dang-thuc
Xuctu.com tuyen-tap-bat-dang-thuc
Xuctu.com tuyen-tap-bat-dang-thuc
Xuctu.com tuyen-tap-bat-dang-thuc
Xuctu.com tuyen-tap-bat-dang-thuc
Xuctu.com tuyen-tap-bat-dang-thuc
Xuctu.com tuyen-tap-bat-dang-thuc
Xuctu.com tuyen-tap-bat-dang-thuc
Xuctu.com tuyen-tap-bat-dang-thuc
Xuctu.com tuyen-tap-bat-dang-thuc
Xuctu.com tuyen-tap-bat-dang-thuc
Xuctu.com tuyen-tap-bat-dang-thuc
Xuctu.com tuyen-tap-bat-dang-thuc
Xuctu.com tuyen-tap-bat-dang-thuc
Xuctu.com tuyen-tap-bat-dang-thuc
Xuctu.com tuyen-tap-bat-dang-thuc
Xuctu.com tuyen-tap-bat-dang-thuc
Xuctu.com tuyen-tap-bat-dang-thuc
Xuctu.com tuyen-tap-bat-dang-thuc
Xuctu.com tuyen-tap-bat-dang-thuc
Xuctu.com tuyen-tap-bat-dang-thuc
Xuctu.com tuyen-tap-bat-dang-thuc
Xuctu.com tuyen-tap-bat-dang-thuc
Xuctu.com tuyen-tap-bat-dang-thuc
Xuctu.com tuyen-tap-bat-dang-thuc
Xuctu.com tuyen-tap-bat-dang-thuc
Xuctu.com tuyen-tap-bat-dang-thuc
Xuctu.com tuyen-tap-bat-dang-thuc
Xuctu.com tuyen-tap-bat-dang-thuc
Xuctu.com tuyen-tap-bat-dang-thuc
Xuctu.com tuyen-tap-bat-dang-thuc
Xuctu.com tuyen-tap-bat-dang-thuc
Xuctu.com tuyen-tap-bat-dang-thuc
Xuctu.com tuyen-tap-bat-dang-thuc
Xuctu.com tuyen-tap-bat-dang-thuc
Xuctu.com tuyen-tap-bat-dang-thuc
Xuctu.com tuyen-tap-bat-dang-thuc
Xuctu.com tuyen-tap-bat-dang-thuc
Xuctu.com tuyen-tap-bat-dang-thuc
Xuctu.com tuyen-tap-bat-dang-thuc
Xuctu.com tuyen-tap-bat-dang-thuc
Xuctu.com tuyen-tap-bat-dang-thuc
Xuctu.com tuyen-tap-bat-dang-thuc
Xuctu.com tuyen-tap-bat-dang-thuc
Xuctu.com tuyen-tap-bat-dang-thuc
Xuctu.com tuyen-tap-bat-dang-thuc
Xuctu.com tuyen-tap-bat-dang-thuc
Xuctu.com tuyen-tap-bat-dang-thuc
Xuctu.com tuyen-tap-bat-dang-thuc
Xuctu.com tuyen-tap-bat-dang-thuc
Xuctu.com tuyen-tap-bat-dang-thuc
Xuctu.com tuyen-tap-bat-dang-thuc
Xuctu.com tuyen-tap-bat-dang-thuc
Xuctu.com tuyen-tap-bat-dang-thuc
Xuctu.com tuyen-tap-bat-dang-thuc
Xuctu.com tuyen-tap-bat-dang-thuc
Xuctu.com tuyen-tap-bat-dang-thuc
Xuctu.com tuyen-tap-bat-dang-thuc
Xuctu.com tuyen-tap-bat-dang-thuc
Xuctu.com tuyen-tap-bat-dang-thuc
Xuctu.com tuyen-tap-bat-dang-thuc
Xuctu.com tuyen-tap-bat-dang-thuc
Xuctu.com tuyen-tap-bat-dang-thuc
Xuctu.com tuyen-tap-bat-dang-thuc
Xuctu.com tuyen-tap-bat-dang-thuc
Xuctu.com tuyen-tap-bat-dang-thuc
Xuctu.com tuyen-tap-bat-dang-thuc
Xuctu.com tuyen-tap-bat-dang-thuc
Xuctu.com tuyen-tap-bat-dang-thuc
Xuctu.com tuyen-tap-bat-dang-thuc
Xuctu.com tuyen-tap-bat-dang-thuc
Xuctu.com tuyen-tap-bat-dang-thuc
Xuctu.com tuyen-tap-bat-dang-thuc
Xuctu.com tuyen-tap-bat-dang-thuc
Xuctu.com tuyen-tap-bat-dang-thuc
Xuctu.com tuyen-tap-bat-dang-thuc
Xuctu.com tuyen-tap-bat-dang-thuc
Xuctu.com tuyen-tap-bat-dang-thuc
Xuctu.com tuyen-tap-bat-dang-thuc
Xuctu.com tuyen-tap-bat-dang-thuc
Xuctu.com tuyen-tap-bat-dang-thuc
Xuctu.com tuyen-tap-bat-dang-thuc
Xuctu.com tuyen-tap-bat-dang-thuc
Xuctu.com tuyen-tap-bat-dang-thuc
Xuctu.com tuyen-tap-bat-dang-thuc
Xuctu.com tuyen-tap-bat-dang-thuc
Xuctu.com tuyen-tap-bat-dang-thuc
Xuctu.com tuyen-tap-bat-dang-thuc
Xuctu.com tuyen-tap-bat-dang-thuc
Xuctu.com tuyen-tap-bat-dang-thuc
Xuctu.com tuyen-tap-bat-dang-thuc
Xuctu.com tuyen-tap-bat-dang-thuc
Xuctu.com tuyen-tap-bat-dang-thuc
Xuctu.com tuyen-tap-bat-dang-thuc
Xuctu.com tuyen-tap-bat-dang-thuc
Xuctu.com tuyen-tap-bat-dang-thuc
Xuctu.com tuyen-tap-bat-dang-thuc
Xuctu.com tuyen-tap-bat-dang-thuc
Xuctu.com tuyen-tap-bat-dang-thuc
Xuctu.com tuyen-tap-bat-dang-thuc
Xuctu.com tuyen-tap-bat-dang-thuc
Xuctu.com tuyen-tap-bat-dang-thuc
Xuctu.com tuyen-tap-bat-dang-thuc
Xuctu.com tuyen-tap-bat-dang-thuc
Xuctu.com tuyen-tap-bat-dang-thuc
Xuctu.com tuyen-tap-bat-dang-thuc
Xuctu.com tuyen-tap-bat-dang-thuc
Xuctu.com tuyen-tap-bat-dang-thuc
Xuctu.com tuyen-tap-bat-dang-thuc
Xuctu.com tuyen-tap-bat-dang-thuc
Xuctu.com tuyen-tap-bat-dang-thuc
Xuctu.com tuyen-tap-bat-dang-thuc
Xuctu.com tuyen-tap-bat-dang-thuc
Xuctu.com tuyen-tap-bat-dang-thuc
Xuctu.com tuyen-tap-bat-dang-thuc
Xuctu.com tuyen-tap-bat-dang-thuc
Xuctu.com tuyen-tap-bat-dang-thuc
Xuctu.com tuyen-tap-bat-dang-thuc
Xuctu.com tuyen-tap-bat-dang-thuc
Xuctu.com tuyen-tap-bat-dang-thuc
Xuctu.com tuyen-tap-bat-dang-thuc
Xuctu.com tuyen-tap-bat-dang-thuc
Xuctu.com tuyen-tap-bat-dang-thuc
Xuctu.com tuyen-tap-bat-dang-thuc
Xuctu.com tuyen-tap-bat-dang-thuc
Xuctu.com tuyen-tap-bat-dang-thuc
Xuctu.com tuyen-tap-bat-dang-thuc
Xuctu.com tuyen-tap-bat-dang-thuc
Xuctu.com tuyen-tap-bat-dang-thuc
Xuctu.com tuyen-tap-bat-dang-thuc
Xuctu.com tuyen-tap-bat-dang-thuc
Xuctu.com tuyen-tap-bat-dang-thuc
Xuctu.com tuyen-tap-bat-dang-thuc
Xuctu.com tuyen-tap-bat-dang-thuc
Xuctu.com tuyen-tap-bat-dang-thuc
Xuctu.com tuyen-tap-bat-dang-thuc
Xuctu.com tuyen-tap-bat-dang-thuc
Xuctu.com tuyen-tap-bat-dang-thuc
Xuctu.com tuyen-tap-bat-dang-thuc
Xuctu.com tuyen-tap-bat-dang-thuc
Xuctu.com tuyen-tap-bat-dang-thuc
Xuctu.com tuyen-tap-bat-dang-thuc
Xuctu.com tuyen-tap-bat-dang-thuc
Xuctu.com tuyen-tap-bat-dang-thuc
Xuctu.com tuyen-tap-bat-dang-thuc
Xuctu.com tuyen-tap-bat-dang-thuc
Xuctu.com tuyen-tap-bat-dang-thuc

More Related Content

What's hot

100 cau hoi ve hop dong trong xay dung 2010
100 cau hoi ve hop dong trong xay dung 2010100 cau hoi ve hop dong trong xay dung 2010
100 cau hoi ve hop dong trong xay dung 2010Hung Nguyen
 
Bai Giang 1
Bai Giang 1Bai Giang 1
Bai Giang 1nbb3i
 
Chuong 14 16
Chuong 14   16Chuong 14   16
Chuong 14 16Vcoi Vit
 
KáșŸT Cáș€U Gáș CH ĐÁ GHỖ : CHÆŻÆ NG 2
KáșŸT Cáș€U Gáș CH ĐÁ GHỖ : CHÆŻÆ NG 2KáșŸT Cáș€U Gáș CH ĐÁ GHỖ : CHÆŻÆ NG 2
KáșŸT Cáș€U Gáș CH ĐÁ GHỖ : CHÆŻÆ NG 2nguyenxuan8989898798
 
Ky thuat lap trinh
Ky thuat lap trinhKy thuat lap trinh
Ky thuat lap trinhptquang160492
 
Tỏ lĂČng (thuáș­t hoĂ i) PháșĄm ngĆ© LĂŁo - ngữ văn lớp 10
Tỏ lĂČng (thuáș­t hoĂ i) PháșĄm ngĆ© LĂŁo - ngữ văn lớp 10Tỏ lĂČng (thuáș­t hoĂ i) PháșĄm ngĆ© LĂŁo - ngữ văn lớp 10
Tỏ lĂČng (thuáș­t hoĂ i) PháșĄm ngĆ© LĂŁo - ngữ văn lớp 10Nguyễn Háș­u
 
Nhung dieu can biet ve hoat dong gioi tinh
Nhung dieu can biet ve hoat dong gioi tinhNhung dieu can biet ve hoat dong gioi tinh
Nhung dieu can biet ve hoat dong gioi tinhLÆ°u CĂŽng HoĂ n
 
Thai giao day con tu trong bung me
Thai giao   day con tu trong bung meThai giao   day con tu trong bung me
Thai giao day con tu trong bung meSelf-employed
 
CĂČn chĂșt gĂŹ để nhớ nguyễn nháș­t ĂĄnhtruonghocso.com
CĂČn chĂșt gĂŹ để nhớ  nguyễn nháș­t ĂĄnhtruonghocso.comCĂČn chĂșt gĂŹ để nhớ  nguyễn nháș­t ĂĄnhtruonghocso.com
CĂČn chĂșt gĂŹ để nhớ nguyễn nháș­t ĂĄnhtruonghocso.comTháșż Giới Tinh Hoa
 
TỐI ÆŻU HÓA LÆŻá»šI ĐIỆN PHÂN PHỐI THÀNH PHỐ ĐỒNG HÀ - TỈNH QUáșąNG TRỊ dc98e6a1
TỐI ÆŻU HÓA LÆŻá»šI ĐIỆN PHÂN PHỐI THÀNH PHỐ ĐỒNG HÀ - TỈNH QUáșąNG TRỊ dc98e6a1TỐI ÆŻU HÓA LÆŻá»šI ĐIỆN PHÂN PHỐI THÀNH PHỐ ĐỒNG HÀ - TỈNH QUáșąNG TRỊ dc98e6a1
TỐI ÆŻU HÓA LÆŻá»šI ĐIỆN PHÂN PHỐI THÀNH PHỐ ĐỒNG HÀ - TỈNH QUáșąNG TRỊ dc98e6a1nataliej4
 
Hoa Dai Cuong
Hoa Dai CuongHoa Dai Cuong
Hoa Dai Cuongclayqn88
 

What's hot (17)

Luáș­n văn: Chuỗi LAURENT P-ADIC
Luáș­n văn: Chuỗi LAURENT P-ADICLuáș­n văn: Chuỗi LAURENT P-ADIC
Luáș­n văn: Chuỗi LAURENT P-ADIC
 
100 cau hoi ve hop dong trong xay dung 2010
100 cau hoi ve hop dong trong xay dung 2010100 cau hoi ve hop dong trong xay dung 2010
100 cau hoi ve hop dong trong xay dung 2010
 
Bai Giang 1
Bai Giang 1Bai Giang 1
Bai Giang 1
 
Chuong 14 16
Chuong 14   16Chuong 14   16
Chuong 14 16
 
KáșŸT Cáș€U Gáș CH ĐÁ GHỖ : CHÆŻÆ NG 2
KáșŸT Cáș€U Gáș CH ĐÁ GHỖ : CHÆŻÆ NG 2KáșŸT Cáș€U Gáș CH ĐÁ GHỖ : CHÆŻÆ NG 2
KáșŸT Cáș€U Gáș CH ĐÁ GHỖ : CHÆŻÆ NG 2
 
Ky thuat lap trinh
Ky thuat lap trinhKy thuat lap trinh
Ky thuat lap trinh
 
Tuan 27
Tuan 27Tuan 27
Tuan 27
 
Tỏ lĂČng (thuáș­t hoĂ i) PháșĄm ngĆ© LĂŁo - ngữ văn lớp 10
Tỏ lĂČng (thuáș­t hoĂ i) PháșĄm ngĆ© LĂŁo - ngữ văn lớp 10Tỏ lĂČng (thuáș­t hoĂ i) PháșĄm ngĆ© LĂŁo - ngữ văn lớp 10
Tỏ lĂČng (thuáș­t hoĂ i) PháșĄm ngĆ© LĂŁo - ngữ văn lớp 10
 
Nhung dieu can biet ve hoat dong gioi tinh
Nhung dieu can biet ve hoat dong gioi tinhNhung dieu can biet ve hoat dong gioi tinh
Nhung dieu can biet ve hoat dong gioi tinh
 
Luáș­n văn: Về dáșĄng chuáș©n Edwards vĂ  một vĂ i ứng dỄng, HAY
Luáș­n văn: Về dáșĄng chuáș©n Edwards vĂ  một vĂ i ứng dỄng, HAYLuáș­n văn: Về dáșĄng chuáș©n Edwards vĂ  một vĂ i ứng dỄng, HAY
Luáș­n văn: Về dáșĄng chuáș©n Edwards vĂ  một vĂ i ứng dỄng, HAY
 
Thai giao day con tu trong bung me
Thai giao   day con tu trong bung meThai giao   day con tu trong bung me
Thai giao day con tu trong bung me
 
De thi phuong phap so va phan mem ung dung
De thi phuong phap so va phan mem ung dungDe thi phuong phap so va phan mem ung dung
De thi phuong phap so va phan mem ung dung
 
CĂČn chĂșt gĂŹ để nhớ nguyễn nháș­t ĂĄnhtruonghocso.com
CĂČn chĂșt gĂŹ để nhớ  nguyễn nháș­t ĂĄnhtruonghocso.comCĂČn chĂșt gĂŹ để nhớ  nguyễn nháș­t ĂĄnhtruonghocso.com
CĂČn chĂșt gĂŹ để nhớ nguyễn nháș­t ĂĄnhtruonghocso.com
 
TỐI ÆŻU HÓA LÆŻá»šI ĐIỆN PHÂN PHỐI THÀNH PHỐ ĐỒNG HÀ - TỈNH QUáșąNG TRỊ dc98e6a1
TỐI ÆŻU HÓA LÆŻá»šI ĐIỆN PHÂN PHỐI THÀNH PHỐ ĐỒNG HÀ - TỈNH QUáșąNG TRỊ dc98e6a1TỐI ÆŻU HÓA LÆŻá»šI ĐIỆN PHÂN PHỐI THÀNH PHỐ ĐỒNG HÀ - TỈNH QUáșąNG TRỊ dc98e6a1
TỐI ÆŻU HÓA LÆŻá»šI ĐIỆN PHÂN PHỐI THÀNH PHỐ ĐỒNG HÀ - TỈNH QUáșąNG TRỊ dc98e6a1
 
Hoa Dai Cuong
Hoa Dai CuongHoa Dai Cuong
Hoa Dai Cuong
 
Phu dao hs y k lĂ­p 5
Phu dao hs y k lĂ­p 5Phu dao hs y k lĂ­p 5
Phu dao hs y k lĂ­p 5
 
Do Hau Mon Ok
Do Hau Mon OkDo Hau Mon Ok
Do Hau Mon Ok
 

Similar to Xuctu.com tuyen-tap-bat-dang-thuc

Tuyển chọn 410 Hệ phÆ°ÆĄng trĂŹnh ver2 Nguyễn Minh Tuáș„n
Tuyển chọn 410 Hệ phÆ°ÆĄng trĂŹnh ver2 Nguyễn Minh Tuáș„nTuyển chọn 410 Hệ phÆ°ÆĄng trĂŹnh ver2 Nguyễn Minh Tuáș„n
Tuyển chọn 410 Hệ phÆ°ÆĄng trĂŹnh ver2 Nguyễn Minh Tuáș„nDÆ°ÆĄng Ngọc Taeny
 
Giaotrinhxacsuatthongke dh chu y
Giaotrinhxacsuatthongke dh chu yGiaotrinhxacsuatthongke dh chu y
Giaotrinhxacsuatthongke dh chu yhaychotoi
 
Váș­n DỄng PhĂ©p Đáșżm NĂąng Cao VĂ o GiáșŁi Một Số BĂ i ToĂĄn Thi Học Sinh Giỏi.doc
Váș­n DỄng PhĂ©p Đáșżm NĂąng Cao VĂ o GiáșŁi Một Số BĂ i ToĂĄn Thi Học Sinh Giỏi.docVáș­n DỄng PhĂ©p Đáșżm NĂąng Cao VĂ o GiáșŁi Một Số BĂ i ToĂĄn Thi Học Sinh Giỏi.doc
Váș­n DỄng PhĂ©p Đáșżm NĂąng Cao VĂ o GiáșŁi Một Số BĂ i ToĂĄn Thi Học Sinh Giỏi.docDV ViĂȘ́t LuĂąÌŁn văn luanvanmaster.com ZALO 0973287149
 
Chuong i phan i tinhthechatran ma
Chuong i phan i tinhthechatran maChuong i phan i tinhthechatran ma
Chuong i phan i tinhthechatran mawww. mientayvn.com
 
hệ thống cĂŽng thức trong cÆĄ học đáș„t
hệ thống cĂŽng thức trong cÆĄ học đáș„thệ thống cĂŽng thức trong cÆĄ học đáș„t
hệ thống cĂŽng thức trong cÆĄ học đáș„tAnh Anh
 
Hệ thống cĂŽng thức cÆĄ học đáș„t
Hệ thống cĂŽng thức cÆĄ học đáș„tHệ thống cĂŽng thức cÆĄ học đáș„t
Hệ thống cĂŽng thức cÆĄ học đáș„tTtx Love
 
Luáș­n văn: Háș„p thỄ sĂłng điện từ trong Graphene Ä‘ÆĄn lớp dưới áșŁnh hưởng của từ t...
Luáș­n văn: Háș„p thỄ sĂłng điện từ trong Graphene Ä‘ÆĄn lớp dưới áșŁnh hưởng của từ t...Luáș­n văn: Háș„p thỄ sĂłng điện từ trong Graphene Ä‘ÆĄn lớp dưới áșŁnh hưởng của từ t...
Luáș­n văn: Háș„p thỄ sĂłng điện từ trong Graphene Ä‘ÆĄn lớp dưới áșŁnh hưởng của từ t...Dịch vỄ viáșżt thuĂȘ Luáș­n Văn - ZALO 0932091562
 
Bat dang thuc
Bat dang thucBat dang thuc
Bat dang thucthanhgand
 
Ky thuat lap_trinh
Ky thuat lap_trinhKy thuat lap_trinh
Ky thuat lap_trinhtienhien110293
 
Sach bat dang thuc rat hay
Sach bat dang thuc rat haySach bat dang thuc rat hay
Sach bat dang thuc rat hayTuĂąn NgĂŽ
 
Báș„t đáșłng thức mathscope
Báș„t đáșłng thức mathscopeBáș„t đáșłng thức mathscope
Báș„t đáșłng thức mathscopePhĂșc VĂ”
 
Bai giang he thong vien thong 1
Bai giang he thong vien thong 1Bai giang he thong vien thong 1
Bai giang he thong vien thong 1Huynh MVT
 
PhĂąn loáșĄi bĂ i táș­p tĂ­nh thể tĂ­ch khối trĂČn xoay truonghocso.com
PhĂąn loáșĄi bĂ i táș­p tĂ­nh thể tĂ­ch khối trĂČn xoay   truonghocso.comPhĂąn loáșĄi bĂ i táș­p tĂ­nh thể tĂ­ch khối trĂČn xoay   truonghocso.com
PhĂąn loáșĄi bĂ i táș­p tĂ­nh thể tĂ­ch khối trĂČn xoay truonghocso.comTháșż Giới Tinh Hoa
 
Đề cÆ°ÆĄng ĂŽn thi tráșŻc địa tráșŻc địa đáșĄi cÆ°ÆĄng
Đề cÆ°ÆĄng ĂŽn thi tráșŻc địa tráșŻc địa đáșĄi cÆ°ÆĄng Đề cÆ°ÆĄng ĂŽn thi tráșŻc địa tráșŻc địa đáșĄi cÆ°ÆĄng
Đề cÆ°ÆĄng ĂŽn thi tráșŻc địa tráșŻc địa đáșĄi cÆ°ÆĄng Ttx Love
 

Similar to Xuctu.com tuyen-tap-bat-dang-thuc (20)

Đồng quy vĂ  tháșłng hĂ ng Trong hĂŹnh học pháșłng.doc
Đồng quy vĂ  tháșłng hĂ ng Trong hĂŹnh học pháșłng.docĐồng quy vĂ  tháșłng hĂ ng Trong hĂŹnh học pháșłng.doc
Đồng quy vĂ  tháșłng hĂ ng Trong hĂŹnh học pháșłng.doc
 
Tuyen tap he hay
Tuyen tap he hayTuyen tap he hay
Tuyen tap he hay
 
Tuyển chọn 410 Hệ phÆ°ÆĄng trĂŹnh ver2 Nguyễn Minh Tuáș„n
Tuyển chọn 410 Hệ phÆ°ÆĄng trĂŹnh ver2 Nguyễn Minh Tuáș„nTuyển chọn 410 Hệ phÆ°ÆĄng trĂŹnh ver2 Nguyễn Minh Tuáș„n
Tuyển chọn 410 Hệ phÆ°ÆĄng trĂŹnh ver2 Nguyễn Minh Tuáș„n
 
Giaotrinhxacsuatthongke dh chu y
Giaotrinhxacsuatthongke dh chu yGiaotrinhxacsuatthongke dh chu y
Giaotrinhxacsuatthongke dh chu y
 
Váș­n DỄng PhĂ©p Đáșżm NĂąng Cao VĂ o GiáșŁi Một Số BĂ i ToĂĄn Thi Học Sinh Giỏi.doc
Váș­n DỄng PhĂ©p Đáșżm NĂąng Cao VĂ o GiáșŁi Một Số BĂ i ToĂĄn Thi Học Sinh Giỏi.docVáș­n DỄng PhĂ©p Đáșżm NĂąng Cao VĂ o GiáșŁi Một Số BĂ i ToĂĄn Thi Học Sinh Giỏi.doc
Váș­n DỄng PhĂ©p Đáșżm NĂąng Cao VĂ o GiáșŁi Một Số BĂ i ToĂĄn Thi Học Sinh Giỏi.doc
 
Chuong i phan i tinhthechatran ma
Chuong i phan i tinhthechatran maChuong i phan i tinhthechatran ma
Chuong i phan i tinhthechatran ma
 
hệ thống cĂŽng thức trong cÆĄ học đáș„t
hệ thống cĂŽng thức trong cÆĄ học đáș„thệ thống cĂŽng thức trong cÆĄ học đáș„t
hệ thống cĂŽng thức trong cÆĄ học đáș„t
 
Hệ thống cĂŽng thức cÆĄ học đáș„t
Hệ thống cĂŽng thức cÆĄ học đáș„tHệ thống cĂŽng thức cÆĄ học đáș„t
Hệ thống cĂŽng thức cÆĄ học đáș„t
 
Luáș­n văn: Háș„p thỄ sĂłng điện từ trong Graphene Ä‘ÆĄn lớp dưới áșŁnh hưởng của từ t...
Luáș­n văn: Háș„p thỄ sĂłng điện từ trong Graphene Ä‘ÆĄn lớp dưới áșŁnh hưởng của từ t...Luáș­n văn: Háș„p thỄ sĂłng điện từ trong Graphene Ä‘ÆĄn lớp dưới áșŁnh hưởng của từ t...
Luáș­n văn: Háș„p thỄ sĂłng điện từ trong Graphene Ä‘ÆĄn lớp dưới áșŁnh hưởng của từ t...
 
Luáș­n văn: Háș„p thỄ sĂłng điện từ trong Graphene Ä‘ÆĄn lớp, HAY
Luáș­n văn: Háș„p thỄ sĂłng điện từ trong Graphene Ä‘ÆĄn lớp, HAYLuáș­n văn: Háș„p thỄ sĂłng điện từ trong Graphene Ä‘ÆĄn lớp, HAY
Luáș­n văn: Háș„p thỄ sĂłng điện từ trong Graphene Ä‘ÆĄn lớp, HAY
 
Bat dang thuc
Bat dang thucBat dang thuc
Bat dang thuc
 
Ky thuat lap_trinh
Ky thuat lap_trinhKy thuat lap_trinh
Ky thuat lap_trinh
 
Ká»č thuáș­t láș­p trĂŹnh.
Ká»č thuáș­t láș­p trĂŹnh.Ká»č thuáș­t láș­p trĂŹnh.
Ká»č thuáș­t láș­p trĂŹnh.
 
TĂ­nh báș„t kháșŁ quy Của đa thức với hệ số nguyĂȘn.doc
TĂ­nh báș„t kháșŁ quy Của đa thức với hệ số nguyĂȘn.docTĂ­nh báș„t kháșŁ quy Của đa thức với hệ số nguyĂȘn.doc
TĂ­nh báș„t kháșŁ quy Của đa thức với hệ số nguyĂȘn.doc
 
Sach bat dang thuc rat hay
Sach bat dang thuc rat haySach bat dang thuc rat hay
Sach bat dang thuc rat hay
 
Báș„t đáșłng thức mathscope
Báș„t đáșłng thức mathscopeBáș„t đáșłng thức mathscope
Báș„t đáșłng thức mathscope
 
Bai giang he thong vien thong 1
Bai giang he thong vien thong 1Bai giang he thong vien thong 1
Bai giang he thong vien thong 1
 
PhĂąn loáșĄi bĂ i táș­p tĂ­nh thể tĂ­ch khối trĂČn xoay truonghocso.com
PhĂąn loáșĄi bĂ i táș­p tĂ­nh thể tĂ­ch khối trĂČn xoay   truonghocso.comPhĂąn loáșĄi bĂ i táș­p tĂ­nh thể tĂ­ch khối trĂČn xoay   truonghocso.com
PhĂąn loáșĄi bĂ i táș­p tĂ­nh thể tĂ­ch khối trĂČn xoay truonghocso.com
 
Đề cÆ°ÆĄng ĂŽn thi tráșŻc địa tráșŻc địa đáșĄi cÆ°ÆĄng
Đề cÆ°ÆĄng ĂŽn thi tráșŻc địa tráșŻc địa đáșĄi cÆ°ÆĄng Đề cÆ°ÆĄng ĂŽn thi tráșŻc địa tráșŻc địa đáșĄi cÆ°ÆĄng
Đề cÆ°ÆĄng ĂŽn thi tráșŻc địa tráșŻc địa đáșĄi cÆ°ÆĄng
 
Luáș­n văn: TĂ­ch phĂąn Volkenborn, HAY
Luáș­n văn: TĂ­ch phĂąn Volkenborn, HAYLuáș­n văn: TĂ­ch phĂąn Volkenborn, HAY
Luáș­n văn: TĂ­ch phĂąn Volkenborn, HAY
 

More from Minh Đức

Untitled Presentation
Untitled PresentationUntitled Presentation
Untitled PresentationMinh Đức
 
De l10-dhqghn-2013-toan
De l10-dhqghn-2013-toanDe l10-dhqghn-2013-toan
De l10-dhqghn-2013-toanMinh Đức
 
Xuctu.com chuyen-de-so-hoc-vmf
Xuctu.com chuyen-de-so-hoc-vmfXuctu.com chuyen-de-so-hoc-vmf
Xuctu.com chuyen-de-so-hoc-vmfMinh Đức
 
50dethihsgtoan9 140928111901-phpapp01
50dethihsgtoan9 140928111901-phpapp0150dethihsgtoan9 140928111901-phpapp01
50dethihsgtoan9 140928111901-phpapp01Minh Đức
 
Xuctu.com cd pt_hpt_bpt_phuong_trinh_bat_phuong_trinh_sieu_viet
Xuctu.com cd pt_hpt_bpt_phuong_trinh_bat_phuong_trinh_sieu_vietXuctu.com cd pt_hpt_bpt_phuong_trinh_bat_phuong_trinh_sieu_viet
Xuctu.com cd pt_hpt_bpt_phuong_trinh_bat_phuong_trinh_sieu_vietMinh Đức
 
Xuctu.com hoc nhanh-daiso
Xuctu.com hoc nhanh-daisoXuctu.com hoc nhanh-daiso
Xuctu.com hoc nhanh-daisoMinh Đức
 
Xuctu.com de thi_tuyen_sinh_10_lhp_tdn_chuyen_dhsp(hcm)
Xuctu.com de thi_tuyen_sinh_10_lhp_tdn_chuyen_dhsp(hcm)Xuctu.com de thi_tuyen_sinh_10_lhp_tdn_chuyen_dhsp(hcm)
Xuctu.com de thi_tuyen_sinh_10_lhp_tdn_chuyen_dhsp(hcm)Minh Đức
 
Xuctu.com tuyen chon_cac_chuyen_de_tren_tap_chi_toan_hoc_va_tuoi_tre
Xuctu.com tuyen chon_cac_chuyen_de_tren_tap_chi_toan_hoc_va_tuoi_treXuctu.com tuyen chon_cac_chuyen_de_tren_tap_chi_toan_hoc_va_tuoi_tre
Xuctu.com tuyen chon_cac_chuyen_de_tren_tap_chi_toan_hoc_va_tuoi_treMinh Đức
 
Xuctu.com sach ky yeu bdhsg dong thap 2013-2014
Xuctu.com sach ky yeu bdhsg dong thap 2013-2014Xuctu.com sach ky yeu bdhsg dong thap 2013-2014
Xuctu.com sach ky yeu bdhsg dong thap 2013-2014Minh Đức
 
Xuctu.com tuyen tap-de-thi-olympic-52-de-thi-va-giai-tap-1
Xuctu.com tuyen tap-de-thi-olympic-52-de-thi-va-giai-tap-1Xuctu.com tuyen tap-de-thi-olympic-52-de-thi-va-giai-tap-1
Xuctu.com tuyen tap-de-thi-olympic-52-de-thi-va-giai-tap-1Minh Đức
 
Xuctu.com ch de-cuctri-gtln-gtnn
Xuctu.com ch de-cuctri-gtln-gtnnXuctu.com ch de-cuctri-gtln-gtnn
Xuctu.com ch de-cuctri-gtln-gtnnMinh Đức
 
Xuctu.com 30 de_thi_hoc_sinh_gioi_thcs_nguyen_vu_thanh
Xuctu.com 30 de_thi_hoc_sinh_gioi_thcs_nguyen_vu_thanhXuctu.com 30 de_thi_hoc_sinh_gioi_thcs_nguyen_vu_thanh
Xuctu.com 30 de_thi_hoc_sinh_gioi_thcs_nguyen_vu_thanhMinh Đức
 

More from Minh Đức (12)

Untitled Presentation
Untitled PresentationUntitled Presentation
Untitled Presentation
 
De l10-dhqghn-2013-toan
De l10-dhqghn-2013-toanDe l10-dhqghn-2013-toan
De l10-dhqghn-2013-toan
 
Xuctu.com chuyen-de-so-hoc-vmf
Xuctu.com chuyen-de-so-hoc-vmfXuctu.com chuyen-de-so-hoc-vmf
Xuctu.com chuyen-de-so-hoc-vmf
 
50dethihsgtoan9 140928111901-phpapp01
50dethihsgtoan9 140928111901-phpapp0150dethihsgtoan9 140928111901-phpapp01
50dethihsgtoan9 140928111901-phpapp01
 
Xuctu.com cd pt_hpt_bpt_phuong_trinh_bat_phuong_trinh_sieu_viet
Xuctu.com cd pt_hpt_bpt_phuong_trinh_bat_phuong_trinh_sieu_vietXuctu.com cd pt_hpt_bpt_phuong_trinh_bat_phuong_trinh_sieu_viet
Xuctu.com cd pt_hpt_bpt_phuong_trinh_bat_phuong_trinh_sieu_viet
 
Xuctu.com hoc nhanh-daiso
Xuctu.com hoc nhanh-daisoXuctu.com hoc nhanh-daiso
Xuctu.com hoc nhanh-daiso
 
Xuctu.com de thi_tuyen_sinh_10_lhp_tdn_chuyen_dhsp(hcm)
Xuctu.com de thi_tuyen_sinh_10_lhp_tdn_chuyen_dhsp(hcm)Xuctu.com de thi_tuyen_sinh_10_lhp_tdn_chuyen_dhsp(hcm)
Xuctu.com de thi_tuyen_sinh_10_lhp_tdn_chuyen_dhsp(hcm)
 
Xuctu.com tuyen chon_cac_chuyen_de_tren_tap_chi_toan_hoc_va_tuoi_tre
Xuctu.com tuyen chon_cac_chuyen_de_tren_tap_chi_toan_hoc_va_tuoi_treXuctu.com tuyen chon_cac_chuyen_de_tren_tap_chi_toan_hoc_va_tuoi_tre
Xuctu.com tuyen chon_cac_chuyen_de_tren_tap_chi_toan_hoc_va_tuoi_tre
 
Xuctu.com sach ky yeu bdhsg dong thap 2013-2014
Xuctu.com sach ky yeu bdhsg dong thap 2013-2014Xuctu.com sach ky yeu bdhsg dong thap 2013-2014
Xuctu.com sach ky yeu bdhsg dong thap 2013-2014
 
Xuctu.com tuyen tap-de-thi-olympic-52-de-thi-va-giai-tap-1
Xuctu.com tuyen tap-de-thi-olympic-52-de-thi-va-giai-tap-1Xuctu.com tuyen tap-de-thi-olympic-52-de-thi-va-giai-tap-1
Xuctu.com tuyen tap-de-thi-olympic-52-de-thi-va-giai-tap-1
 
Xuctu.com ch de-cuctri-gtln-gtnn
Xuctu.com ch de-cuctri-gtln-gtnnXuctu.com ch de-cuctri-gtln-gtnn
Xuctu.com ch de-cuctri-gtln-gtnn
 
Xuctu.com 30 de_thi_hoc_sinh_gioi_thcs_nguyen_vu_thanh
Xuctu.com 30 de_thi_hoc_sinh_gioi_thcs_nguyen_vu_thanhXuctu.com 30 de_thi_hoc_sinh_gioi_thcs_nguyen_vu_thanh
Xuctu.com 30 de_thi_hoc_sinh_gioi_thcs_nguyen_vu_thanh
 

Xuctu.com tuyen-tap-bat-dang-thuc

  • 1.
  • 2.
  • 3. Möc löc LĂ­i nĂąi Šu 4 CÂĄc th nh vi¶n tham gia bi¶n so€n 5 1 CÂĄc b§t ÂŻng thĂčc kinh iÂșn 6 1.1 B§t ÂŻng thĂčc giĂșa trung bÂŒnh cĂ«ng v  trung bÂŒnh nh„n (AM-GM). . . . . . . . . 6 1.2 B§t ÂŻng thĂčc giĂșa trung bÂŒnh cĂ«ng v  trung bÂŒnh i·u ho  (AM-HM). . . . . . . 6 1.3 B§t ÂŻng thĂčc Cauchy - Schwarz. . . . . . . . . . . . . . . . . . . . . . . . . . . . 6 1.4 B§t ÂŻng thĂčc Holder. . . . . . . . . . . . . . . . . . . . . . . . . . . . . . . . . . 7 1.5 B§t ÂŻng thĂčc Chebyshev. . . . . . . . . . . . . . . . . . . . . . . . . . . . . . . . 7 1.6 B§t ÂŻng thĂčc Minkowski. . . . . . . . . . . . . . . . . . . . . . . . . . . . . . . . 7 1.7 B§t ÂŻng thĂčc Schur. . . . . . . . . . . . . . . . . . . . . . . . . . . . . . . . . . . 7 1.8 B§t ÂŻng thĂčc Vornicu - Schur. . . . . . . . . . . . . . . . . . . . . . . . . . . . . 8 1.9 B§t ÂŻng thĂčc Bernoulli. . . . . . . . . . . . . . . . . . . . . . . . . . . . . . . . . 8 1.10 Ba ti¶u chu©n SOS thĂ·Ă­ng g°p. . . . . . . . . . . . . . . . . . . . . . . . . . . . . 9 2 MĂ«t sĂš ÂĄnh giÂĄ quen thuĂ«c 9 3 TuyÂșn tÂȘp b§t ÂŻng thĂčc 10 3.1 B i 1.1 žn b i 1.40 . . . . . . . . . . . . . . . . . . . . . . . . . . . . . . . . . . . 10 3.2 B i 2.1 žn b i 2.40 . . . . . . . . . . . . . . . . . . . . . . . . . . . . . . . . . . . 39 3.3 B i 3.1 žn b i 3.40 . . . . . . . . . . . . . . . . . . . . . . . . . . . . . . . . . . . 59 3.4 B i 4.1 žn b i 4.40 . . . . . . . . . . . . . . . . . . . . . . . . . . . . . . . . . . . 80 3.5 B i 5.1 žn b i 5.40 . . . . . . . . . . . . . . . . . . . . . . . . . . . . . . . . . . . 104 3.6 B i 6.1 žn b i 6.40 . . . . . . . . . . . . . . . . . . . . . . . . . . . . . . . . . . . 132 3.7 B i 7.1 žn b i 7.40 . . . . . . . . . . . . . . . . . . . . . . . . . . . . . . . . . . . 148 3.8 B i 8.1 žn b i 8.40 . . . . . . . . . . . . . . . . . . . . . . . . . . . . . . . . . . . 168 3.9 B i 9.1 žn b i 9.40 . . . . . . . . . . . . . . . . . . . . . . . . . . . . . . . . . . . 193 3.10 B i 10.1 žn b i 10.40 . . . . . . . . . . . . . . . . . . . . . . . . . . . . . . . . . 211 3
  • 4. LĂ­i nĂąi Šu BiÂșn všn mÂąi nh§p nhĂŠ vĂźi nhĂșng con sĂąng d€t v o bĂ­, thuy·n všn mÂąi l¶nh ¶nh theo tĂžng con sĂąng i v o €i dĂ·ĂŹng, v  trong §t li·n cuĂ«c sĂšng všn cĂą nhi·u b§t cÂȘp cĂĄn ang xÂŁy ra,: : : , t§t cÂŁ nhĂșng i·u Ăą ·u l  cÂĄc b§t ÂŻng thĂčc trong ph€m trĂČ Â°c thĂČ cĂ”a tĂžng lŸnh vĂŒc. Trong toÂĄn hĂ„c cĂŽng vÂȘy nĂąi žn b§t ÂŻng thĂčc l  chĂłng ta nĂąi žn mĂ«t lĂźp b i toÂĄn khĂą m  ©n chĂča b¶n trong cĂą nhi·u lĂ­i giÂŁi ”p l€ kÂŒ l m say ­m bižt bao nhi¶u ngĂ·Ă­i. Trong thĂ­i €i cĂŠng ngh» thĂŠng tin vĂźi vi»c kžt nĂši internet b€n cĂą thÂș giao lĂ·u hĂ„c hĂ€i ֖c r§t nhi·u v· cÂĄc phĂ·ĂŹng phÂĄp l m b i b§t ÂŻng thĂčc, ho°c hĂ„c hĂ€i vĂźi nhi·u cuĂšn sÂĄch v· b§t ÂŻng thĂčc ang b y bÂĄn tr¶n thĂ  trĂ·Ă­ng nhĂ·ng Âș cĂą mĂ«t cuĂšn sÂĄch b§t ÂŻng thĂčc hay vĂźi sĂŒ hĂ«i tö tinh hoa kižn thĂčc cĂ”a nhi·u ngĂ·Ă­i thÂŒ i·u Ăą chÂœnh l  iÂșm m€nh cĂ”a cuĂšn sÂĄch b§t ÂŻng thĂčc m  cÂĄc b€n ang cŠm tr¶n tay. TuyÂșn TÂȘp B§t ÂŻng ThĂčc vĂźi khoÂŁng bĂšn tr«m b i toÂĄn b§t ÂŻng thĂčc chĂ„n lĂ„c ֖c gĂ»i tĂźi tĂž cÂĄc b€n trÂŽ, cÂĄc thŠy cĂŠ giÂĄo y¶u toÂĄn tr¶n mĂ„i mi·n cĂ”a tĂȘ quĂšc, Ă° Ăą bao gçm cÂĄc b i toÂĄn b§t ÂŻng thĂčc mĂźi sÂĄng t€o, cÂĄc b i toÂĄn b§t ÂŻng thĂčc khĂą, cÂĄc b i toÂĄn b§t ÂŻng thĂčc hay v  thĂł vĂ  m  cÂĄc b€n trÂŽ muĂšn chia sÂŽ vĂźi mĂ„i ngĂ·Ă­i. i·u Ăą t€o n¶n sĂŒ h§p dšn, tÂœnh cÂȘp nhÂȘt v  thĂ­i €i cĂ”a cuĂšn sÂĄch n y. B€n Ă„c hÂąy nh„m nhi vĂźi nhĂșng lĂ­i giÂŁi hay, nhĂșng ĂŸ tĂ·Ă°ng Ă«c ÂĄo, nhĂșng sÂĄng kižn l€ kÂŒ trong cÂĄch giÂŁi tĂžng b i toÂĄn Âș tĂž Ăą rĂłt kinh nghi»m hĂ„c tÂȘp cho mÂŒnh, giĂłp cho b€n th¶m y¶u, th¶m tin v o vi»c giÂŁi nhi·u b i toÂĄn b§t ÂŻng thĂčc. VĂźi tinh thŠn l m vi»c nghi¶m tĂłc, ham hĂ„c hĂ€i nhĂąm bi¶n tÂȘp xin ֖c gĂ»i lĂ­i cÂŁm ĂŹn s„u s­c tĂźi t§t cÂŁ cÂĄc b€n Âą tham gia gĂ»i b i v  giÂŁi b i, çng thĂ­i cĂŽng xin b y tĂ€ sĂŒ cÂŁm ĂŹn v  kÂœnh trĂ„ng tĂźi thŠy giÂĄo Ch„u NgĂ„c HĂČng - THPT Ninh HÂŁi - Ninh ThuÂȘn Âą nhi»t tÂŒnh cĂš všn kŸ thuÂȘt latex. NhĂąm bi¶n tÂȘp cĂŽng xin gĂ»i lĂ­i cÂŁm ĂŹn tĂźi ban quÂŁn trĂ  diÂčn  n http://forum.mathscope.org/index.php Âą cĂȘ vĂŽ, Ă«ng vi¶n anh em trong quÂĄ trÂŒnh l m vi»c Âș ng y hĂŠm nay chĂłng ta cĂą mĂ«t cuĂšn sÂĄch hay, cĂą giÂĄ trĂ  cao v· kižn thĂčc chuy¶n mĂŠn m  l€i ho n to n miÂčn phÂœ v· t i chÂœnh. TUYĆĄN TĆ P B‡T NG THÙC chÂœnh thĂčc ֖c phÂĄt h nh tr¶n cĂ«ng çng m€ng nhĂșng ngĂ·Ă­i y¶u toÂĄn, Âș tĂž Ăą thĂȘi mĂ«t luçng giĂą mĂźi em l€i nhi·u i·u mĂźi l€ cho hĂ„c sinh, l  t i li»u tham khÂŁo hĂșu Âœch cho giÂĄo vi¶n trong vi»c giÂŁng d€y v  hĂ„c tÂȘp b§t ÂŻng thĂčc. Do thĂ­i gian g§p rĂłt v  trÂŒnh Ă« cĂą h€n, dĂČ r§t cĂš g­ng song nhĂșng sai sĂąt l  khĂą trÂĄnh khĂ€i r§t mong nhÂȘn ֖c sĂŒ thĂŠng cÂŁm, chia sÂŽ, gĂąp ĂŸ cĂ”a cÂĄc b€n Âș nhĂąm bi¶n tÂȘp ho n thi»n cuĂšn sÂĄch tĂšt hĂŹn. MĂ„i ĂŸ kižn Ăąng gĂąp xin gĂ»i v· Ă a chÂż hoangquan9@gmail. Thay m°t nhĂąm bi¶n so€n, tĂŠi xin ch„n th nh cÂŁm ĂŹn! H  NĂ«i, ng y 10 thÂĄng 8 n«m 2011 €i di»n nhĂąm bi¶n so€n ChĂ” bi¶n Ho ng Minh Qu„n-Batigoal 4
  • 5. CÂĄc th nh vi¶n tham gia bi¶n so€n NĂ«i dung Ho ng Minh Qu„n - THPT NgĂ„c TÂŁo - H  NĂ«i. T«ng HÂŁi Tu„n - THPT NguyÂčn Ăčc CÂŁnh - TP. ThÂĄi BÂŒnh. L¶ Ăčc CÂŁnh - THPT Chuy¶n L¶ Hçng Phong-Nam Ă nh.  o ThÂĄi Hi»p - PTNK - HQG HCM. Ph€m Tu§n Huy - PTNK - HQG HCM. Ph€m Quang HĂ·ng - THPT Cao BÂĄ QuÂĄt - H  NĂ«i. Ph€m Tižn Kha - THPT Chuy¶n L¶ Hçng Phong - TP. HCM. NguyÂčn V«n KhÂĄnh - THPT Chuy¶n B­c Ninh - TP. B­c Ninh. NguyÂčn ThĂ  Nguy¶n Khoa - THCS NguyÂčn Tri PhĂ·ĂŹng - TP. Huž. M€c Ăčc TrÂœ - HÂŁi DĂ·ĂŹng. LATEX HĂ© trñ kŸ thuÂȘt Latex 1. Ch„u NgĂ„c HĂČng - THPT Ninh HÂŁi -Ninh ThuÂȘn. 2. CÂĄc th nh vi¶n trong nhĂąm bi¶n so€n. TrÂŒnh b y bÂŒa Ho ng Minh Qu„n - THPT NgĂ„c TÂŁo - H  NĂ«i. 5
  • 6. 1 CÂĄc b§t ÂŻng thĂčc kinh iÂșn 1.1 B§t ÂŻng thĂčc giĂșa trung bÂŒnh cĂ«ng v  trung bÂŒnh nh„n (AM-GM). Nžu a1; a2; : : : ; an l  cÂĄc sĂš thĂŒc khĂŠng „m, thÂŒ a1 + a2 + : : : + an n n p a1a2 : : : an: ÂŻng thĂčc xÂŁy ra khi v  chÂż khi a1 = a2 = : : : = an. 1.2 B§t ÂŻng thĂčc giĂșa trung bÂŒnh cĂ«ng v  trung bÂŒnh i·u ho  (AM-HM). Nžu a1; a2; : : : ; an l  cÂĄc sĂš thĂŒc dĂ·ĂŹng, thÂŒ a1 + a2 + : : : + an n n 1 a1 + 1 a2 + : : : + 1 an : ÂŻng thĂčc xÂŁy ra khi v  chÂż khi a1 = a2 = : : : = an. ThĂŒc ch§t „y l  mĂ«t h» quÂŁ trĂŒc tižp cĂ”a b§t ÂŻng thĂčc Cauchy - Schwarz. Hai trĂ·Ă­ng hñp thĂ·Ă­ng ֖c sĂ» döng nh§t cĂ”a b§t ÂŻng thĂčc n y l  khi n = 3 hay n = 4. VĂźi n = 3, ta cĂą a + b + c 3 3 1 a + 1 b + 1 c ; 1 a + 1 b + 1 c 9 a + b + c : VĂźi n = 4, ta cĂą a + b + c + d 4 4 1 a + 1 b + 1 c + 1 d ; 1 a + 1 b + 1 c + 1 d 16 a + b + c + d : 1.3 B§t ÂŻng thĂčc Cauchy - Schwarz. D€ng sĂŹ c§p cĂ”a nĂą ֖c phÂĄt biÂșu nhĂ· sau: Nžu a1; a2; : : : ; an v  b1; b2; : : : ; bn l  cÂĄc sĂš thĂŒc tuĂœ ĂŸ, thÂŒ (a1b1 + a2b2 + : : : + anbn)2 (a21 + a22 + : : : + a2 n)(b1 + b2 + : : : + b2 n): ÂŻng thĂčc xÂŁy ra khi v  chÂż khi a1 b1 = a2 b2 = : : : = an bn , trong Ăą ta sĂ» döng quy Ă·Ăźc: nžu mšu bÂŹng 0 thÂŒ tĂ» cĂŽng bÂŹng 0. Trong ÂĄnh giÂĄ tr¶n, chĂ„n ai = xi p yi ,bi = p yi vĂźi xi; yi 2 R; yi 0, ta thu ֖c b§t ÂŻng thĂčc Cauchy - Schwarz d€ng ph„n thĂčc: Nžu x1; x2; : : : ; xn l  cÂĄc sĂš thĂŒc v  y1; y2; : : : ; yn, l  cÂĄc sĂš thĂŒc dĂ·ĂŹng, thÂŒ x21 y1 + x22 y2 + : : : + x2 n yn (x1 + x2 + : : : + xn)2 y1 + y2 + : : : + yn : ÂŻng thĂčc xÂŁy ra khi v  chÂż khi x1 y1 = x2 y2 = : : : = xn yn . 6
  • 7. 1.4 B§t ÂŻng thĂčc Holder. Cho xij (i = 1; 2; : : : ;m; j = 1; 2; : : : ; n) l  cÂĄc sĂš thĂŒc khĂŠng „m. Khi Ăą ta cĂą Ym i=1 Xn j=1 xij ! 1 m Xn j=1 Ym i=1 x 1 m ij ! : TĂȘng quÂĄt hĂŹn, nžu p1; p2; : : : ; pn l  cÂĄc sĂš thĂŒc dĂ·ĂŹng thoÂŁ mÂąn p1 + p2 + : : : + pn = 1, thÂŒ Ym i=1 Xn j=1 xij !pi Xn j=1 Ym i=1 xpi ij ! : 1.5 B§t ÂŻng thĂčc Chebyshev. Cho hai dÂąy sĂš thĂŒc a1 a2 : : : an v  b1; b2; : : : ; bn. Khi Ăą 1. Nžu b1 b2 : : : bn thÂŒ n Xn i=1 aibi Xn i=1 ai ! Xn i=1 bi ! ; 2. Nžu b1 b2 : : : bn thÂŒ n Xn i=1 aibi Xn i=1 ai ! Xn i=1 bi ! . 1.6 B§t ÂŻng thĂčc Minkowski. Cho hai dÂąy sĂš dĂ·ĂŹng a1; a2; : : : ; an v  b1; b2; : : : ; bn. VĂźi mĂ„i r 1, ta cĂą Xn i=1 (ai + bi)r #1 r Xn i=1 ari !1 r + Xn i=1 bri !1 r : vuut TrĂ·Ă­ng hñp r = 2 l  trĂ·Ă­ng hñp thĂ·Ă­ng ֖c sĂ» döng nh§t cĂ”a b§t ÂŻng thĂčc Minkowski. Khi Ăą ta cĂą Xn i=1 (ai + bi)2 vuut Xn i=1 a2i + vuut Xn i=1 b2i : 1.7 B§t ÂŻng thĂčc Schur. Cho cÂĄc sĂš thĂŒc khĂŠng „m a; b; c. Khi Ăą vĂźi mĂ„i sĂš thĂŒc dĂ·ĂŹng r, ta cĂą ar(a b)(a c) + br(b a)(b c) + cr(c a)(c b) 0: ÂŻng thĂčc xÂŁy ra khi v  chÂż khi a = b = c, ho°c a = 0 v  b = c, ho°c cÂĄc hoÂĄn vĂ  tĂ·ĂŹng Ăčng. Hai trĂ·Ă­ng hñp thĂ·Ă­ng ֖c sĂ» döng nh§t cĂ”a b§t ÂŻng thĂčc Schur l  r = 1 v  r = 2. VĂźi r = 1, ta cĂą b§t ÂŻng thĂčc Schur bÂȘc ba a3 + b3 + c3 + 3abc ab(a + b) + bc(b + c) + ca(c + a); (a + b + c)3 + 9abc 4(a + b + c)(ab + bc + ca); (b c)2(b + c a) + (c a)2(c + a b) + (a b)2(a + b c) 0; 7
  • 8. a2 + b2 + c2 + 9abc a + b + c 2(ab + bc + ca); a b + c + b c + a + c a + b + 4abc (a + b)(b + c)(c + a) 2: VĂźi r = 2, ta thu ֖c b§t ÂŻng thĂčc Schur bÂȘc bĂšn a4 + b4 + c4 + abc(a + b + c) ab(a2 + b2) + bc(b2 + c2) + ca(c2 + a2): 1.8 B§t ÂŻng thĂčc Vornicu - Schur. VĂźi mĂ„i sĂš thĂŒc a; b; c v  x; y; z 0, b§t ÂŻng thĂčc x(a b)(a b) + y(b c)(b a) + z(c a)(c b) 0 Ăłng nžu mĂ«t trong cÂĄc i·u ki»n sau ֖c thoÂŁ mÂąn 1. a b c v  x y; 2. a b c v  z y; 3. a b c v  x + z y; 4. a b c 0 v  ax by; 5. a b c 0 v  cz by; 6. a b c 0 v  ax + cz by; 7. x; y; z l  Ă« d i ba c€nh cĂ”a mĂ«t tam giÂĄc; 8. x; y; z l  bÂŒnh phĂ·ĂŹng Ă« d i ba c€nh cĂ”a mĂ«t tam giÂĄc; 9. ax; by; cz l  Ă« d i ba c€nh cĂ”a mĂ«t tam giÂĄc; 10. ax; by; cz l  bÂŒnh phĂ·ĂŹng Ă« d i ba c€nh cĂ”a mĂ«t tam giÂĄc; 11. Tçn t€i mĂ«t h m lçi t : I ! R+, trong Ăą I l  tÂȘp xÂĄc Ă nh cĂ”a a; b; c, sao cho x = t(a); y = t(b); z = t(c). 1.9 B§t ÂŻng thĂčc Bernoulli. Nžu 1 ho°c 0 thÂŒ (1 + x) 1 + x; 8x 1. Nžu 0 1 thÂŒ (1 + x) 1 + x; 8x 1. 8
  • 9. 1.10 Ba ti¶u chu©n SOS thĂ·Ă­ng g°p. GiÂŁ sĂ» a b c v  cĂą: Sa(b c)2 + Sb(c a)2 + Sc(a b)2 0(Sa; Sb; Sc l  cÂĄc h m chĂča bižn a; b; c). Khi Ăą b§t ÂŻng thĂčc Ăłng nžu thĂ€a mÂąn mĂ«t trong cÂĄc ti¶u chu©n. 1.Sb 0; Sb + Sc 0; Sb + Sa 0. 2.VĂźi a; b; c 0 thĂ€a mÂąn Sb 0; Sc 0; a2Sb + b2Sa 0. 3.Sb 0; Sc 0; Sa(b c) + Sb(a c) 0 2 MĂ«t sĂš ÂĄnh giÂĄ quen thuĂ«c 1 VĂźi mĂ„i sĂš thĂŒc a; b, ta luĂŠn cĂą 2(a2 + b2) (a + b)2 ChĂčng minh. Âș ĂŸ rÂŹng 2(a2 + b2) (a + b)2 = (a b)2 0; do Ăą ta cĂą i·u phÂŁi chĂčng minh. ÂŻng thĂčc xÂŁy ra khi v  chÂż khi a = b. 2 2 VĂźi mĂ„i sĂš thĂŒc a; b; c, ta luĂŠn cĂą a2 + b2 + c2 ab + bc + ca ChĂčng minh. Âș ĂŸ rÂŹng a2 + b2 + c2 (ab + bc + ca) = 1 2 [(a b)2 + (b c)2 + (c a)2] 0; do vÂȘy ta cĂą i·u phÂŁi chĂčng minh. ÂŻng thĂčc xÂŁy ra khi v  chÂż khi a = b = c. 2 LĂ·u ĂŸ. TĂž ÂĄnh giÂĄ n y ta suy ra (a + b + c)2 3(ab + bc + ca); v  3(a2 + b2 + c2) (a + b + c)2: 3 VĂźi mĂ„i sĂš thĂŒc dĂ·ĂŹng a; b; c, ta luĂŠn cĂą 1 a + 1 b + 1 c 9 a + b + c ChĂčng minh. „y l  mĂ«t kžt quÂŁ Âą ֖c · cÂȘp Ă° tr¶n. LĂ­i giÂŁi cĂą thÂș sĂ» döng b§t ÂŻng thĂčc AM-HM ho°c Cauchy - Schwarz. ÂŻng thĂčc xÂŁy ra khi v  chÂż khi a = b = c. 2 9
  • 10. 3 TuyÂșn tÂȘp b§t ÂŻng thĂčc 3.1 B i 1.1 žn b i 1.40 1.1 Cho x; y; z l  cÂĄc sĂš thĂŒc dĂ·ĂŹng thĂ€a mÂąn x + y + z = 1. ChĂčng minh rÂŹng: 8x + 8y + 8z 4x+1 + 4y+1 + 4z+1 LĂ­i giÂŁi. °t a = 2x; b = 2y; c = 2z. Khi Ăą i·u ki»n Âą cho ֖c vižt l€i th nh a; b; c 0; abc = 2x+y+z = 64; v  ta cŠn chĂčng minh a3 + b3 + c3 4(a2 + b2 + c2): Âș ĂŸ rÂŹng ta cĂą ÂŻng thĂčc a3 + 32 6a2 = (a 4)2(a + 2); tĂž Ăą sĂ» döng giÂŁ thižt a 0 ta suy ra a3 + 32 6a2. Thižt lÂȘp cÂĄc b§t ÂŻng thĂčc tĂ·ĂŹng tĂŒ cho b v  c v  cĂ«ng vž theo vž cÂĄc b§t ÂŻng thĂčc thu ֖c, ta cĂą a3 + b3 + c3 + 96 6(a2 + b2 + c2): NhĂ· vÂȘy Âș kžt thĂłc chĂčng minh ta cŠn chÂż ra rÂŹng 6(a2 + b2 + c2) 4(a2 + b2 + c2) + 96; hay 2(a2 + b2 + c2) 96. Tuy nhi¶n b§t ÂŻng thĂčc n y Ăłng theo b§t ÂŻng thĂčc AM-GM cho ba sĂš: 2(a2 + b2 + c2) 2:3 3 p a2b2c2 = 6 3 p 4096 = 96: NhĂ· vÂȘy phÂČp chĂčng minh žn „y ho n t§t.2 1.2 Cho a; b; c l  cÂĄc sĂš thĂŒc thoÂŁ mÂąn a 4; b 5; c 6 v  a2 + b2 + c2 = 90. TÂŒm giÂĄ trĂ  nhĂ€ nh§t cĂ”a biÂșu thĂčc: P = a + b + c LĂ­i giÂŁi. °t a = m+ 4; b = n + 5; c = p + 6, khi Ăą m; n; p 0 v  tĂž giÂŁ thižt a2 + b2 + c2 = 90 ta suy ra m2 + n2 + p2 + 8m + 10n + 12p = 13: Âș ĂŸ rÂŹng ta cĂą ÂŻng thĂčc sau (m+ n + p)2 + 12(m+ n + p) = (m2 + n2 + p2 + 8m+ 10n + 12p) + 2(mn + np + pm + 2m+ n): žn „y ta sĂ» döng cÂĄc giÂŁ thižt Âą cho Âș cĂą (m + n + p)2 + 12(m + n + p) 13; tĂž Ăą ta suy ra m+ n + p 1. Thay m = a 4; n = b 5; p = c 6 ta suy ra a + b + c 10 hay P 16. 10
  • 11. Cuçi cĂČng, vĂźi a = 4; b = 5; c = 7 (thoÂŁ mÂąn cÂĄc i·u ki»n Âą cho) ta cĂą P = 16 n¶n ta kžt luÂȘn 16 l  giÂĄ trĂ  nhĂ€ nh§t cĂ”a biÂșu thĂčc P. PhÂČp chĂčng minh ho n t§t. 2 1.3 Cho x; y; z l  cÂĄc sĂš thĂŒc thoÂŁ mÂąn xy + yz + 3zx = 1. TÂŒm giÂĄ trĂ  nhĂ€ nh§t cĂ”a biÂșu thĂčc: P = x2 + y2 + z2 LĂ­i giÂŁi. °t a = p 17 4 9 + 3 v  b = 3 + p 17 4 , khi Ăą a = 3b v  a+1 = 2b2 = c = p 17 4 13 + 3 . p döng b§t ÂŻng thĂčc AM-GM ta thu ֖c cÂĄc b§t ÂŻng thĂčc sau x2 + b2y2 2bxy; by2 + z2 2byz; a(z2 + x2) 2azx: žn „y ta cĂ«ng vž theo vž cÂĄc b§t ÂŻng thĂčc thu ֖c Âș cĂą (a + 1)(x2 + z2) + 2b2y2 2b(xy + yz) + 2azx; hay c(x2 + y2 + z2) 2b(xy + yz + 3zx). TĂž Ăą ta thay cÂĄc giÂĄ trĂ  cĂ”a xy + yz + 3zx, b v  c Âș ֖c P = x2 + y2 + z2 p 17 3 2 : CuĂši cĂČng, vĂźi x = z = 1 4 p 17 v  y = r p 17 51 13 34 (thoÂŁ mÂąn giÂŁ thižt) thÂŒ P = p 17 3 2 n¶n ta kžt luÂȘn p 17 3 2 l  giÂĄ trĂ  nhĂ€ nh§t cĂ”a biÂșu thĂčc P. PhÂČp chĂčng minh ho n t§t.2 1.4 Cho a; b; c l  cÂĄc sĂš thĂŒc dĂ·ĂŹng thoÂŁ mÂąn a + b + c = 1. ChĂčng minh rÂŹng: a7 + b7 a5 + b5 + b7 + c7 b5 + c5 + c7 + a7 c5 + a5 1 3 LĂ­i giÂŁi. TrĂ·Ăźc hžt ta cĂą ÂŻng thĂčc sau 2(a7 + b7) (a2 + b2)(a5 + b5) = (a b)2(a + b)(a4 + a3b + a2b2 + ab3 + b4); do vÂȘy tĂž giÂŁ thižt a; b 0 ta suy ra a7 + b7 a5 + b5 a2 + b2 2 : Ho n to n tĂ·ĂŹng tĂŒ ta cĂŽng cĂą b7 + c7 b5 + c5 b2 + c2 2 v  c7 + a7 c5 + a5 c2 + a2 2 . žn „y ta cĂ«ng vž theo vž ba b§t ÂŻng thĂčc thu ֖c Âș cĂą a7 + b7 a5 + b5 + b7 + c7 b5 + c5 + c7 + a7 c5 + a5 a2 + b2 + c2: 11
  • 12. NhĂ· vÂȘy Âș kžt thĂłc chĂčng minh ta cŠn chÂż ra rÂŹng a2 + b2 + c2 1 3 : Tuy nhi¶n b§t ÂŻng thĂčc tr¶n Ăłng do a2 + b2 + c2 1 3 = a2 + b2 + c2 (a + b + c)2 3 = (a b)2 + (b c)2 + (c a)2 3 0: NhĂ· vÂȘy phÂČp chĂčng minh žn „y ho n t§t.2 1.5 Cho a; b; c l  cÂĄc sĂš thĂŒc dĂ·ĂŹng. ChĂčng minh rÂŹng: b2c a3(b + c) + c2a b3(c + a) + a2b c3(a + b) 1 2 (a + b + c) LĂ­i giÂŁi. Ta ÂĄp döng AM-GM cho ba sĂš nhĂ· sau: b2c a3(b + c) + b + c 4bc + 1 2b s 3 3 b2c a3(b + c) : (b + c) 4bc : 1 2b = 3 2a ; tĂž Ăą ta suy ra b2c a3(b + c) 3 2a 3 4b 1 4c : Thižt lÂȘp hai b§t ÂŻng thĂčc tĂ·ĂŹng tĂŒ v  cĂ«ng l€i, ta suy ra b2c a3(b + c) + c2a b3(c + a) + a2b c3(a + b) 3 2 3 4 1 4 (a + b + c) = 1 2 (a + b + c): PhÂČp chĂčng minh ho n t§t.2 1.6 Cho a; b; c l  cÂĄc sĂš thĂŒc khĂŠng „m. ChĂčng minh rÂŹng: p 3(a b)(b c)(c a) (a + b + c)3 6 LĂ­i giÂŁi. B§t ÂŻng thĂčc ban Šu mang tÂœnh hoÂĄn vĂ  giĂșa cÂĄc bižn n¶n khĂŠng m§t tÂœnh tĂȘng quÂĄt, ta giÂŁ sĂ» a = max fa; b; cg. VĂźi a b c thÂŒ vž phÂŁi l  biÂșu thĂčc khĂŠng dĂ·ĂŹng, trong khi vž trÂĄi l  biÂșu thĂčc khĂŠng „m n¶n b§t ÂŻng thĂčc cŠn chĂčng minh hiÂșn nhi¶n Ăłng. Do vÂȘy ta xÂČt trĂ·Ă­ng hñp a c b. Khi Ăą bÂŒnh phĂ·ĂŹng hai vž ta thu ֖c b§t ÂŻng thĂčc tĂ·ĂŹng Ă·ĂŹng sau: (a + b + c)6 108[(a b)(b c)(c a)]2: Âș ĂŸ rÂŹng cÂĄc bižn khĂŠng „m, v  vĂźi vi»c s­p thĂč tĂŒ nhĂ· tr¶n thÂŒ [(a b)(b c)(c a)]2 = [(a b)(c b)(a c)]2 (a c)2a2c2: žn „y ta ÂĄp döng b§t ÂŻng thĂčc AM-GM Âș cĂą 4(a c)2a2c2 = (a c)2:2ac:2ac [(a c)2 + 2ac + 2ac]3 27 = (a + c)6 27 ; tĂž Ăą ta suy ra [(a b)(b c)(c a)]2 (a + c)6 108 ; 12
  • 13. v  nhĂ· vÂȘy ta Âą chĂčng minh ֖c b§t ÂŻng thĂčc ban Šu vÂŒ (a + b + c)6 (a + c)6 108[(a b)(b c)(c a)]2: PhÂČp chĂčng minh ho n t§t.2 1.7 Cho a; b; c l  cÂĄc sĂš thĂŒc dĂ·ĂŹng thoÂŁ mÂąn a + b + c = 1 a + 1 b + 1 c . ChĂčng minh rÂŹng: 2(a + b + c) p a2 + 3 + p b2 + 3 + p c2 + 3 LĂ­i giÂŁi. DÂč th§y b§t ÂŻng thĂčc cŠn chĂčng minh tĂ·ĂŹng Ă·ĂŹng vĂźi mĂ©i b§t ÂŻng thĂčc trong dÂąy sau (2a p a2 + 3) + (2b p b2 + 3) + (2c p c2 + 3) 0; a2 1 2a + p a2 + 3 + b2 1 2b + p b2 + 3 + c2 1 2c + p c2 + 3 0; a2 1 a 2 + r 1 + 3 a2 + b2 1 b 2 + r 1 + 3 b2 + c2 1 c 2 + r 1 + 3 c2 0: CÂĄc b§t ÂŻng thĂčc tr¶n ·u mang tÂœnh Ăši xĂčng giĂșa cÂĄc bižn n¶n khĂŠng m§t tÂœnh tĂȘng quÂĄt ta ho n to n cĂą thÂș giÂŁ sĂ» a b c. Khi Ăą khĂŠng khĂą Âș ta suy ra a2 1 a b2 1 b c2 1 c v  1 2 + q 1 + 3 a2 1 2 + q 1 + 3 b2 1 2 + q 1 + 3 b2 : NhĂ· vÂȘy theo b§t ÂŻng thĂčc Chebyshev ta ֖c a2 1 a 2 + q 1 + 3 a2 + b2 1 b 2 + r 1 + 3 b2 + c21 c 2 + r 1 + 3 c2 1 3 Xa2 1 a 0 BB@ X 1 2 + r 1 + 3 a2 1 CCA NhĂ·ng theo giÂŁ thižt ta l€i cĂą Xa2 1 a = (a + b + c) 1 a + 1 b + 1 c = 0 n¶n ta suy ra a2 1 a 2 + q 1 + 3 a2 + b2 1 b 2 + r 1 + 3 b2 + c2 1 c 2 + r 1 + 3 c2 0, v  vÂŒ vÂȘy b§t ÂŻng thĂčc Âą cho cĂŽng Ăłng. PhÂČp chĂčng minh ho n t§t.2 1.8 Cho a; b; c l  cÂĄc sĂš thĂŒc dĂ·ĂŹng thoÂŁ mÂąn a + b + c = 3. ChĂčng minh rÂŹng: ab p c2 + 3 + bc p a2 + 3 + ca p b2 + 3 3 2 13
  • 14. LĂ­i giÂŁi. TrĂ·Ăźc hžt Âș ĂŸ rÂŹng ab + bc + ca (a + b + c)2 3 = (a b)2 + (b c)2 + (c a)2 6 0; do Ăą tĂž giÂŁ thižt ta suy ra ab + bc + ca 3. NhĂ· vÂȘy ab p c2 + 3 ab p c2 + ab + bc + ca = ab p (c + a)(b + c) : žn „y ta ÂĄp döng b§t ÂŻng thĂčc AM-GM Âș cĂą ab p c2 + 3 1 2 ab c + a + ab b + c : Thižt lÂȘp hai b§t ÂŻng thĂčc tĂ·ĂŹng tĂŒ v  cĂ«ng l€i, ta suy ra dÂąy cÂĄc ÂĄnh giÂĄ sau ab p c2 + 3 + bc p a2 + 3 + ca p b2 + 3 1 2 ab c + a + bc c + a + bc a + b + ca a + b + ca b + c + ab b + c ; ab p c2 + 3 + bc p a2 + 3 + ca p b2 + 3 a + b + c 2 ; tĂž Ăą vĂźi lĂ·u ĂŸ a + b + c = 3 ta suy ra b§t ÂŻng thĂčc Âą cho l  Ăłng. PhÂČp chĂčng minh ho n t§t.2 1.9 Cho a; b; c l  cÂĄc sĂš thĂŒc dĂ·ĂŹng thay ĂȘi b§t kÂŒ. ChĂčng minh rÂŹng: b + c a + c + a b + a + b c 2 4(ab + bc + ca) 1 a2 + 1 b2 + 1 c2 LĂ­i giÂŁi 1. DÂč th§y rÂŹng b§t ÂŻng thĂčc ban Šu tĂ·ĂŹng Ă·ĂŹng vĂźi mĂ©i b§t ÂŻng thĂčc trong dÂąy sau [ab(a + b) + bc(b + c) + ca(c + a)]2 4(a + b + c)(a2b2 + b2c2 + c2a2) X a2b2(a + b)2 + 2abc[ X a(a + b)(a + c)] 4 nX a3b3 + abc[ X o ab(a + b)] Tuy nhi¶n Âș ĂŸ rÂŹng X X a2b2(a + b)2 4( a3b3) = X a2b2(a b)2 0 v  2abc[ X a(a + b)(a + c)] 4 n abc[ X o = 2abc[a3 + b3 + c3 + 3abc ab(a + b)] X ab(a + b)] 0; do Ăą b§t ÂŻng thĂčc ban Šu l  Ăłng. PhÂČp chĂčng minh žn „y ho n t§t.2 LĂ­i giÂŁi 2. B§t ÂŻng thĂčc ban Šu mang tÂœnh hoÂĄn vĂ  giĂșa cÂĄc bižn, n¶n khĂŠng m§t tÂœnh tĂȘng quÂĄt, ta giÂŁ sĂ» b = max fa; b; cg. Ta ÂĄp döng b§t ÂŻng thĂčc AM-GM nhĂ· sau b + c a + c + a b + a + b c 2 = a b + b a + a c + b c + c b + c a 2 4 a b + b a + a c b c + c b + c a : 14
  • 15. NhĂ· vÂȘy Âș kžt thĂłc chĂčng minh, ta cŠn chÂż ra rÂŹng a b + b a + a c b c + c b + c a (ab + bc + ca) 1 a2 + 1 b2 + 1 c2 : Tuy nhi¶n bÂŹng phÂČp bižn ĂȘi tĂ·ĂŹng Ă·ĂŹng ta ֖c (b a)(b c) ca 0; l  mĂ«t ÂĄnh giÂĄ Ăłng do ta Âą giÂŁ sĂ» b = max fa; b; cg. PhÂČp chĂčng minh žn „y ho n t§t.2 LĂ­i giÂŁi 3. B§t ÂŻng thĂčc ban Šu mang tÂœnh Ăši xĂčng giĂșa cÂĄc bižn n¶n khĂŠng m§t tÂœnh tĂȘng quÂĄt, ta giÂŁ sĂ» b nÂŹm giĂșa a v  c. Ta ÂĄp döng b§t ÂŻng thĂčc AM-GM nhĂ· sau: 4(ab + bc + ca) 1 a2 + 1 b2 + 1 c2 ab + bc + ca ca + ca 1 a2 + 1 b2 + 1 c2 2 : NhĂ· vÂȘy Âș kžt thĂłc chĂčng minh, ta cŠn chÂż ra rÂŹng b + c a + c + a b + a + b c ab + bc + ca ca + ca 1 a2 + 1 b2 + 1 c2 : ThĂŒc hi»n phÂČp bižn ĂȘi tĂ·ĂŹng Ă·ĂŹng ta ֖c b§t ÂŻng thĂčc (a b)(b c) b2 0; tuy nhi¶n „y l€i l  mĂ«t ÂĄnh giÂĄ Ăłng do ta Âą giÂŁ sĂ» b nÂŹm giĂșa a v  c. PhÂČp chĂčng minh žn „y ho n t§t.2 NhÂȘn xÂČt. LĂ­i giÂŁi Šu ti¶n khĂŠng mang nhi·u ĂŸ nghŸa l­m, vÂŒ nĂą ĂŹn thuŠn chÂż l  bižn ĂȘi tĂ·ĂŹng Ă·ĂŹng k±m theo mĂ«t chĂłt tinh ĂŸ trong sĂ» döng cÂĄc ÂĄnh giÂĄ quen thuĂ«c v  cĂŹ bÂŁn. Ð „y ta b n th¶m v· hai lĂ­i giÂŁi bÂŹng AM-GM. Ta nh ÂȘn th§y rÂŹng phÂĄt biÂșu cĂ”a b i toÂĄn cĂą d€ng ChĂčng minh rÂŹng A2 4BC (Ă° „y b + c A = a + c + a b + a + b c 2 , B = ab + bc + ca v  C = 1 a2 + 1 b2 + 1 c2 . NhÂȘn xÂČt n y khÂĄ °c bi»t, nĂą giĂłp ta li¶n tĂ·Ă°ng žn mĂ«t ÂĄnh giÂĄ quen thuĂ«c sau bÂŹng AM-GM: (x + y)2 4xy 8x; y 0: Do vÂȘy, mĂ«t cÂĄch tĂŒ nhi¶n ta nghŸ ra hai hĂ·Ăźng Âș giÂŁi quyžt b i toÂĄn tr¶n bÂŹng AM-GM: 1. BiÂșu diÂčn A = X +Y , vĂźi X v  Y l  hai €i l֖ng thÂœch hñp, sau Ăą ÂĄp döng b§t ÂŻng thĂčc AM-GM Âș cĂą A2 4XY , tĂž Ăą i chĂčng minh XY BC; ho°c 15
  • 16. 2. BiÂșu diÂčn BC = B D :CD, vĂźi D l  mĂ«t €i l֖ng thÂœch hñp, sau Ăą ÂĄp döng b§t ÂŻng thĂčc AM-GM Âș cĂą 4BC B D + CD 2 , tĂž Ăą i chĂčng minh A B D + CD. Ð „y ta hiÂșu cöm tĂž thÂœch hñp l  nhĂ· thž n o? LĂ·u ĂŸ rÂŹng mĂ«t trong nhĂșng i·u cŠn Âș ĂŸ trong mĂ„i chĂčng minh b§t ÂŻng thĂčc l  cŠn phÂŁi ĂŹn giÂŁn hoÂĄ b§t ÂŻng thĂčc cŠn chĂčng minh. Ta cĂą thÂș tÂŒm cÂĄch giÂŁm bÂȘc, chu©n hoÂĄ i·u ki»n, : : :, nhĂ·ng tĂŒu chung l€i, ta luĂŠn muĂšn b§t ÂŻng thĂčc cŠn chĂčng minh trĂ° n¶n ĂŹn giÂŁn nh§t cĂą thÂș, Âș tĂž Ăą ÂĄp döng nh” nh ng cÂĄc ÂĄnh giÂĄ quen thuĂ«c ho°c bižn ĂȘi tĂ·ĂŹng Ă·ĂŹng. Ð „y ta tÂŒm cÂĄch thu gĂ„n ÂĄnh giÂĄ sau cĂČng theo kiÂșu tri»t ti¶u mĂ«t l֖ng ÂĄng kÂș cÂĄc phŠn tĂ» chung, tĂčc l  Ă° ÂĄnh giÂĄ XY BC ho°c A B D + CD, cÂĄc €i l֖ng X; Y;D ֖c chĂ„n sao cho Ă° hai vž cĂ”a b§t ÂŻng thĂčc cĂą nhi·u phŠn tĂ» chung Âș ta rĂłt gĂ„n. Cö thÂș: HĂ·Ăźng 1. TrĂ·Ăźc ti¶n ta vižt l€i A v  khai triÂșn tÂœch BC nhĂ· sau: A = b a + c a + c b + a b + a c + b c = X + Y; BC = a c + c b + b a + a b + b c + c a + ca b2 + ab c2 + bc a2 : Âș ĂŸ rÂŹng trong BC cĂą phŠn tĂ» ca b2 , n¶n ta cŠn cĂą a b v  c b Ă° X v  Y tĂ·ĂŹng Ăčng: X = a b + : : : ; Y = c b + : : : M°t khÂĄc, trong BC cĂą phŠn tĂ» a b , m  Ă° Y Âą cĂą c b n¶n ta cŠn phŠn tĂ» a c Ă° trong X: X = a b + a c + : : : ; Y = c b + : : : Tižp töc, trong BC cĂą phŠn tĂ» ab c2 , n¶n ta cŠn cĂą a c v  b c Ă° X v  Y tĂ·ĂŹng Ăčng: X = a b + a c + : : : ; Y = c b + b c + : : : Tižp töc nhĂ· vÂȘy ta sÂł tÂŒm ֖c hai €i l֖ng X; Y chÂŻng h€n nhĂ· sau: X = a b + b a + a c ; Y = b c + c b + c a ; v  ta cĂą ֖c lĂ­i giÂŁi thĂč hai. CŠn lĂ·u ĂŸ rÂŹng „y khĂŠng phÂŁi l  cÂĄch chĂ„n duy nh§t. HĂ·Ăźng 2. XÂČt hi»u sau A B D CD = b + c a + c + a b + a + b c ab + bc + ca D D 1 a2 + 1 b2 + 1 c2 : Âș ĂŸ rÂŹng trong hi»u tr¶n thÂŒ h» sĂš cĂ”a bižn b bÂŹng 1 c + 1 a c + a D ; nhĂ· vÂȘy Âș tÂŒm cÂĄch thu gĂ„n b§t ÂŻng thĂčc, t€i sao ta khĂŠng cho h» sĂš cĂ”a bižn b bÂŹng khĂŠng? Cö thÂș, nžu chĂ„n D = ca thÂŒ 16
  • 17. A B D CD = b + c a + c + a b + a + b c ab + bc + ca ca ca 1 a2 + 1 b2 + 1 c2 = (a b)(b c) b2 ; v  nhĂ· vÂȘy ta Âą cĂą lĂ­i giÂŁi thĂč ba. 1.10 Cho a; b; c l  cÂĄc sĂš thĂŒc dĂ·ĂŹng thoÂŁ mÂąn a + b + c = 1. TÂŒm giÂĄ trĂ  lĂźn nh§t cĂ”a biÂșu thĂčc: P = ab + bc + ca + 5 2 p ab + (b + c) [(a + b) p bc + (c + a) p ca] LĂ­i giÂŁi. TrĂ·Ăźc hžt ta ÂĄp döng b§t ÂŻng thĂčc AM-GM nhĂ· sau: 2(a + b)2 + 2ab = (a + b)2 2 + (a + b)2 2 + (a + b)2 2 + (a + b)2 2 r + 2ab 5 5 ab(a + b)8 8 v  (a + b)3 (2 p ab)3 = 8( p ab)3; tĂž Ăą kžt hñp hai b§t ÂŻng thĂčc n y Âș cĂą 2(a + b)2 + 2ab 5(a + b) p ab: Thižt lÂȘp hai b§t ÂŻng thĂčc tĂ·ĂŹng tĂŒ v  cĂ«ng l€i, ta suy ra p ab + (b + c) 5[(a + b) p bc + (c + a) p ca] 4(a2 + b2 + c2) + 6(ab + bc + ca) žn „y ta cĂ«ng th¶m 2(ab + bc + ca) v o mĂ©i vž Âș cĂą p ab + (b + c) 2(ab + bc + ca) + 5[(a + b) p bc + (c + a) p ca] 4(a + b + c)2; tĂž Ăą ta suy ra P 2(a + b + c)2 = 2. CuĂši cĂČng, vĂźi a = b = c = 1 3 (thoÂŁ mÂąn i·u ki»n) thÂŒ P = 2 n¶n ta suy ra 2 l  giÂĄ trĂ  lĂźn nh§t cĂ”a biÂșu thĂčc P. PhÂČp chĂčng minh ho n t§t.2 1.11 Cho a; b; c l  cÂĄc sĂš thĂŒc dĂ·ĂŹng thoÂŁ mÂąn 1 a + 1 b + 1 c 16(a+b+c). ChĂčng minh rÂŹng: 1 (a + b + 2 p a + c)3 + 1 (b + c + 2 p b + a)3 + 1 (c + a + 2 p c + b)3 8 9 LĂ­i giÂŁi. TrĂ·Ăźc hžt ta ÂĄp döng b§t ÂŻng thĂčc AM-GM nhĂ· sau: a + b + r a + c 2 + r a + c 2 r 3 3 (a + b)(a + c) 2 ; tĂž Ăą ta suy ra 1 (a + b + 2 p a + c)3 2 27(a + b)(a + c) : 17
  • 18. CĂ«ng vž theo vž b§t ÂŻng thĂčc n y vĂźi hai b§t ÂŻng thĂčc tĂ·ĂŹng tĂŒ cho ta 1 (a + b + 2 p a + c)3 + 1 (b + c + 2 p b + a)3 + 1 (c + a + 2 p c + b)3 4(a + b + c) 27(a + b)(b + c)(c + a) : HĂŹn nĂșa, theo mĂ«t kžt quÂŁ quen thuĂ«c, ta l€i cĂą (a + b)(b + c)(c + a) 8 9 (a + b + c)(ab + bc + ca); do vÂȘy 1 (a + b + 2 p a + c)3 + 1 (b + c + 2 p b + a)3 + 1 (c + a + 2 p c + b)3 1 6(ab + bc + ca) :() žn „y ta sĂ» döng giÂŁ thižt v  ÂĄnh giÂĄ cĂŹ bÂŁn (ab + bc + ca)2 3abc(a + b + c) Âș cĂą 16(a + b + c) 1 a + 1 b + 1 c 3(a + b + c) ab + bc + ca ; tĂž Ăą suy ra ab + bc + ca 3 16 . Kžt hñp vĂźi () ta suy ra 1 (a + b + 2 p a + c)3 + 1 (b + c + 2 p b + a)3 + 1 (c + a + 2 p c + b)3 8 9 : PhÂČp chĂčng minh žn „y ho n t§t.2 NhÂȘn xÂČt. 1. CĂą thÂș th§y ÂĄnh giÂĄ ban Šu a+b+ r a + c 2 + r a + c 2 r 3 3 (a + b)(a + c) 2 chÂœnh l  iÂșm m§u chĂšt Âș giÂŁi quyžt b i toÂĄn. ThĂŒc ra ÂĄnh giÂĄ n y khĂŠng khĂą nghŸ tĂźi vÂŒ · b i Âą ngŠm gñi ĂŸ cho chĂłng ta phÂŁi ÂĄp döng b§t ÂŻng thĂčc AM-GM cho ba sĂš. 2. Sau khi ÂĄnh giÂĄ bÂŹng AM-GM, ta cĂą thÂș sĂ» döng luĂŠn giÂŁ thižt Âș Ă·a v· b§t ÂŻng thĂčc thuŠn nh§t sau: (a + b + c) (a + b)(b + c)(c + a) 3(ab + bc + ca) 8abc(a + b + c) : B§t ÂŻng thĂčc n y cĂą thÂș ֖c chĂčng minh bÂŹng nhi·u cÂĄch khÂĄc nhau. 1.12 Cho a; b; c l  cÂĄc sĂš thĂŒc dĂ·ĂŹng thoÂŁ mÂąn a + b + c = 1 a + 1 b + 1 c . ChĂčng minh rÂŹng: 5(a + b + c) 7 + 8abc LĂ­i giÂŁi. TrĂ·Ăźc hžt tĂž giÂŁ thižt ta cĂą a + b + c = 1 a + 1 b + 1 c 9 a + b + c ; tĂž Ăą suy ra a + b + c = 3. CĂŽng tĂž giÂŁ thižt ta cĂą ab+bc+ca = abc(a+b+c), tĂž „y ta suy ra b§t ÂŻng thĂčc sau l  tĂ·ĂŹng Ă·ĂŹng vĂźi b§t ÂŻng thĂčc cŠn chĂčng minh 5(a + b + c)2 7(a + b + c) + 8(ab + bc + ca): 18
  • 19. Âș ĂŸ rÂŹng ta cĂą ÂĄnh giÂĄ cĂŹ bÂŁn sau: (a + b + c)2 3(ab + bc + ca); do vÂȘy Âș cĂą kžt luÂȘn cho b i toÂĄn ta cŠn chÂż ra rÂŹng 5(a + b + c)2 7(a + b + c) + 8(a + b + c)2 3 ; hay a + b + c 3, l  mĂ«t ÂĄnh giÂĄ Ăłng do ta Âą chĂčng minh Ă° tr¶n. Do vÂȘy b§t ÂŻng thĂčc ban Šu ֖c chĂčng minh xong. B i toÂĄn kžt thĂłc.2 1.13 Cho a; b; c l  cÂĄc sĂš thĂŒc dĂ·ĂŹng thoÂŁ mÂąn 1 a + 1 b + 1 c 16(a+b+c). ChĂčng minh rÂŹng: 1 2 + a2 + 1 2 + b2 + 1 2 + c2 1 LĂ­i giÂŁi. B§t ÂŻng thĂčc cŠn chĂčng minh tĂ·ĂŹng Ă·ĂŹng vĂźi a2 2 + a2 + b2 2 + b2 + c2 2 + c2 1: p döng b§t ÂŻng thĂčc Cauchy - Schwarz, ta cĂą a2 2 + a2 + b2 2 + b2 + c2 2 + c2 (a + b + c)2 a2 + b2 + c2 + 6 : NhĂ· vÂȘy Âș kžt thĂłc chĂčng minh ta cŠn chÂż ra rÂŹng (a + b + c)2 a2 + b2 + c2 + 6 1: ThĂŒc hi»n phÂČp khai triÂșn tĂ·ĂŹng Ă·ĂŹng ta ֖c ab + bc + ca 3. Tuy nhi¶n b§t ÂŻng thĂčc n y Ăłng nhĂ­ v o giÂŁ thižt cĂ”a b i toÂĄn. LĂ·u ĂŸ rÂŹng tĂž giÂŁ thižt ta cĂą ab + bc + ca = abc(a + b + c); v  theo mĂ«t ÂĄnh giÂĄ quen thuĂ«c thÂŒ abc(a + b + c) (ab + bc + ca)2 3 , tĂž Ăą ta suy ra ab + bc + ca (ab + bc + ca)2 3 ; hay ab + bc + ca 3. PhÂČp chĂčng minh žn „y ho n t§t.2 1.14 Cho a; b; c; d l  cÂĄc sĂš thĂŒc dĂ·ĂŹng thoÂŁ mÂąn a+b+c+d = 1. TÂŒm giÂĄ trĂ  nhĂ€ nh§t cĂ”a biÂșu thĂčc: P = 1 a2 + b2 + c2 + d2 + 1 abc + 1 bcd + 1 cda + 1 dab LĂ­i giÂŁi. KÂœ hi»u X l  tĂȘng hoÂĄn vĂ . TrĂ·Ăźc hžt ta sĂ» döng AM-GM v  giÂŁ thižt Âș cĂą cÂĄc ÂĄnh giÂĄ sau: abcd a + b + c + d 4 4 = 1 256 ; ab + ac + ad + bc + bd + cd 3(a + b + c + d)2 8 = 3 8 : Kžt hñp cÂĄc ÂĄnh giÂĄ n y vĂźi b§t ÂŻng thĂčc Cauchy - Schwarz ta suy ra ֖c cÂĄc b§t ÂŻng thĂčc sau: 19
  • 20. 1. 1 a2 + b2 + c2 + d2 + X 1 4ab 72 a2 + b2 + c2 + d2 + X 4ab = 49 (a + b + c + d)2 + 2 X ab 49 1 + 2: 3 8 = 28; 2. 7 X 1 4ab 7:62 X 4ab 7:36 4: 3 8 = 168: M°t khÂĄc ÂĄp döng b§t ÂŻng thĂčc AM-GM cho bĂšn sĂš ta l€i cĂą X a bcd r 4 1 4abcd vuut 4 1 1 256 = 64: Kžt hñp ba b§t ÂŻng thĂčc vĂža chĂčng minh Ă° tr¶n, ta suy ra 1 a2 + b2 + c2 + d2 + 2 X 1 ab + X a bcd 28 + 168 + 64 = 260: HĂŹn nĂșa, sĂ» döng giÂŁ thižt a + b + c + d = 1 ta suy ra P = 1 a2 + b2 + c2 + d2 + (a + b + c + d) 1 abc + 1 bcd + 1 cda + 1 dab = 1 a2 + b2 + c2 + d2 + 2 X 1 ab + X a bcd : Do vÂȘy P 260. CuĂši cĂČng, vĂźi a = b = c = d = 1 4 (thoÂŁ mÂąn i·u ki»n) thÂŒ P = 260 n¶n ta suy ra 260 l  giÂĄ trĂ  nhĂ€ nh§t cĂ”a biÂșu thĂčc P. PhÂČp chĂčng minh ho n t§t.2 1.15 Cho x; y; z l  cÂĄc sĂš thĂŒc dĂ·ĂŹng thoÂŁ mÂąn xyz = 1. ChĂčng minh rÂŹng: 18 1 x3 + 1 + 1 y3 + 1 + 1 z3 + 1 (x + y + z)3 LĂ­i giÂŁi. SĂ» döng giÂŁ thižt, dÂč th§y b§t ÂŻng thĂčc cŠn chĂčng minh tĂ·ĂŹng Ă·ĂŹng vĂźi mĂ©i b§t ÂŻng thĂčc trong dÂąy sau: 18 3 x3 x3 + 1 y3 y3 + 1 z3 z3 + 1 (x + y + z)3; 18 x2 x2 + yz + y2 y2 + zx + z2 z2 + xy + (x + y + z)3 54: () p döng b§t ÂŻng thĂčc Cauchy - Schwarz, ta cĂą x2 x2 + yz + y2 y2 + zx + z2 z2 + xy (x + y + z)2 x2 + y2 + z2 + xy + yz + zx : 20
  • 21. NhĂ· vÂȘy nžu kÂœ hi»u V T() l  vž trÂĄi cĂ”a b§t ÂŻng thĂčc () thÂŒ ta cĂą V T() 18(x + y + z)2 x2 + y2 + z2 + xy + yz + zx + (x + y + z)3: žn „y ta ÂĄp döng b§t ÂŻng thĂčc AM-GM Âș cĂą s V T() 2 18(x + y + z)5 x2 + y2 + z2 + xy + yz + zx : NhĂ· vÂȘy Âș kžt thĂłc chĂčng minh, ta cŠn chÂż ra rÂŹng (x + y + z)5 81 2 (x2 + y2 + z2 + xy + yz + zx): TrĂ·Ăźc hžt ta ÂĄp döng b§t ÂŻng thĂčc AM-GM nhĂ· sau: (x+y+z)6 = [(x2+y2+z2)+(xy+yz +zx)+(xy+yz +zx)]3 27(x2+y2+z2)(xy+yz +zx)2: HĂŹn nĂșa, theo mĂ«t kžt quÂŁ quen thuĂ«c ta cĂą (xy + yz + zx)2 3xyz(x + y + z), do Ăą (x + y + z)6 81xyz(x2 + y2 + z2)(x + y + z); hay (x + y + z)5 81(x2 + y2 + z2) do xyz = 1. NhĂ· vÂȘy ta cŠn chÂż ra rÂŹng 2(x2 + y2 + z2) x2 + y2 + z2 + xy + yz + zx: Tuy nhi¶n bÂŹng phÂČp bižn ĂȘi tĂ·ĂŹng Ă·ĂŹng ta thu ֖c 1 2 [(a b)2 + (b c)2 + (c a)2] 0; l  mĂ«t b§t ÂŻng thĂčc hiÂșn nhi¶n Ăłng. Do vÂȘy b§t ÂŻng thĂčc ban Šu Âą ֖c chĂčng minh. B i toÂĄn kžt thĂłc.2 1.16 Cho a; b; c l  cÂĄc sĂš thĂŒc dĂ·ĂŹng thoÂŁ mÂąn a4 + b4 + c4 = 3. ChĂčng minh rÂŹng: a2 b + c + b2 c + a + c2 a + b 3 2 LĂ­i giÂŁi. Ta sÂł i chĂčng minh a2 b + c + b2 c + a + c2 a + b 3 2 r a4 + b4 + c4 4 3 ; tĂž Ăą sĂ» döng giÂŁ thižt Âș suy ra kžt luÂȘn cho b i toÂĄn. ThÂȘt vÂȘy, ÂĄp döng b§t ÂŻng thĂčc Holder, ta cĂą a2 b + c + b2 c + a + c2 a + b 2 [a2(b + c)2 + b2(c + a)2 + c2(a + b)2] (a2 + b2 + c2)3: HĂŹn nĂșa, theo mĂ«t kžt quÂŁ quen thuĂ«c, ta cĂą 2(a2 + b2) (a + b)2; 21
  • 22. tĂž „y ta thižt lÂȘp hai ÂĄnh giÂĄ tĂ·ĂŹng tĂŒ Âș cĂą a2 b + c + b2 c + a + c2 a + b 2 [2a2(b2 + c2) + 2b2(c2 + a2) + 2c2(a2 + b2)] (a2 + b2 + c2)3; hay a2 b + c + b2 c + a + c2 a + b 1 2 s (a2 + b2 + c2)3 a2b2 + b2c2 + c2a2 : NhĂ· vÂȘy Âș kžt thĂłc chĂčng minh ta cŠn chÂż ra rÂŹng s (a2 + b2 + c2)3 a2b2 + b2c2 + c2a2 r a4 + b4 + c4 3 4 3 : ThĂŒc hi»n phÂČp bižn ĂȘi tĂ·ĂŹng Ă·ĂŹng ta thu ֖c (a2 + b2 + c2)6 27(a4 + b4 + c4)(a2b2 + b2c2 + c2a2)2: Tuy nhi¶n b§t ÂŻng thĂčc tr¶n Ăłng nžu ta ÂĄp döng b§t ÂŻng thĂčc AM-GM nhĂ· sau: (a2 + b2 + c2)6 = [(a4 + b4 + c4) + (a2b2 + b2c2 + c2a2) + (a2b2 + b2c2 + c2a2)]3 27(a4 + b4 + c4)(a2b2 + b2c2 + c2a2)2 PhÂČp chĂčng minh žn „y ho n t§t.2 1.17 Cho a; b; c l  cÂĄc sĂš thĂŒc dĂ·ĂŹng thoÂŁ mÂąn a + b + c = 3. ChĂčng minh rÂŹng: a a + b + 1 + b b + c + 1 + c c + a + 1 1 LĂ­i giÂŁi. SĂ» döng giÂŁ thižt, ta th§y rÂŹng cÂĄc b§t ÂŻng thĂčc sau l  tĂ·ĂŹng Ă·ĂŹng vĂźi b§t ÂŻng thĂčc cŠn chĂčng minh a 4 c + b 4 a + c 4 b 1; a(4 a)(4 b) + b(4 b)(4 c) + c(4 c)(4 a) (4 a)(4 b)(4 c); a2b + b2c + c2a + abc 4: B§t ÂŻng thĂčc tr¶n mang tÂœnh hoÂĄn vĂ  giĂșa cÂĄc bižn n¶n khĂŠng m§t tÂœnh tĂȘng quÂĄt, ta giÂŁ sĂ» c nÂŹm giĂșa a v  b. Khi Ăą a(a c)(b c) 0: ThĂŒc hi»n phÂČp khai triÂșn ta ֖c a2b+c2a a2c+abc. TĂž „y ta cĂ«ng th¶m €i l֖ng (b2c+abc) v o hai vž Âș ֖c a2b + b2c + c2a + abc a2c + b2c + 2abc = c(a + b)2: žn „y ta ÂĄp döng AM-GM nhĂ· sau: c(a + b)2 = 1 2 2c(a + b)(a + b) (2c + a + b + a + b)3 2:27 = 4; tĂž Ăą suy ra a2b + b2c + c2a + abc 4, tĂčc l  b§t ÂŻng thĂčc ban Šu Âą ֖c chĂčng minh. 22
  • 23. B i toÂĄn ho n t§t.2 1.18 Cho a; b; c l  cÂĄc sĂš thĂŒc khĂŠng „m thoÂŁ mÂąn a + b + c = 1. ChĂčng minh rÂŹng: 25 27 (1 4ab)2 + (1 4bc)2 + (1 4ca)2 3 LĂ­i giÂŁi. 1. ChĂčng minh (1 4ab)2 + (1 4bc)2 + (1 4ca)2 3. TrĂ·Ăźc hžt ta cĂą p ab; 1 = a + b + c a + b 2 tĂž Ăą suy ra 1 4ab. žn „y ta sĂ» döng giÂŁ thižt cÂĄc bižn khĂŠng „m Âș cĂą 0 1 4ab 1; tĂž Ăą m  (1 4ab)2 1. Thižt lÂȘp hai ÂĄnh giÂĄ tĂ·ĂŹng tĂŒ v  cĂ«ng l€i ta cĂą ngay i·u phÂŁi chĂčng minh. 2. ChĂčng minh (1 4ab)2 + (1 4bc)2 + (1 4ca)2 25 27 . DÂč th§y b§t ÂŻng thĂčc tr¶n tĂ·ĂŹng Ă·ĂŹng vĂźi mĂ©i b§t ÂŻng thĂčc trong dÂąy sau: 3 8(ab + bc + ca) + 16(a2b2 + b2c2 + c2a2) 25 27 ; ab + bc + ca 2(a2b2 + b2c2 + c2a2) 7 27 : Âș ĂŸ rÂŹng ta cĂą ÂŻng thĂčc sau ab 2a2b2 5 9 ab 1 9 7 81 = 2 ab 1 9 2 ; do Ăą ta suy ra ab 2a2b2 5 9 ab 1 9 + 7 81 . žn „y ta thižt lÂȘp hai ÂĄnh giÂĄ tĂ·ĂŹng tĂŒ v  cĂ«ng l€i Âș cĂą ab + bc + ca 2(a2b2 + b2c2 + c2a2) 5 9 ab + bc + ca 1 3 7 27 : HĂŹn nĂșa, theo mĂ«t kžt quÂŁ quen thuĂ«c ta cĂą ab+bc+ca (a + b + c)2 3 = 1 3 , do vÂȘy ta suy ra ab + bc + ca 2(a2b2 + b2c2 + c2a2) 7 27 ; tĂčc l  b§t ÂŻng thĂčc ban Šu Âą ֖c chĂčng minh. TĂąm l€i ta Âą chĂčng minh ֖c 25 27 (1 4ab)2 + (1 4bc)2 + (1 4ca)2 3. PhÂČp chĂčng minh ho n t§t.2 1.18 Cho x; y; z l  cÂĄc sĂš thĂŒc dĂ·ĂŹng thoÂŁ mÂąn xy + yz + zx = 1. ChĂčng minh rÂŹng: 1 1 + xy + z2 + 1 1 + yz + x2 + 1 1 + zx + y2 9 5 LĂ­i giÂŁi. °t x = 1 a ; y = 1 b ; z = 1 c . Khi Ăą sĂ» döng giÂŁ thižt xy + yz + zx = 1, ta th§y rÂŹng 23
  • 24. 1 1 + xy + z2 = xy + yz + zx x2 + xy + xz + 2yz = 1 ab + 1 bc + 1 ca 1 a2 + 1 ab + 1 ac + 2 bc = a(a + b + c) 2a2 + ab + bc + ca ; do Ăą b§t ÂŻng thĂčc Âą cho tĂ·ĂŹng Ă·ĂŹng vĂźi X a 2a2 + ab + bc + ca 9 5(a + b + c) : Nh„n cÂŁ hai vž cĂ”a b§t ÂŻng thĂčc n y vĂźi ab + bc + ca v  chĂł ĂŸ rÂŹng a(ab + bc + ca) 2a2 + ab + bc + ca = a 2a3 2a2 + ab + bc + ca ; ta ֖c 2 X a3 2a2 + ab + bc + ca + 9(ab + bc + ca) 5(a + b + c) a + b + c: p döng b§t ÂŻng thĂčc Cauchy - Schwarz, ta cĂą X a3 2a2 + ab + bc + ca ( X a2)2 X a(2a2 + ab + bc + ca) = ( X a2)2 6abc + ( X a)(2 X a2 X ab) : (1) M°t khÂĄc, tĂž b§t ÂŻng thĂčc cĂŹ bÂŁn (ab + bc + ca)2 3abc(a + b + c), ta l€i cĂą 3abc (ab + bc + ca)2 a + b + c : (2) Kžt hñp (1) v  (2), ta suy ra X a3 2a2 + ab + bc + ca X ( a2)2( X a) X 2( ab + bc + ca)2 + ( X a)2(2 X a2 X ab) : = X ( X a2)( a) 2 X a2 + 3 X ab : CuĂši cĂČng ta chÂż cŠn chĂčng minh 2(a2 + b2 + c2)(a + b + c) 2(a2 + b2 + c2) + 3(ab + bc + ca) + 9(ab + bc + ca) 5(a + b + c) a + b + c: Sau khi khai triÂșn v  rĂłt gĂ„n, ta ֖c b§t ÂŻng thĂčc hiÂșn nhi¶n Ăłng (ab + bc + ca)(a2 + b2 + c2 ab bc ca) 0: B i toÂĄn ֖c chĂčng minh xong.2 24
  • 25. 1.19 Cho a; b; c l  cÂĄc sĂš thĂŒc dĂ·ĂŹng thoÂŁ mÂąn a + b + c = 1 a + 1 b + 1 c . ChĂčng minh rÂŹng: (b + c a)(c + a b)(a + b c) 1 LĂ­i giÂŁi 1. B§t ÂŻng thĂčc cŠn chĂčng minh mang tÂœnh Ăši xĂčng giĂșa cÂĄc bižn, do Ăą khĂŠng m§t tÂœnh tĂȘng quÂĄt, ta giÂŁ sĂ» a b c. Khi Ăą a + b c 0 v  c + a b 0. Nžu b+ca 0 thÂŒ b§t ÂŻng thĂčc hiÂșn nhi¶n Ăłng do (b+ca)(c+ab)(a+bc) 0 1. Do Ăą ta chÂż cŠn giÂŁi quyžt b i toÂĄn trong trĂ·Ă­ng hñp b+ca 0. LĂłc n y ta °t x = b+ca; y = c + a b; z = a + b c. Khi Ăą ta vižt l€i i·u ki»n nhĂ· sau x; y; z 0; x + y + z = 2 x + y + 2 y + z + 2 z + x ; v  ta cŠn chĂčng minh xyz 1: Ta sÂł giÂŁi quyžt b i toÂĄn bÂŹng phĂ·ĂŹng phÂĄp phÂŁn chĂčng. ThÂȘt vÂȘy, giÂŁ sĂ» rÂŹng xyz 1. Khi Ăą sĂ» döng b§t ÂŻng thĂčc AM-GM, ta suy ra x + y + z = 2 x + y + 2 y + z + 2 z + x 1 p xy + 1 p yz + 1 p zx ; hay p x + p y + p z p xyz(x + y + z). HĂŹn nĂșa, ta cĂŽng cĂą xyz 1 n¶n p x + p y + p z x + y + z: Tuy nhi¶n theo b§t ÂŻng thĂčc AM-GM, ta l€i cĂą p x x + 1 2 . Ta thižt lÂȘp th¶m hai ÂĄnh giÂĄ tĂ·ĂŹng tĂŒ nĂșa Âș cĂą x + y + z + 3 2 p x + p y + p z x + y + z; hay x + y + z 3. NhĂ·ng „y l  mĂ«t ÂĄnh giÂĄ sai vÂŒ theo mĂ«t kžt quÂŁ quen thuĂ«c, ta cĂą x + y + z = 2 x + y + 2 y + z + 2 z + x 9 x + y + z ; dšn tĂźi x + y + z 3. M„u thušn n y chĂčng tĂ€ i·u giÂŁ sĂ» ban Šu l  sai, do vÂȘy xyz 1. PhÂČp chĂčng minh ho n t§t.2 LĂ­i giÂŁi 2. B§t ÂŻng thĂčc cŠn chĂčng minh mang tÂœnh Ăši xĂčng giĂșa cÂĄc bižn, do Ăą khĂŠng m§t tÂœnh tĂȘng quÂĄt, ta giÂŁ sĂ» a b c. Khi Ăą a + b c 0 v  c + a b 0. Nžu b+ca 0 thÂŒ b§t ÂŻng thĂčc hiÂșn nhi¶n Ăłng do (b+ca)(c+ab)(a+bc) 0 1. Do Ăą ta chÂż cŠn giÂŁi quyžt b i toÂĄn trong trĂ·Ă­ng hñp b+ca 0. LĂłc n y ta °t x = b+ca; y = c + a b; z = a + b c. Khi Ăą ta vižt l€i i·u ki»n nhĂ· sau x; y; z 0; x + y + z = 2 x + y + 2 y + z + 2 z + x ; v  ta cŠn chĂčng minh xyz 1: 25
  • 26. Ta sÂł giÂŁi quyžt b i toÂĄn bÂŹng phĂ·ĂŹng phÂĄp phÂŁn chĂčng. ThÂȘt vÂȘy, giÂŁ sĂ» rÂŹng xyz 1. Khi Ăą, tĂž giÂŁ thižt, ta suy ra (x + y + z)2(xy + yz + zx) = 2(x + y + z)2 + 2(xy + yz + zx) + xyz(x + y + z): () Tuy nhi¶n, theo b§t ÂŻng thĂčc AM-GM v  theo i·u giÂŁ sĂ» Ă° tr¶n, ta cĂą cÂĄc ÂĄnh giÂĄ xy + yz + zx 3 3 p x2y2z2 3; x + y + z 3 3 p xyz 3; do vÂȘy ta suy ra 2(x + y + z)2(xy + yz + zx) 3 2(x + y + z)2; 2(x + y + z)2(xy + yz + zx) 9 2(xy + yz + zx); (x + y + z)2(xy + yz + zx) 9 xyz(x + y + z): CĂ«ng vž theo vž cÂĄc ÂĄnh giÂĄ tr¶n l€i, ta ֖c (x + y + z)2(xy + yz + zx) 2(x + y + z)2 + 2(xy + yz + zx) + xyz(x + y + z); trÂĄi vĂźi (). M„u thušn n y chĂčng tĂ€ i·u giÂŁ sĂ» ban Šu l  sai, do vÂȘy xyz 1. PhÂČp chĂčng minh ho n t§t.2 1.20 Cho a; b; c l  cÂĄc sĂš thĂŒc dĂ·ĂŹng thoÂŁ mÂąn a + b + c = 3. ChĂčng minh rÂŹng: 1 5a2 + ab + bc + 1 5b2 + bc + ca + 1 5c2 + ca + ab 3 7 LĂ­i giÂŁi. p döng b§t ÂŻng thĂčc Cauchy - Schwarz, ta cĂą 1 5a2 + ab + bc + 1 5b2 + bc + ca + 1 5c2 + ca + ab = X cyc (b + c)2 (b + c)2(5a2 + ab + bc) 4(a + b + c)2 X cyc (b + c)2(5a2 + ab + bc) : Theo Ăą, ta cŠn chĂčng minh rÂŹng 4(a + b + c)2 X cyc (b + c)2(5a2 + ab + bc) 3 7 : SĂ» döng giÂŁ thižt a + b + c = 3, ta th§y rÂŹng b§t ÂŻng thĂčc tr¶n tĂ·ĂŹng Ă·ĂŹng vĂźi X 28(a + b + c)4 27[ cyc (b + c)2(5a2 + ab + bc)]: Sau khi khai triÂșn v  rĂłt gĂ„n, ta ֖c 28 X a4 + 58 X cyc a3b + 85 X cyc ab3 156 X a2b2 + 15abc(a + b + c): 26
  • 27. Âș chĂčng minh b§t ÂŻng thĂčc n y, trĂ·Ăźc hžt ta chĂł ĂŸ žn cÂĄc ÂĄnh giÂĄ cĂŹ bÂŁn sau (thu ֖c bÂŹng b§t ÂŻng thĂčc AM-GM): X cyc a3b + X cyc ab3 2 X a2b2; X a4 + X cyc ab3 X cyc a3b + X cyc ab3 2 X a2b2; X a4 X a2b2 abc(a + b + c): TĂž Ăą ta suy ra 58 X cyc a3b + 58 X cyc ab3 116 X a2b2; 27 X a4 + 27 X cyc ab3 54 X a2b2; X a4 + 14 X a2b2 15abc(a + b + c): CĂ«ng vž theo vž cÂĄc ÂĄnh giÂĄ tr¶n, ta thu ֖c b§t ÂŻng thĂčc cŠn chĂčng minh. B i toÂĄn kžt thĂłc.2 1.21 Cho a; b; c l  cÂĄc sĂš thĂŒc dĂ·ĂŹng thay ĂȘi b§t kÂŒ. ChĂčng minh rÂŹng: b + c 2a2 + bc + c + a 2b2 + ca + a + b 2c2 + ab 6 a + b + c LĂ­i giÂŁi. Nh„n cÂŁ hai vž cĂ”a b§t ÂŻng thĂčc cho 4(a + b + c), ta ֖c 4(b + c)(a + b + c) 2a2 + bc + 4(c + a)(a + b + c) 2b2 + ca + 4(a + b)(a + b + c) 2c2 + ab 24: Do 4(b + c)(a + b + c) 2a2 + bc = (a + 2b + 2c)2 2a2 + bc a2 2a2 + bc n¶n ta cĂą X(a + 2b + 2c)2 2a2 + bc 24 + X a2 2a2 + bc : B§t ÂŻng thĂčc n y ֖c suy ra bÂŹng cÂĄch cĂ«ng hai b§t ÂŻng thĂčc a2 2a2 + bc + b2 2b2 + ca + c2 2c2 + ab 1; (a + 2b + 2c)2 2a2 + bc + (b + 2c + 2a)2 2b2 + ca + (c + 2c + 2b)2 2c2 + ab 25: Do a2 2a2 + bc = 1 2 bc 2(2a2 + bc) n¶n b§t ÂŻng thĂčc thĂč nh§t tĂ·ĂŹng Ă·ĂŹng vĂźi bc 2a2 + bc + ca 2b2 + ca + ab 2c2 + ab 1; Ăłng vÂŒ theo b§t ÂŻng thĂčc Cauchy - Schwarz X bc 2a2 + bc X bc 2 X bc(2a2 + bc) = 1: 27
  • 28. B„y giĂ­ ta sÂł chĂčng minh b§t ÂŻng thĂčc thĂč hai. „y l  b§t ÂŻng thĂčc Ăši xĂčng n¶n khĂŠng m§t tÂœnh tĂȘng quÂĄt, ta giÂŁ sĂ» c = minfa; b; cg. °t t = b + c 2 , ta sÂł chĂčng minh (a + 2b + 2c)2 2a2 + bc + (b + 2c + 2a)2 2b2 + ca 2(3t + 2c)2 2t2 + tc : () SĂ» döng b§t ÂŻng thĂčc Cauchy - Schwarz, ta cĂą (a + 2b + 2c)2 2a2 + bc + (b + 2c + 2a)2 2b2 + ca [b(a + 2b + 2c) + a(b + 2c + 2a)]2 b2(2a2 + bc) + a2(2b2 + ca) = 2(4t2 ab + 2tc)2 2a2b2 3abtc + 4t3c : VÂŒ tc ab t2 n¶n 2a2b2 3abtc (2t4 3t3c) = (t2 ab)(2t2 + 2ab 3tc) 0; tĂž Ăą dšn žn (a + 2b + 2c)2 2a2 + bc + (b + 2c + 2a)2 2b2 + ca 2(4t2 ab + 2tc)2 2a2b2 3abtc + 4t3c 2(3t2 + 2tc)2 2t4 3t3c + 4t3c = 2(3t + 2c)2 2t2 + tc : M°t khÂĄc, ta l€i cĂą (c + 2c + 2b)2 2c2 + ab (4t + c)2 t2 + 2c2 : () Kžt hñp hai ÂĄnh giÂĄ () v  (), ta Ă·a b i toÂĄn v· vi»c chĂčng minh 2(3t + 2c)2 2t2 + tc + (4t + c)2 t2 + 2c2 25: Sau khi thu gĂ„n, ta ֖c b§t ÂŻng thĂčc hiÂșn nhi¶n Ăłng c(31t + 16c)(t c)2 t(2t + c)(t2 + 2c2) 0: B i toÂĄn ֖c chĂčng minh xong.2 1.22 Cho a; b; c; d l  cÂĄc sĂš thĂŒc khĂŠng „m thĂ€a mÂąn a2+b2+c2+d2 = 1. ChĂčng minh rÂŹng: a b2 + 1 + b c2 + 1 + c d2 + 1 + d a2 + 1 p a + b 4(a p b + c p c + d p d)2 5 LĂ­i giÂŁi. p döng b§t ÂŻng thĂčc Cauchy - Schwarz, ta cĂą a b2 + 1 + b c2 + 1 + c d2 + 1 + d a2 + 1 = a3 a2b2 + a2 + b3 b2c2 + b2 + c3 c2d2 + c2 + d3 d2a2 + d2 p a + b (a p b + c p d)2 p c + d a2 + b2 + c2 + d2 + a2b2 + b2c2 + c2d2 + a2d2 : 28
  • 29. NhĂ· vÂȘy, Âș kžt thĂłc chĂčng minh, ta cŠn chÂż ra rÂŹng a2 + b2 + c2 + d2 + a2b2 + b2c2 + c2d2 + a2d2 5 4 ; hay (a2 + c2)(b2 + d2) 1 4 . Tuy nhi¶n „y l€i l  ÂĄnh giÂĄ Ăłng vÂŒ theo b§t ÂŻng thĂčc AM-GM: (a2 + c2)(b2 + d2) (a2 + c2 + b2 + d2)2 4 = 1 4 ; do vÂȘy b§t ÂŻng thĂčc ban Šu ֖c chĂčng minh xong. B i toÂĄn kžt thĂłc.2 1.23 Cho x,y,z l  cÂĄc sĂš thĂŒc thuĂ«c o€n [0; 1]. ChĂčng minh rÂŹng: x 3 p 1 + y3 + y 3 p 1 + z3 + z 3 p 1 + x3 3 3 p 1 + xyz LĂ­i giÂŁi. Do x; y; z 2 [0; 1] n¶n ta cĂą x 3 p 1 + y3 + y 3 p 1 + z3 + z 3 p 1 + x3 1 3 p 1 + y3 + 1 3 p 1 + z3 + 1 3 p 1 + x3 : Âș ĂŸ rÂŹng theo b§t ÂŻng thĂčc Holder, ta ֖c ÂĄnh giÂĄ sau vĂźi mĂ„i sĂš thĂŒc dĂ·ĂŹng a; b; c: (a + b + c)3 9(a3 + b3 + c3); hay (a + b + c) 3 p 9(a3 + b3 + c3). SĂ» döng ÂĄnh giÂĄ n y, ta cĂą 1 3 p 1 + y3 + 1 3 p 1 + z3 + 1 3 p 1 + x3 s 9 3 1 1 + y3 + 1 1 + x3 + 1 1 + z3 : NhĂ· vÂȘy, Âș kžt thĂłc chĂčng minh, ta cŠn chÂż ra rÂŹng 1 1 + y3 + 1 1 + x3 + 1 1 + z3 3 1 + xyz : () Âș ĂŸ rÂŹng vĂźi hai sĂš thĂŒc a; b thay ĂȘi trong o€n [0; 1] ta luĂŠn cĂą 1 1 + a2 + 1 1 + b2 2 1 + ab = (ab 1)(a b)2 (1 + a2)(1 + b2)(1 + ab) 0: SĂ» döng ÂĄnh giÂĄ n y, ta ֖c 1 1 + x3 + 1 1 + y3 + 1 1 + z3 + 1 1 + xyz 2 1 + p x3y3 + 2 1 + p z4xy 4 1 + xyz : Do vÂȘy ÂĄnh giÂĄ () ֖c chĂčng minh, dšn žn b§t ÂŻng thĂčc ban Šu Ăłng. PhÂČp chĂčng minh ho n t§t.2 1.24 Cho a; b; c l  cÂĄc sĂš thĂŒc dĂ·ĂŹng thay ĂȘi b§t kÂŒ. ChĂčng minh rÂŹng: a2 b + c + b2 a + c + c2 a + b a + b + c 2 29
  • 30. LĂ­i giÂŁi 1. p döng b§t ÂŻng thĂčc Cauchy-Schwartz, ta cĂą a2 b + c + b2 a + c + c2 a + b (a + b + c)2 2(a + b + c) = a + b + c 2 : PhÂČp chĂčng minh ho n t§t. 2 LĂ­i giÂŁi 2. p döng b§t ÂŻng thĂčc AM-GM cho hai sĂš dĂ·ĂŹng, ta cĂą a2 b + c + b + c 4 a: CĂ«ng vž theo vž ÂĄnh giÂĄ n y vĂźi hai ÂĄnh giÂĄ tĂ·ĂŹng tĂŒ khÂĄc, ta ֖c: a2 b + c + b2 a + c + c2 a + b + a + b + c 2 a + b + c; tĂž Ăą ta thu ֖c b§t ÂŻng thĂčc cŠn chĂčng minh. B i toÂĄn kžt thĂłc.2 LĂ­i giÂŁi 3. B§t ÂŻng thĂčc ban Šu mang tÂœnh Ăši xĂčng giĂșa cÂĄc bižn, do Ăą khĂŠng m§t tÂœnh tĂȘng quÂĄt, ta giÂŁ sĂ» a b c. Khi Ăą ta cĂą 1 b + c 1 a + c 1 a + b : NhĂ· vÂȘy, theo b§t ÂŻng thĂčc Chebyshev, ta cĂą a2 b + c + b2 a + c + c2 a + b 1 3 :(a2 + b2 + c2):( 1 a + b + 1 b + c + 1 a + c ): žn „y ta ÂĄp döng hai ÂĄnh giÂĄ cĂŹ bÂŁn x2 + y2 + z2 (x + y + z)2 3 v  1 x + 1 y + 1 z 9 x + y + z Âș cĂą a2 b + c + b2 a + c + c2 a + b 1 3 : (a + b + c)2 3 : 9 2(a + b + c) = a + b + c 2 : PhÂČp chĂčng minh ho n t§t.2 1.25 Cho a; b; c l  cÂĄc sĂš thĂŒc dĂ·ĂŹng thay ĂȘi b§t kÂŒ. ChĂčng minh rÂŹng: 4 p 3 + a4 + 4 p 3 + b4 + 4 p 3 + c4 4 p 108(a + b + c) LĂ­i giÂŁi. p döng b§t ÂŻng thĂčc Holder, ta cĂą (1 + 3)(1 + 3)(1 + 3)(a4 + 3) (a + 3)4; tĂž Ăą suy ra 4 p 3 + a4 3 + a 4 p 64 . Thižt lÂȘp cÂĄc ÂĄnh giÂĄ tĂ·ĂŹng tĂŒ v  cĂ«ng l€i, ta ֖c 4 p 3 + a4 + 4 p 3 + b4 + 4 p 3 + c4 9 + a + b + c 4 p 64 : HĂŹn nĂșa, theo b§t ÂŻng thĂčc AM-GM, ta cĂą 9 + a + b + c = 3 + 3 + 3 + (a + b + c) 4 4 p 27(a + b + c); 30
  • 31. nhĂ· vÂȘy 4 p 3 + a4 + 4 p 3 + b4 + 4 p 3 + c4 4 4 p 27(a + b + c) 4 p 64 = 4 p 108(a + b + c): PhÂČp chĂčng minh ho n t§t.2 1.26 Cho a; b l  cÂĄc sĂš thĂŒc dĂ·ĂŹng thoÂŁ mÂąn ab 1. ChĂčng minh rÂŹng: 1 1 + a2 + 1 1 + b2 2 1 + ab LĂ­i giÂŁi. ThĂŒc hi»n phÂČp bižn ĂȘi tĂ·ĂŹng Ă·ĂŹng, ta thu ֖c dÂąy cÂĄc ÂĄnh giÂĄ sau: 2 + a2 + b2 a2b2 + a2 + b2 + 1 2 1 + ab ; 2 + 2ab + a3b + b3a + a2 + b2 2a2b2 2a2 2b2 2 0; (ab 1)(a b)2 0: ÂĄnh giÂĄ cuĂši cĂČng Ăłng do ab 1, do vÂȘy b§t ÂŻng thĂčc ban Šu ֖c chĂčng minh. B i toÂĄn kžt thĂłc.2 1.27 Cho a; b; c l  cÂĄc sĂš thĂŒc dĂ·ĂŹng thoÂŁ mÂąn a + b + c = 1. ChĂčng minh rÂŹng: c + ab a + b + a + bc b + c + b + ac a + c 2 LĂ­i giÂŁi. Âș ĂŸ rÂŹng ta cĂą c + ab = c(a + b + c) + ab = (c + a)(c + b); do vÂȘy b§t ÂŻng thĂčc cŠn chĂčng minh tĂ·ĂŹng Ă·ĂŹng vĂźi (c + a)(c + b) a + b + (b + a)(b + c) a + c + (a + b)(a + c) b + c 2: p döng ÂĄnh giÂĄ cĂŹ bÂŁn x2 + y2 + z2 xy + yz + zx, ta th§y ÂĄnh giÂĄ tr¶n Ăłng do (c + a)(c + b) a + b + (b + a)(b + c) a + c + (a + b)(a + c) b + c b + c + a + b + c + a = 2: PhÂČp chĂčng minh ho n t§t.2 1.28 Cho x; y; z l  cÂĄc sĂš thĂŒc dĂ·ĂŹng thoÂŁ mÂąn 2x + 3y + z = 1. TÂŒm giÂĄ trĂ  nhĂ€ nh§t cĂ”a biÂșu thĂčc: P = x3 + y3 + z3 LĂ­i giÂŁi. p döng b§t ÂŻng thĂčc Holder, ta cĂą p 2 + 3 P(2 p 3 + 1)2 = (x3 + y3 + z3)(2 p 2 + 3 p 3 + 1)(2 p 2 + 3 p 3 + 1) (2x + 3y + z)3 = 1: 31
  • 32. NhĂ· vÂȘy P 1 (2 p 2 + 3 p 3 + 1)2 . CuĂši cĂČng, vĂźi x = p 2 p 2 + 3 2 p 3 + 1 ,y = p 3 p 2 + 3 2 p 3 + 1 v  z = 1 p 2 + 3 2 p 3 + 1 (thoÂŁ mÂąn i·u ki»n) thÂŒ P = 1 p 2 + 3 (2 p 3 + 1)2 n¶n ta kžt luÂȘn 1 p 2 + 3 (2 p 3 + 1)2 l  giÂĄ trĂ  nhĂ€ nh§t cĂ”a biÂșu thĂčc P. B i toÂĄn kžt thĂłc.2 1.29 Cho a; b; c l  cÂĄc sĂš thĂŒc thoÂŁ mÂąn a2 +ab+b2 = 3. TÂŒm giÂĄ trĂ  nhĂ€ nh§t v  giÂĄ trĂ  lĂźn nh§t cĂ”a biÂșu thĂčc P = a2 ab 3b2 LĂ­i giÂŁi. VĂźi b = 0 thÂŒ tĂž giÂŁ thižt ta suy ra a2 = 3, tĂž Ăą biÂșu thĂčc P cĂą giÂĄ trĂ  l  3. VĂźi b6= 0, xÂČt biÂșu thĂčc Q = P 3 = a2 ab 3b2 a2 + ab + b2 = x2 x 3 x2 + x + 1 ; trong Ăą x = a b . TĂž „y ta suy ra (Q 1)x2 + (Q + 1)x + Q + 3 = 0: Coi Ăą l  mĂ«t phĂ·ĂŹng trÂŒnh theo ©n x. XÂČt bi»t thĂčc cĂ”a phĂ·ĂŹng trÂŒnh tr¶n, ta th§y rÂŹng Âș phĂ·ĂŹng trÂŒnh tr¶n cĂą nghi»m thÂŒ (Q + 1)2 4(Q 1)(Q + 3) 0; tĂž „y ta suy ra p 3 3 4 3 Q 3 + 4 p 3 3 . HĂŹn nĂșa, do P = 3Q n¶n ta cĂą p 3 P 3 + 4 3 4 p 3: CuĂši cĂČng, vĂźi a = p 2 p 3 v  b = p 2 + p 3 thÂŒ P = 3 4 p 3; vĂźi a = p 2 + p 3 v  b = p 2 p 3 thÂŒ P = 3 + 4 p 3 n¶n ta kžt luÂȘn 3 4 p 3 v  3 + 4 p 3 lŠn l֖t l  giÂĄ trĂ  nhĂ€ nh§t v  giÂĄ trĂ  lĂźn nh§t cĂ”a biÂșu thĂčc P. B i toÂĄn kžt thĂłc.2 1.30 Cho a; b; c l  cÂĄc sĂš thĂŒc thoÂŁ mÂąn a2 + 2b2 = 3c2. ChĂčng minh rÂŹng: 1 a + 2 b 3 c LĂ­i giÂŁi. TĂž giÂŁ thižt, ta suy ra (3c)2 = (a2 + 2b2)(1 + 2). TĂž „y ta ÂĄp döng b§t ÂŻng thĂčc Cauchy - Schwarz Âș cĂą (3c)2 (a + 2b)2; tĂž Ăą suy ra 3c a + 2b. () HĂŹn nĂșa, cĂŽng theo b§t ÂŻng thĂčc Cauchy - Schwarz, ta cĂą 1 a + 1 b + 1 b 9 a + 2b : () 32
  • 33. Kžt hñp hai ÂĄnh giÂĄ () v  (), ta thu ֖c b§t ÂŻng thĂčc cŠn chĂčng minh. B i toÂĄn kžt thĂłc.2 1.31 Cho x; y; z l  cÂĄc sĂš thĂŒc khĂŠng „m thoÂŁ mÂąn x2 + y2 + z2 = 3. TÂŒm giÂĄ trĂ  lĂźn nh§t cĂ”a biÂșu thĂčc: P = xy + yz + zx + 5 x + y + z LĂ­i giÂŁi. Âș ĂŸ rÂŹng P = (x + y + z)2 x2 y2 z2 2 + 5 x + y + z = (x + y + z)2 2 + 5 x + y + z 3 2 ; tĂž Ăą °t t = x + y + z, ta Ă·a b i toÂĄn v· vi»c tÂŒm giÂĄ trĂ  lĂźn nh§t cĂ”a biÂșu thĂčc Q = t2 + 10 t : Âș ĂŸ rÂŹng tĂž ÂĄnh giÂĄ x2 +y2 +z2 (x+y +z)2 3(x2 +y2 +z2), ta suy ra p 3 t 3, do vÂȘy t2 + 10 t 37 3 = (t 3)(3t2 + 9t 10) 3t 0: NhĂ· vÂȘy Q 37 3 , v  vÂŒ P = Q 2 3 2 n¶n P 37 6 3 2 = 14 3 : CuĂši cĂČng, vĂźi x = y = z = 1 (thoÂŁ mÂąn i·u ki»n) thÂŒ P = 14 3 n¶n ta kžt luÂȘn 14 3 l  giÂĄ trĂ  lĂźn nh§t cĂ”a biÂșu thĂčc P. B i toÂĄn kžt thĂłc.2 1.32 Cho x; y l  cÂĄc sĂš thĂŒc dĂ·ĂŹng thoÂŁ mÂąn 2y x. ChĂčng minh rÂŹng: 1 x3(2y x) + x2 + y2 3 LĂ­i giÂŁi. Ta th§y rÂŹng 1 x3(2y x) + x2 + y2 = 1 x2(2xy x2) + +x2 + (y2 + x2 x2); v  vÂŒ x2 + y2 2xy theo b§t ÂŻng thĂčc AM-GM n¶n 1 x3(2y x) + x2 + y2 1 x2(2xy x2) + x2 + (2xy x2): žn „y ta ÂĄp döng b§t ÂŻng thĂčc AM-GM mĂ«t lŠn nĂșa Âș cĂą 1 x3(2y x) s + x2 + y2 3 3 1 x2(2xy x2) :x2:(2xy x2) = 3: PhÂČp chĂčng minh ho n t§t.2 33
  • 34. 1.33 Cho a; b; c l  cÂĄc sĂš thĂŒc dĂ·ĂŹng thoÂŁ mÂąn ab + bc + ca = 2abc. ChĂčng minh rÂŹng: 1 a(2a 1)2 + 1 b(2b 1)2 + 1 c(2c 1)2 1 2 LĂ­i giÂŁi. °t m = 1 a ; n = 1 b ; p = 1 c . Khi Ăą i·u ki»n Âą cho tĂ·ĂŹng Ă·ĂŹng vĂźi m + n + p = 2 (Âș ĂŸ rÂŹng tĂž „y ta cĂą m; n; p 2), v  b§t ÂŻng thĂčc Âą cho ֖c vižt l€i th nh m3 (2 m)2 + n3 (2 n)2 + p3 (2 p)2 1 2 : p döng b§t ÂŻng thĂčc AM-GM, ta cĂą m3 (2 m)2 + 2 m 8 + 2 m 8 3m 4 ; tĂž Ăą suy ra m3 (2 m)2 m 1 2 . Thižt lÂȘp hai ÂĄnh giÂĄ tĂ·ĂŹng tĂŒ cho n v  p v  cĂ«ng l€i, ta ֖c m3 (2 m)2 + n3 (2 n)2 + p3 (2 p)2 m + n + p 3 2 = 1 2 : PhÂČp chĂčng minh ho n t§t.2 1.34 Cho a; b; c l  cÂĄc sĂš thĂŒc khĂŠng „m thoÂŁ mÂąn a + 2b + 3c = 4. ChĂčng minh rÂŹng: (a2b + b2c + c2a + abc)(ab2 + bc2 + ca2 + abc) 8 LĂ­i giÂŁi. p döng b§t ÂŻng thĂčc AM-GM, ta cĂą 8(a2b + b2c + c2a + abc)(ab2 + bc2 + ca2 + abc) = 4(a2b + b2c + c2a + abc):2(ab2 + bc2 + ca2 + abc) (a2b + b2c + c2a + 2ab2 + 2bc2 + 2ca2 + 3abc)2: HĂŹn nĂșa, ta cĂŽng cĂą (a + 2b)(b + 2c)(c + 2a) = 9abc + 2a2b + 2ac2 + 4a2c + 2b2c + 4b2a + 4c2b 2(a2b + b2c + c2a + 2ab2 + 2bc2 + 2ca2 + 3abc); do vÂȘy 8(a2b + b2c + c2a + abc)(ab2 + bc2 + ca2 + abc) (a + 2b)(b + 2c)(c + 2a) 2 2 . M°t khÂĄc, theo b§t ÂŻng thĂčc AM-GM, ta cĂą 4(a + 2b)(b + 2c)(c + 2a) = (a + 2b)(4b + 8c)(c + 2a) 3a + 6b + 9c 3 3 = (a + 2b + 3c)3 = 64: NhĂ· vÂȘy, ta suy ra 8(a2b + b2c + c2a + abc)(ab2 + bc2 + ca2 + abc) 64 4:2 2 = 64; hay (a2b + b2c + c2a + abc)(ab2 + bc2 + ca2 + abc) 8. 34
  • 35. PhÂČp chĂčng minh ho n t§t.2 1.35 Cho a; b; c l  cÂĄc sĂš thĂŒc dĂ·ĂŹng thay ĂȘi b§t kÂŒ. ChĂčng minh rÂŹng: ab a + 3b + 2c + bc b + 3c + 2a + ac c + 3a + 2b a + b + c 6 LĂ­i giÂŁi. SĂ» döng ÂĄnh giÂĄ cĂŹ bÂŁn 9 x + y + z 1 x + 1 y + 1 z , ta cĂą 9 a + 3b + 2c = 9 (a + c) + (b + c) + 2b 1 a + c + 1 b + c + 1 2b : TĂž Ăą ta suy ra 9ab a + 3b + 2c ab a + c + ab b + c + a 2 . Ho n to n tĂ·ĂŹng tĂŒ, ta cĂŽng cĂą 9bc b + 3c + 2a bc b + a + bc c + a + b 2 ; v  9ca c + 3a + 2b ca c + b + ca a + b + c 2 : CĂ«ng vž theo vž cÂĄc ÂĄnh giÂĄ tr¶n, ta thu ֖c 9ab a + 3b + 2c + 9bc b + 3c + 2a + 9ca c + 3a + 2b ca + ab b + c + ab + bc a + c + bc + ca b + a + a + b + c 2 = 3(a + b + c) 2 ; tĂž „y ta ֖c b§t ÂŻng thĂčc cŠn chĂčng minh. B i toÂĄn kžt thĂłc.2 1.36 Cho a; b; c l  cÂĄc sĂš thĂŒc dĂ·ĂŹng thoÂŁ mÂąn abc = 1. ChĂčng minh rÂŹng: 1 a + b + 4 + 1 b + c + 4 + 1 c + a + 4 1 2 LĂ­i giÂŁi 1. °t x = p a; y = p b; z = p c. Khi Ăą ta phÂŁi chĂčng minh 1 x2 + y2 + 4 + 1 y2 + z2 + 4 + 1 z2 + x2 + 4 1 2 vĂźi x; y; z 0 v  xyz = 1. Do 1 x2 + y2 + 4 = 1 x2 + y2 x2 + y2 + 4 = 1 (x + y)2 + (x y)2 2(x2 + y2 + 4) n¶n b§t ÂŻng thĂčc n y cĂą thÂș ֖c vižt l€i th nh X (x + y)2 x2 + y2 + 4 + X (x y)2 x2 + y2 + 4 2: KhĂŠng m§t tÂœnh tĂȘng quÂĄt, ta giÂŁ sĂ» x y z. SĂ» döng b§t ÂŻng thĂčc Cauchy - Schwarz, ta cĂą X (x + y)2 x2 + y2 + 4 [(x + y) + (y + z) + (z + x)]2 X (x2 + y2 + 4) ; v  X (x y)2 x2 + y2 + 4 [x y + y z + x z]2 X (x2 + y2 + 4) : 35
  • 36. TĂž „y ta Ă·a b i toÂĄn v· chĂčng minh 2(x + y + z)2 + 2(x z)2 2(x2 + y2 + z2) + 12; hay 2(x z)2 + 4(xy + yz + zx 3) 0. Tuy nhi¶n „y l€i l  ÂĄnh giÂĄ Ăłng do (x z)2 0 v  theo b§t ÂŻng thĂčc AM-GM thÂŒ xy + yz + zx 3 3 p x2y2z2 = 3; do vÂȘy b§t ÂŻng thĂčc ban Šu ֖c chĂčng minh xong. B i toÂĄn kžt thĂłc.2 LĂ­i giÂŁi 2. °t x = 3 p a; y = 3 p b; z = 3 p c. Khi Ăą x; y; z 0; xyz = 1 v  ta cŠn chĂčng minh 1 x3 + y3 + 4 + 1 y3 + z3 + 4 + 1 z3 + x3 + 4 1 2 VĂźi chĂł ĂŸ ta cĂą ÂĄnh giÂĄ x3 + y3 xy(x + y), çng thĂ­i l€i cĂą 4 = 4xyz, ta Ă·a b i toÂĄn v· vi»c chĂčng minh 1 xy(x + y + 4z) + 1 yz(y + z + 4x) + 1 zx(z + x + 4y) 1 2 ; hay x + y x + y + 4z + y + z y + z + 4x + z + x z + x + 4y 1: p döng b§t ÂŻng thĂčc Cauchy-Schwartz, ta cĂą x + y x + y + 4z + y + z y + z + 4x + z + x z + x + 4y 4(x + y + z)2 X (x + y)(x + y + 4z) = 4(x + y + z)2 2(x2 + y2 + z2) + 10(xy + yz + zx) ; nhĂ· vÂȘy, Âș kžt thĂłc chĂčng minh, ta cŠn chÂż ra rÂŹng 4(x + y + z)2 2(x2 + y2 + z2) + 10(xy + yz + zx); hay x2 + y2 + z2 xy + yz + zx. Tuy nhi¶n „y l€i l  mĂ«t ÂĄnh giÂĄ Ăłng, do vÂȘy b§t ÂŻng thĂčc ban Šu ֖c chĂčng minh xong. B i toÂĄn kžt thĂłc.2 1.37 Cho a; b; c l  cÂĄc sĂš thĂŒc dĂ·ĂŹng thoÂŁ mÂąn ab + bc + ca = 1. ChĂčng minh rÂŹng: r 3 1 a r + 6b + 3 1 b r + 6c + 3 1 c + 6a 1 abc LĂ­i giÂŁi. p döng b§t ÂŻng thĂčc Holder ta cĂą r 3 1 a r + 6b + 3 1 b r + 6c + 3 1 c !3 + 6a 1 a + 6b + 1 b + 6c + 1 c + 6a 1 p 3 :3 1 p 3 :3 = 9 1 a + 1 b + 1 c + 6a + 6b + 6c : () 36
  • 37. HĂŹn nĂșa, sĂ» döng ÂĄnh giÂĄ cĂŹ bÂŁn xy + yz + zx (x + y + z)2 3 , ta cĂą abc 1 a + 1 b + 1 c + 6a + 6b + 6c = ab + bc + ca + 6abc(a + b + c) ab + bc + ca + 2(ab + bc + ca)2 = 3; do vÂȘy 1 a + 1 b + 1 c + 6a + 6b + 6c 3 abc . Kžt hñp vĂźi ÂĄnh giÂĄ () Ă° tr¶n, ta ֖c r 3 1 a r + 6b + 3 1 b r + 6c + 3 1 c !3 + 6a 27 abc ; tĂž Ăą ta l§y c«n bÂȘc ba hai vž Âș thu ֖c b§t ÂŻng thĂčc cŠn chĂčng minh. B i toÂĄn kžt thĂłc.2 1.38 Cho a; b; c l  cÂĄc sĂš thĂŒc dĂ·ĂŹng thoÂŁ mÂąn abc = 1. ChĂčng minh rÂŹng: 1 1 + a + b + 1 1 + b + c + 1 1 + a + c 1 LĂ­i giÂŁi. °t x = p a; y = p b; z = p c. Khi Ăą ta phÂŁi chĂčng minh 1 x2 + y2 + 1 + 1 y2 + z2 + 1 + 1 z2 + x2 + 1 1 vĂźi x; y; z 0 v  xyz = 1. Do 1 x2 + y2 + 1 = 1 x2 + y2 x2 + y2 + 1 = 1 (x + y)2 + (x y)2 2(x2 + y2 + 1) n¶n b§t ÂŻng thĂčc n y cĂą thÂș ֖c vižt l€i th nh X (x + y)2 x2 + y2 + 1 + X (x y)2 x2 + y2 + 1 4: KhĂŠng m§t tÂœnh tĂȘng quÂĄt, ta giÂŁ sĂ» x y z. SĂ» döng b§t ÂŻng thĂčc Cauchy - Schwarz, ta cĂą X (x + y)2 x2 + y2 + 1 [(x + y) + (y + z) + (z + x)]2 X (x2 + y2 + 1) ; v  X (x y)2 x2 + y2 + 1 [x y + y z + x z]2 X (x2 + y2 + 1) : TĂž „y ta Ă·a b i toÂĄn v· chĂčng minh (x + y + z)2 + (x z)2 2(x2 + y2 + z2) + 3: M°t khÂĄc, theo b§t ÂŻng thĂčc AM-GM, ta l€i cĂą 3 = 3 3 p x2y2z2 xy + yz + zx; do vÂȘy ta chÂż cĂĄn phÂŁi chĂčng minh (x + y + z)2 + (x z)2 2(x2 + y2 + z2) + xy + yz + zx: 37
  • 38. Sau khi thu gĂ„n, ta ֖c b§t ÂŻng thĂčc hiÂșn nhi¶n Ăłng (x y)(y z) 0: B i toÂĄn do Ăą ֖c chĂčng minh xong.2 1.39 Cho a; b; c l  cÂĄc sĂš thĂŒc dĂ·ĂŹng thoÂŁ mÂąn abc = 1. ChĂčng minh rÂŹng: a3 b(c + 2) + b3 c(a + 2) + c3 a(b + 2) 1 LĂ­i giÂŁi. p döng b§t ÂŻng thĂčc AM-GM, ta cĂą a3 b(c + 2) + b 3 + c + 2 9 a: LÂȘp hai b§t ÂŻng thĂčc tĂ·ĂŹng tĂŒ v  cĂ«ng l€i, ta ֖c a3 b(c + 2) + b3 c(a + 2) + c3 a(b + 2) + a + b + c 3 + a + b + c + 6 9 a + b + c; hay a3 b(c + 2) + b3 c(a + 2) + c3 a(b + 2) 5(a + b + c) 9 2 3 : M°t khÂĄc cĂŽng theo b§t ÂŻng thĂčc AM-GM thÂŒ a + b + c 3 3 p abc = 3, do vÂȘy a3 b(c + 2) + b3 c(a + 2) + c3 a(b + 2) 5 3 2 3 = 1: PhÂČp chĂčng minh ho n t§t.2 1.40 Cho a; b; c l  cÂĄc sĂš thĂŒc khĂŠng „m thay ĂȘi b§t kÂŒ. ChĂčng minh rÂŹng: a3 + b3 + c3 + 3abc ab(a + b) + bc(b + c) + ca(c + a) LĂ­i giÂŁi. B§t ÂŻng thĂčc ban Šu mang tÂœnh Ăši xĂčng giĂșa cÂĄc bižn, n¶n khĂŠng m§t tÂœnh tĂȘng quÂĄt, ta giÂŁ sĂ» a = maxfa; b; cg. Khi Ăą thĂŒc hi»n bižn ĂȘi tĂ·ĂŹng Ă·ĂŹng, ta thu ֖c dÂąy b§t ÂŻng thĂčc tĂ·ĂŹng Ă·ĂŹng vĂźi b§t ÂŻng thĂčc cŠn chĂčng minh a(a b)(a c) + b(b a)(b c) + c(c a)(c b) 0; (a b)(a2 ac b2 + bc) + c(a c)(b c) 0; (a b)2(a + b c) + c(a c)(b c) 0: ÂĄnh giÂĄ cuĂši cĂČng Ăłng do a = maxfa; b; cg, do vÂȘy b§t ÂŻng thĂčc ban Šu ֖c chĂčng minh xong. B i toÂĄn kžt thĂłc.2 38
  • 39. 3.2 B i 2.1 žn b i 2.40 2.1 Cho a; b; c l  cÂĄc sĂš thĂŒc dĂ·ĂŹng thay ĂȘi b§t kÂŒ. ChĂčng minh rÂŹng: a (b + c)2 + b (a + c)2 + c (a + b)2 9 4(a + b + c) LĂ­i giÂŁi. B§t ÂŻng thĂčc ban Šu tĂ·ĂŹng Ă·ĂŹng vĂźi (a + b + c) a (b + c)2 + b (a + c)2 + c (a + b)2 9 4 : °t k = a b + c + b a + c + c a + b . Ta th§y rÂŹng (a+b+c) a (b + c)2 + b (a + c)2 + c (a + b)2 = a2 (b + c)2 + b2 (a + c)2 + c2 (a + b)2 + a b + c + b a + c + c a + b ; v  theo mĂ«t ÂĄnh giÂĄ quen thuĂ«c thÂŒ a2 (b + c)2 + b2 (a + c)2 + c2 (a + b)2 k2 3 , do vÂȘy (a + b + c) a (b + c)2 + b (a + c)2 + c (a + b)2 k2 3 + k Ta l€i cĂą chĂł ĂŸ rÂŹng k 3 2 theo b§t ÂŻng thĂčc Nesbitt, do Ăą (a + b + c) a (b + c)2 + b (a + c)2 + c (a + b)2 9 4:3 + 3 2 = 9 4 : PhÂČp chĂčng minh ho n t§t.2 2.2 Cho a; b; c l  cÂĄc sĂš thĂŒc dĂ·ĂŹng thay ĂȘi b§t kÂŒ. ChĂčng minh rÂŹng: a p a2 + 8bc + b p b2 + 8ac + c p c2 + 8ab 1 LĂ­i giÂŁi. p döng b§t ÂŻng thĂčc Cauchy-Schwartz ta cĂą a p a2 + 8bc + b p b2 + 8ac + c p c2 + 8ab (a + b + c)2 p a2 + 8bc + b a p b2 + 8ac + c p c2 + 8ab : M°t khÂĄc, cĂŽng theo b§t ÂŻng thĂčc Cauchy - Schwarz, ta cĂą p a2 + 8bc + b a p b2 + 8ac + c p c2 + 8ab = p a3 + 8abc + p a p b p b3 + 8abc + p c p c3 + 8abc p (a + b + c)(a3 + b3 + c3 + 24abc); do vÂȘy a p a2 + 8bc + b p b2 + 8ac + c p c2 + 8ab (a + b + c)2 p (a + b + c)(a3 + b3 + c3 + 24abc) = s (a + b + c)3 a3 + b3 + c3 + 24abc : 39
  • 40. NhĂ· vÂȘy, Âș kžt thĂłc chĂčng minh, ta cŠn chÂż ra rÂŹng (a + b + c)3 a3 + b3 + c3 + 24abc; hay (a + b)(b + c)(c + a) 8abc. Tuy nhi¶n „y l  mĂ«t ÂĄnh giÂĄ Ăłng vÂŒ theo b§t ÂŻng thĂčc AM-GM, ta cĂą p ab:2 (a + b)(b + c)(c + a) 2 p bc:2 p ca = 8abc; do vÂȘy b§t ÂŻng thĂčc ban Šu ֖c chĂčng minh xong. B i toÂĄn kžt thĂłc.2 2.3 Cho a; b; c l  cÂĄc sĂš thĂŒc dĂ·ĂŹng thoÂŁ mÂąn çng thĂ­i c a v  3a2+4b2+5c2 = 12. ChĂčng minh rÂŹng: 1 a + 1 b + 1 c 3 LĂ­i giÂŁi. TĂž giÂŁ thižt, ta cĂą 4a2 + 4b2 + 4c2 = 12 + a2 c2 12; nhĂ· vÂȘy a2 + b2 + c2 3. TĂž „y ta cĂŽng cĂą a + b + c p 3(a2 + b2 + c2) 3; v  vÂŒ vÂȘy ta chĂčng minh ֖c b§t ÂŻng thĂčc ban Šu vÂŒ 1 a + 1 b + 1 c 9 a + b + c 9 3 = 3: B i toÂĄn kžt thĂłc.2 2.4 Cho a; b; c l  cÂĄc sĂš thĂŒc dĂ·ĂŹng thay ĂȘi b§t kÂŒ. ChĂčng minh rÂŹng: a b + b c + c a a + c b + c + b + a c + a + c + b a + b LĂ­i giÂŁi 1. °t X = a b 2 1 + ; Y = 1 + b c 2 ; Z = 1 + c a 2 : SĂ» döng b§t ÂŻng thĂčc Holder, ta thu ֖c 1 + a b 1 + b c 1 + c a 1 + 3 r a b : b c : c a !3 = 8; tĂž Ăą ta suy ra XY Z 1. B„y giĂ­ ta thĂŒc hi»n bižn ĂȘi b§t ÂŻng thĂčc Âą cho nhĂ· sau a b a + c b + c + b c b + a c + a + c a c + b a + b 0; c(a b) b(b + c) + a(b c) c(c + a) + b(c a) a(a + b) 0; 40
  • 41. a b 1 1 + b c + b c 1 1 + c a + c a 1 1 + a b 0: Âș ĂŸ rÂŹng a b 1 1 + b c = 2X 1 1 2Y = X 1 Y ; do vÂȘy b§t ÂŻng thĂčc cuĂši cĂą thÂș vižt l€i th nh X 1 Y + Y 1 Z + Z 1 X 0; tĂ·ĂŹng Ă·ĂŹng X Y + Y Z + Z X 1 X + 1 Y + 1 Z : SĂ» döng b§t ÂŻng thĂčc AM-GM, ta cĂą 3 XX Y = X X Y + X Y + Z X 3 X r 3 ZX Y 2 = 3 3 p XY Z X 1 Y = 3 X 1 Y : NhĂ· vÂȘy b§t ÂŻng thĂčc ban Šu ֖c chĂčng minh xong. B i toÂĄn kžt thĂłc.2 LĂ­i giÂŁi 2. ThĂŒc hi»n bižn ĂȘi tĂ·ĂŹng tĂŒ nhĂ· cÂĄch 1, ta cŠn chĂčng minh c(a b) b(b + c) + a(b c) c(c + a) + b(c a) a(a + b) 0: KhĂŠng m§t tÂœnh tĂȘng quÂĄt, ta giÂŁ sĂ» b l  sĂš nÂŹm giĂșa a v  c. Khi Ăą (ba)(bc) 0. Âș ĂŸ rÂŹng b(c a) = c(a b) a(b c); vÂŒ vÂȘy b§t ÂŻng thĂčc tr¶n cĂą thÂș vižt l€i th nh c(a b) 1 b(b + c) 1 a(a + b) + a(b c) 1 c(c + a) 1 a(a + b) 0; tĂ·ĂŹng Ă·ĂŹng c[(a b)2(a + b) + b(a b)(a c)] ab(a + b)(b + c) + [(b c)(a c)(a + c) + a(b c)2] c(c + a)(a + b) 0: B§t ÂŻng thĂčc cuĂši n y Ăłng do (a b)(a c) = (a b)2 (b a)(b c) 0; v  (b c)(a c) = (b c)2 (b a)(b c) 0; do vÂȘy b§t ÂŻng thĂčc ban Šu ֖c chĂčng minh xong. B i toÂĄn kžt thĂłc.2 NhÂȘn xÂČt. 41
  • 42. 1. LĂ·u ĂŸ rÂŹng b§t ÂŻng thĂčc sau Ăłng vĂźi a; b; c v  k l  cÂĄc sĂš thĂŒc dĂ·ĂŹng: a b + b c + c a ka + c kb + c + kb + c kc + a + kc + b ka + b : VĂźi k = 1, ta thu ֖c b i toÂĄn tr¶n. 2. Ri¶ng vĂźi trĂ·Ă­ng hñp k = 1, ta cĂą thÂș chĂčng minh b i toÂĄn dĂŒa tr¶n b§t ÂŻng thĂčc sau („y l  mĂ«t b i trong Belarusian Mathematical Olympiad 1998): Cho a; b; c l  cÂĄc sĂš thĂŒc dĂ·ĂŹng. ChĂčng minh rÂŹng a b + b c + c a a + b b + c + b + c a + b + 1: Vi»c chĂčng minh cĂŽng nhĂ· ÂĄp döng xin Âș d nh cho b€n Ă„c. 2.5 Cho a; b; c l  cÂĄc sĂš thĂŒc dĂ·ĂŹng thoÂŁ mÂąn a + b + c = 3. ChĂčng minh rÂŹng: p a + p b + p c ab + bc + ca LĂ­i giÂŁi. B§t ÂŻng thĂčc Âą cho tĂ·ĂŹng Ă·ĂŹng vĂźi a2 + b2 + c2 + 2 p a + 2 p b + 2 p c a2 + b2 + c2 + 2(ab + bc + ca) = 9: p döng b§t ÂŻng thĂčc AM-GM, ta cĂą a2 + 2 p a 3a: LÂȘp cÂĄc b§t ÂŻng thĂčc tĂ·ĂŹng tĂŒ v  cĂ«ng l€i, ta ֖c a2 + b2 + c2 + 2 p a + 2 p b + 2 p c 3(a + b + c) = 9: PhÂČp chĂčng minh ho n t§t.2 2.6 Cho a; b; c l  cÂĄc sĂš thĂŒc dĂ·ĂŹng thoÂŁ mÂąn ab + bc + ca = 1. ChĂčng minh rÂŹng: 1 abc + 4 (a + b)(b + c)(c + a) p 3 2 9 LĂ­i giÂŁi. p döng b§t ÂŻng thĂčc AM-GM, ta cĂą 1 abc + 4 (a + b)(b + c)(c + a) = 1 2abc + 1 2abc + 4 (a + b)(b + c)(c + a) s 3 3 1 a2b2c2(a + b)(b + c)(c + a) s = 3 3 1 abc(ab + ac)(bc + ba)(ca + cb) : M°t khÂĄc, cĂŽng theo b§t ÂŻng thĂčc AM-GM, ta cĂą hai ÂĄnh giÂĄ: a2b2c2 (ab + bc + ca)3 27 ; v  (ab + bc)(bc + ca)(ca + ab) 8(ab + bc + ca)3 27 ; 42
  • 43. tĂž Ăą sĂ» döng giÂŁ thižt ta suy ra abc 1 3 p 3 v  (ab + bc)(bc + ca)(ca + ab) 8 27 . Do vÂȘy 1 abc + 4 (a + b)(b + c)(c + a) 3 s 3 p 3 8 27:3 = p 3 2 9 : PhÂČp chĂčng minh ho n t§t.2 2.7 Cho x; y; z l  cÂĄc sĂš thĂŒc thoÂŁ mÂąn x + y + z = 0, trong Ăą cĂą hai sĂš cĂČng d§u. ChĂčng minh rÂŹng: (x2 + y2 + z2)3 (x3 + y3 + z3)2 6 LĂ­i giÂŁi. KhĂŠng m§t tÂœnh tĂȘng quÂĄt, giÂŁ sĂ» x; y l  hai sĂš cĂČng d§u, tĂčc l  xy 0. VĂźi i·u ki»n z = x y, ta cĂą (x2 + y2 + z2)3 (x3 + y3 + z3)2 = 8(x2 + y2 + xy)3 9x2y2(x + y)2 : NhĂ· vÂȘy, nžu ta °t x2 + y2 = m v  xy = n (Âș ĂŸ rÂŹng m 2n) thÂŒ ta cŠn chĂčng minh 8(m + n)3 9n2(m + 2n) 6; hay 4m3 + 4n3 + 12m2n + 12n2m 27n2m + 54n3: B§t ÂŻng thĂčc tr¶n mang tÂœnh thuŠn nh§t giĂșa cÂĄc bižn, do Ăą ta cho n = 1, lĂłc n y m 2 v  ta cŠn chĂčng minh 4m3 + 12m2 15m 50 0: Tuy nhi¶n bÂŹng bižn ĂȘi tĂ·ĂŹng Ă·ĂŹng, ta ֖c (m2) m 5 2 2 0. „y l  mĂ«t ÂĄnh giÂĄ Ăłng do m 2, do vÂȘy b§t ÂŻng thĂčc ban Šu ֖c chĂčng minh xong. B i toÂĄn kžt thĂłc.2 2.8 Cho a; b; c l  cÂĄc sĂš thĂŒc dĂ·ĂŹng thay ĂȘi trong o€n [0; 1]. ChĂčng minh rÂŹng: p abc + p (1 a)(1 b)(1 c) 1 LĂ­i giÂŁi. p döng b§t ÂŻng thĂčc Cauchy - Schwarz ta cĂą p abc + p (1 a)(1 b)(1 c) p (a + 1 a)[bc + (1 b)(1 c)] = p 2bc b c + 1: NhĂ· vÂȘy, Âș kžt thĂłc chĂčng minh, ta cŠn chÂż ra rÂŹng 2bc b + c: Tuy nhi¶n „y l  mĂ«t ÂĄnh giÂĄ Ăłng vÂŒ theo giÂŁ thižt v  b§t ÂŻng thĂčc AM-GM thÂŒ p bc b + c; 2bc 2 do Ăą b§t ÂŻng thĂčc ban Šu ֖c chĂčng minh xong. 43
  • 44. B i toÂĄn kžt thĂłc.2 2.9 Cho a; b; c l  cÂĄc sĂš thĂŒc khĂŠng „m thay ĂȘi b§t kÂŒ. ChĂčng minh rÂŹng: p (ab + bc + ca) 2 p 3: 3 p (a + b)(b + c)(c + a) LĂ­i giÂŁi. Âș ĂŸ rÂŹng ta cĂą ÂŻng thĂčc (a + b)(b + c)(c + a) = (a + b + c)(ab + bc + ca) abc: M°t khÂĄc, theo cÂĄc ÂĄnh giÂĄ quen thuĂ«c, ta cĂą a + b + c p 3(ab + bc + ca); v  abc r (ab + bc + ca)3 27 ; do vÂȘy p 3(ab + bc + ca) (a + b)(b + c)(c + a) (ab + bc + ca) r (ab + bc + ca)3 27 = p (ab + bc + ca)3 8 p 3 3 : TĂž „y, l§y c«n bÂȘc ba hai vž, ta thu ֖c b§t ÂŻng thĂčc cŠn chĂčng minh. B i toÂĄn kžt thĂłc.2 2.10 Cho a; b; c l  cÂĄc sĂš thĂŒc ĂŠi mĂ«t ph„n bi»t. ChĂčng minh rÂŹng: a2 + b2 a2 2ab + b2 + a2 + c2 a2 2ac + c2 + b2 + c2 b2 2bc + c2 5 2 LĂ­i giÂŁi. DÂąy b§t ÂŻng thĂčc sau tĂ·ĂŹng Ă·ĂŹng vĂźi b§t ÂŻng thĂčc cŠn chĂčng minh (a + b)2 + (a b)2 (a b)2 + (b + c)2 + (b c)2 (b c)2 + (c + a)2 + (c a)2 (c a)2 5; a + b a b 2 + b + c b c 2 + c + a c a 2 2: °t x = a + b a b ; y = b + c b c ; z = c + a c a . Âș ĂŸ rÂŹng ta cĂą ÂŻng thĂčc xy + yz + zx = (a + b)(b + c) (a b)(b c) + (b + c)(c + a) (b c)(c a) + (c + a)(a + b) (c a)(a b) = (a + b)(b + c)(c a) + (b + c)(c + a)(a b) + (c + a)(a + b)(b c) (a b)(b c)(c a) = 1 HĂŹn nĂșa, ta cĂŽng cĂą (x + y + z)2 0, do vÂȘy x2 + y2 + z2 2(xy + yz + zx) = 2: TĂž „y ta thu ֖c b§t ÂŻng thĂčc cŠn chĂčng minh. 44
  • 45. B i toÂĄn kžt thĂłc.2 2.11 Cho a; b l  cÂĄc sĂš thĂŒc khĂŠng „m thoÂŁ mÂąn a + b 4 5 . ChĂčng minh rÂŹng: r 1 a 1 + a + r 1 b 1 + b 1 r 1 a b 1 + a + b LĂ­i giÂŁi. DÂąy b§t ÂŻng thĂčc sau l  tĂ·ĂŹng Ă·ĂŹng vĂźi b§t ÂŻng thĂčc cŠn chĂčng minh 1 a 1 + a + 1 b 1 + b + 2 s (1 a)(1 b) (1 + a)(1 + b) 1 a b 1 + a + b r + 1 + 2 1 a b 1 + a + b ; 2(1 ab) 1 + ab + a + b + 2 r 1 + ab a b 1 + ab + a + b 2 1 + a + b + 2 r 1 a b 1 + a + b : °t u = ab; v = a + b. Khi Ăą u; v 0 v  ta cŠn chĂčng minh 2(1 u) 1 + u + v + 2 r 1 + u v 1 + u + v 2 1 + v + 2 r 1 v 1 + v : ThĂŒc hi»n bižn ĂȘi tĂ·ĂŹng Ă·ĂŹng, ta ֖c dÂąy b§t ÂŻng thĂčc sau 1 + u v 1 + u + v 1 v 1 + v u(2 + v) (1 + v)(1 + v + u) r 1 + u v 1 + u + v + r 1 v 1 + v ! ; 2uv (1 + u + v)(1 + v) u(2 + v) (1 + v)(1 + v + u) r 1 + u v 1 + u + v + r 1 v 1 + v ! : Nžu u = 0 thÂŒ b§t ÂŻng thĂčc tr¶n hiÂșn nhi¶n Ăłng. Nžu u 0, b§t ÂŻng thĂčc tr¶n tĂ·ĂŹng Ă·ĂŹng vĂźi 2v 2 + v r 1 + u v 1 + u + v + r 1 v 1 + v : () Âș ĂŸ rÂŹng vĂźi u 0, ta cĂą ÂĄnh giÂĄ 1 + u v 1 + u + v 1 v 1 + v ; do vÂȘy r 1 + u v 1 + u + v + r 1 v 1 + v r 2 1 v 1 + v r 1 + = 2 2 1 + v : HĂŹn nĂșa, ta l€i cĂą v 4 5 theo giÂŁ thižt n¶n r 1 + u v 1 + u + v + r 1 v 1 + v s 1 + 2 2 1 + 4 5 = 2 3 : Ngo i ra cĂŽng do v 4 5 1 n¶n 2v 2 + v = 2 2 v + 1 2 3 ; do vÂȘy ÂĄnh giÂĄ () Ăłng, cĂŽng cĂą nghŸa b§t ÂŻng thĂčc ban Šu ֖c chĂčng minh. 45
  • 46. B i toÂĄn ho n t§t.2 2.12 Cho a; b; c l  cÂĄc sĂš thĂŒc dĂ·ĂŹng thay ĂȘi b§t kÂŒ. ChĂčng minh rÂŹng: a2 b + b2 c + c2 a + a + b + c 6(a2 + b2 + c2) a + b + c LĂ­i giÂŁi. B§t ÂŻng thĂčc cŠn chĂčng minh mang tÂœnh hoÂĄn vĂ  giĂșa cÂĄc bižn, do Ăą khĂŠng m§t tÂœnh tĂȘng quÂĄt, ta giÂŁ sĂ» b l  sĂš h€ng nÂŹm giĂșa a v  c. Khi Ăą ta bižn ĂȘi b§t ÂŻng thĂčc nhĂ· sau X a2 b + b 2a 6(a2 + b2 + c2) a + b + c 2(a + b + c); X(a b)2 b 6(a2 + b2 + c2) a + b + c 2(a + b + c): p döng b§t ÂŻng thĂčc Cauchy - Schwarz, ta cĂą X(a b)2 b [(a b) + (b c) + (a c)]2 b + c + a = 4(a c)2 a + b + c : Do Ăą ta chÂż cŠn chĂčng minh ֖c 2(a c)2 3(a2 + b2 + c2) (a + b + c)2: Sau khi thu gĂ„n, ta ֖c b§t ÂŻng thĂčc hiÂșn nhi¶n Ăłng do b nÂŹm giĂșa a v  c 2(b c)(b a) 0: B i toÂĄn ho n t§t.2 2.13 Cho x; y; z l  cÂĄc sĂš thĂŒc thoÂŁ mÂąn x2 + y2 + z2 = 1. ChĂčng minh rÂŹng: 1 x3 + y3 + z3 3xyz 1 LĂ­i giÂŁi 1. ChĂł ĂŸ rÂŹng ta cĂą ÂŻng thĂčc (x3 + y3 + z3 3xyz)2 = (x + y + z)2(x2 + y2 + z2 xy yz zx)2 = (1 + 2t)(1 t)(1 t); trong Ăą t = xy + yz + zx. žn „y ta ÂĄp döng b§t ÂŻng thĂčc AM-GM Âș cĂą (x3 + y3 + z3 3xyz)2 [(1 + 2t) + (1 t) + (1 t)]3 27 = 1; do vÂȘy 1 x3 + y3 + z3 3xyz 1. PhÂČp chĂčng minh ho n t§t.2 LĂ­i giÂŁi 2. p döng b§t ÂŻng thĂčc Cauchy - Schwarz, ta cĂą (x3 + y3 + z3 3xyz)2 = [x(x2 yz) + y(y2 zx) + z(z2 xy)]2 (x2 + y2 + z2)[(x2 yz)2 + (y2 zx)2 + (z2 xy)2]: 46
  • 47. HĂŹn nĂșa, ta l€i cĂą (x2 yz)2 + (y2 zx)2 + (z2 xy)2 = (x2 + y2 + z2)2 (xy + yz + zx)2 (x2 + y2 + z2)2; do vÂȘy (x3 + y3 + z3 3xyz)2 (x2 + y2 + z2)3 = 1: TĂž Ăą ta suy ra 1 x3 + y3 + z3 3xyz 1. PhÂČp chĂčng minh ho n t§t.2 2.14 Cho x; y; z l  cÂĄc sĂš thĂŒc dĂ·ĂŹng thay ĂȘi b§t kÂŒ. ChĂčng minh rÂŹng: xyz (1 + 3x)(z + 6)(x + 8y)(y + 9z) 1 74 LĂ­i giÂŁi. p döng b§t ÂŻng thĂčc AM-GM, ta cĂą cÂĄc ÂĄnh giÂĄ sau: z + 6 = z + 1 + 1 + 1 + 1 + 1 + 1 7 7 p z; 1 + 3x = 1 + x 2 + x 2 + x 2 + x 2 + x 2 + x 2 r x6 7 7 26 ; x + 8y = x + 4y 3 + 4y 3 + 4y 3 + 4y 3 + 4y 3 + 4y 3 r xy6: 7 7 46 36 ; y + 9z = y + 3z 2 + 3z 2 + 3z 2 + 3z 2 + 3z 2 + 3z 2 r yz6: 7 7 36 26 : Nh„n cÂĄc b§t ÂŻng thĂčc tr¶n vĂźi nhau, ta ֖c r z: (z + 6)(1 + 3x)(x + 8y)(y + 9z) 74 7 x6 26 :xy6: 46 36 :yz6: 36 26 = 74xyz; tĂž Ăą suy ra xyz (1 + 3x)(z + 6)(x + 8y)(y + 9z) 1 74 : PhÂČp chĂčng minh ho n t§t.2 2.15 Cho a; b; c l  cÂĄc sĂš thĂŒc dĂ·ĂŹng thay ĂȘi b§t kÂŒ. ChĂčng minh rÂŹng: a + b ab + c2 + b + c bc + a2 + a + c ac + b2 1 a + 1 b + 1 c LĂ­i giÂŁi. p döng b§t ÂŻng thĂčc Cauchy - Schwarz, ta cĂą a2 b(a2 + c2) + b2 a(b2 + c2) (a + b)2 b(a2 + c2) + a(b2 + c2) = (a + b)2 (a + b)(ab + c2) ; tĂž „y ta suy ra a + b ab + c2 a2 b(a2 + c2) + b2 a(b2 + c2) . Thižt lÂȘp hai b§t ÂŻng thĂčc tĂ·ĂŹng tĂŒ rçi cĂ«ng l€i, ta ֖c a + b ab + c2 + b + c bc + a2 + a + c ac + b2 a2 b(a2 + c2) + b2 a(b2 + c2) + b2 c(b2 + a2) + c2 b(a2 + c2) + a2 c(a2 + b2) + c2 a(b2 + c2) = 1 a + 1 b + 1 c : 47
  • 48. PhÂČp chĂčng minh ho n t§t.2 2.16 Cho a; b; c l  cÂĄc sĂš thĂŒc khĂŠng „m thoÂŁ mÂąn khĂŠng cĂą b§t kÂŒ hai sĂš n o çng thĂ­i bÂŹng 0. ChĂčng minh rÂŹng: a(b + c) b2 + bc + c2 + b(a + c) a2 + ac + c2 + c(a + b) a2 + ab + b2 2 LĂ­i giÂŁi. p döng b§t ÂŻng thĂčc Cauchy - Schwarz, ta cĂą a(b + c) b2 + bc + c2 + b(a + c) a2 + ac + c2 + c(a + b) a2 + ab + b2 = a2 a(b + c) abc b + c + b2 b(a + c) abc a+c + c2 c(a + b) abc a+b (a + b + c)2 2(ab + bc + ca) abc b + c abc c + a abc a + b : NhĂ· vÂȘy, Âș kžt thĂłc chĂčng minh, ta cŠn chÂż ra rÂŹng (a + b + c)2 4(ab + bc + ca) 2abc:( 1 a + b + 1 b + c + 1 a + c ); hay a2 + b2 + c2 + 2abc:( 1 a + b + 1 b + c + 1 a + c ) 2(ab + bc + ca): p döng ÂĄnh giÂĄ cĂŹ bÂŁn 1 x + 1 y + 1 z 9 x + y + z , ta cĂą a2 + b2 + c2 + 2abc:( 1 a + b + 1 b + c + 1 a + c ) a2 + b2 + c2 + 9abc a + b + c : CĂŠng vi»c cuĂši cĂČng chÂż cŠn chĂčng minh a2 + b2 + c2 + 9abc a + b + c 2(ab + bc + ca); hay a3 + b3 + c3 + 3abc ab(a + b) + bc(b + c) + ca(c + a). Tuy nhi¶n ÂĄnh giÂĄ n y Ăłng theo b§t ÂŻng thĂčc Schur bÂȘc ba n¶n b§t ÂŻng thĂčc ban Šu ֖c chĂčng minh xong. B i toÂĄn kžt thĂłc.2 2.17 Cho a; b; c l  cÂĄc sĂš thĂŒc dĂ·ĂŹng thoÂŁ mÂąn a2 + b2 + c2 = 1. ChĂčng minh rÂŹng: a b2 + c2 + b a2 + c2 + c a2 + b2 p 3 2 3 LĂ­i giÂŁi. SĂ» döng giÂŁ thižt, ta cĂą a b2 + c2 + b a2 + c2 + c a2 + b2 = a 1 a2 + b 1 b2 + c 1 c2 : Âș ĂŸ rÂŹng ta cĂą ÂĄnh giÂĄ a 1 a2 p 3 2 3 a2 = p 3 + 2)(a a(a p 3 1)2 2(1 a2) 0; 48
  • 49. do vÂȘy a 1 a2 p 3 2 3 a2. Thižt lÂȘp hai ÂĄnh giÂĄ tĂ·ĂŹng tĂŒ v  cĂ«ng l€i, ta ֖c a 1 a2 + b 1 b2 + c 1 c2 p 3 2 3 (a2 + b2 + c2); do vÂȘy a b2 + c2 + b a2 + c2 + c a2 + b2 p 3 2 3 : PhÂČp chĂčng minh ho n t§t.2 2.18 Cho x; y; z l  cÂĄc sĂš thĂŒc dĂ·ĂŹng thay ĂȘi b§t kÂŒ. ChĂčng minh rÂŹng: p x + y + z p x y + z + p y x + z + p z x + y p 3 2 3 LĂ­i giÂŁi. B§t ÂŻng thĂčc cŠn chĂčng minh mang tÂœnh thuŠn nh§t, do Ăą ta chu©n hĂąa x+y+z = 1. p p p çng thĂ­i, ta °t a = x; b = y; c = z. NhĂ· vÂȘy ta cŠn chĂčng minh a 1 a2 + b 1 b2 + c 1 c2 p 3 2 3 : Tuy nhi¶n „y l  mĂ«t kžt quÂŁ Âą ֖c chĂčng minh Ă° b i 2.17 . 2 2.19 Cho a; b; c l  cÂĄc sĂš thĂŒc khĂŠng „m thoÂŁ mÂąn khĂŠng cĂą b§t kÂŒ hai sĂš n o çng thĂ­i bÂŹng 0. ChĂčng minh rÂŹng: a3 (2a2 + b2)(2a2 + c2) + b3 (2b2 + a2)(2b2 + c2) + c3 (2c2 + a2)(2c2 + b2) 1 a + b + c LĂ­i giÂŁi. p döng b§t ÂŻng thĂčc Cauchy-Schwartz, ta cĂą (2a2 + b2)(2a2 + c2) = (a2 + a2 + b2)(a2 + c2 + a2) (a2 + ab + ac)2 = a2(a + b + c)2: NhĂ· vÂȘy a3 (2a2 + b2)(2a2 + c2) a (a + b + c)2 . Thižt lÂȘp hai ÂĄnh giÂĄ tĂ·ĂŹng tĂŒ rçi cĂ«ng l€i, ta ֖c a3 (2a2 + b2)(2a2 + c2) + b3 (2b2 + a2)(2b2 + c2) + c3 (2c2 + a2)(2c2 + b2) a + b + c (a + b + c)2 = 1 a + b + c : PhÂČp chĂčng minh ho n t§t.2 2.20 Cho a; b; c l  cÂĄc sĂš thĂŒc khĂŠng „m thay ĂȘi b§t kÂŒ. ChĂčng minh rÂŹng: a2 + b2 a + b + b2 + c2 b + c + c2 + a2 c + a 3(a2 + b2 + c2) a + b + c LĂ­i giÂŁi. DÂąy b§t ÂŻng thĂčc sau tĂ·ĂŹng Ă·ĂŹng vĂźi b§t ÂŻng thĂčc cŠn chĂčng minh 2(a2 + b2 + c2) + c(a2 + b2) a + b + a(b2 + c2) b + c + b(c2 + a2) c + a 3(a2 + b2 + c2); c[(a + b)2 2ab] a + b + a[(b + c)2 2bc] b + c + b[(c + a)2 2ca]) c + a a2 + b2 + c2; 2ab + 2bc + 2ca a2 + b2 + c2 + 2abc 1 a + b + 1 b + c + 1 a + c : 49
  • 50. ÂĄnh giÂĄ cuĂši cĂČng l  mĂ«t kžt quÂŁ Âą ֖c chĂčng minh Ă° b i 2.16 , do vÂȘy ta kžt thĂłc chĂčng minh.2 2.21 Cho x; y; z l  cÂĄc sĂš thĂŒc dĂ·ĂŹng thoÂŁ mÂąn x + y + z = 1. ChĂčng minh rÂŹng: xy 1 + z + yz 1 + x + xz 1 + y 1 4 LĂ­i giÂŁi. ChĂł ĂŸ rÂŹng xy 1 + z = xy (x + z) + (y + z) ; v  theo mĂ«t ÂĄnh giÂĄ quen thuĂ«c thÂŒ 4 (x + z) + (y + z) 1 x + z + 1 y + z ; do vÂȘy xy 1 + z 1 4 xy x + z + xy y + z . Thižt lÂȘp hai ÂĄnh giÂĄ tĂ·ĂŹng tĂŒ rçi cĂ«ng l€i, ta ֖c xy 1 + z + yz 1 + x + xz 1 + y 1 4 xy + yz x + z + yz + zx x + y + zx + xy y + z = x + y + z 4 ; tĂž „y ta thu ֖c b§t ÂŻng thĂčc cŠn chĂčng minh. B i toÂĄn kžt thĂłc.2 2.22 Cho a; b; c l  cÂĄc sĂš thĂŒc dĂ·ĂŹng thay ĂȘi b§t kÂŒ. ChĂčng minh rÂŹng: ( a b + b c + c a p 3(a2 + b2 + c2) )(a + b + c) 3 LĂ­i giÂŁi. p döng b§t ÂŻng thĂčc Cauchy-Schwartz, ta cĂą a b + b c + c a (a + b + c)2 ab + bc + ca ; do vÂȘy ( a b + b c + c a )(a+b+c) (a + b + c)3 ab + bc + ca . NhĂ· vÂȘy, Âș kžt thĂłc chĂčng minh, ta cŠn chÂż ra rÂŹng (a + b + c)3 3(ab + bc + ca) p 3(a2 + b2 + c2); hay (a+b+c)6 27(a2 +b2 +c2)(ab+bc+ca)2. Tuy nhi¶n „y l  mĂ«t ÂĄnh giÂĄ Ăłng vÂŒ theo b§t ÂŻng thĂčc AM-GM, ta cĂą (a + b + c)6 = [(a2 + b2 + c2) + (ab + bc + ca) + (ab + bc + ca)]3 27(a2 + b2 + c2)(ab + bc + ca)2; do vÂȘy b§t ÂŻng thĂčc ban Šu ֖c chĂčng minh xong. B i toÂĄn kžt thĂłc.2 2.23 Cho a; b; c l  cÂĄc sĂš thĂŒc dĂ·ĂŹng thay ĂȘi b§t kÂŒ. ChĂčng minh rÂŹng: a2 b + b2 c + c2 a p 3(a2 + b2 + c2) + a + b + c 2 LĂ­i giÂŁi. p döng b§t ÂŻng thĂčc AM-GM ta ֖c a2 b + b2 c + c2 a r ( + a + b + c 2 a2 b + b2 c + c2 a )(a + b + c): 50
  • 51. NhĂ· vÂȘy, Âș kžt thĂłc chĂčng minh, ta cŠn chÂż ra rÂŹng ( a2 b + b2 c + c2 a )(a + b + c) 3(a2 + b2 + c2): ThÂȘt vÂȘy, ÂĄp döng b§t ÂŻng thĂčc Cauchy-Schwartz, ta cĂą a2 b + b2 c + c2 a (a2 + b2 + c2)2 a2b + b2c + c2a : CĂŠng vi»c cuĂši cĂČng chÂż cŠn chĂčng minh (a + b + c)(a2 + b2 + c2) 3(a2b + b2c + c2a); hay (a3 + ab2) + (b3 + bc2) + (c3 + ca2) 2(a2b + b2c + c2a). Tuy nhi¶n „y l  mĂ«t ÂĄnh giÂĄ Ăłng theo b§t ÂŻng thĂčc AM-GM, do vÂȘy b§t ÂŻng thĂčc ban Šu ֖c chĂčng minh. B i toÂĄn kžt thĂłc.2 2.24 Cho a; b; c l  cÂĄc sĂš thĂŒc dĂ·ĂŹng thuĂ«c khoÂŁng (0; 1) thoÂŁ mÂąn ab + bc + ca = 1. ChĂčng minh rÂŹng: a2 + b2 (1 a2)(1 b2) + b2 + c2 (1 b2)(1 c2) + c2 + a2 (1 a2)(1 c2) 9 2 LĂ­i giÂŁi. B§t ÂŻng thĂčc ban Šu tĂ·ĂŹng Ă·ĂŹng vĂźi mĂ©i b§t ÂŻng thĂčc trong dÂąy sau X a2 + b2 (1 a2)(1 b2) + 1 2 6; X(1 + a2)(1 + b2) (1 a2)(1 b2) 12: Âș ĂŸ rÂŹng ta cĂą (1 a2)(1 b2) (1 ab)2 = (a b)2; do vÂȘy (1 a2)(1 b2) (1 ab)2. M°t khÂĄc, theo b§t ÂŻng thĂčc Cauchy - Schwarz, ta cĂą (1 + a2)(1 + b2) (1 + ab)2; do vÂȘy ta suy ra (1 + a2)(1 + b2) (1 a2)(1 b2) (1 + ab)2 (1 ab)2 : žn „y ta thižt lÂȘp hai ÂĄnh giÂĄ tĂ·ĂŹng tĂŒ v  cĂ«ng l€i Âș cĂą X(1 + a2)(1 + b2) (1 a2)(1 b2) X(1 + ab)2 (1 ab)2 : Ta ÂĄp döng tižp b§t ÂŻng thĂčc AM-GM Âș suy ra X(1 + a2)(1 + b2) (1 a2)(1 b2) s 3 3 (1 + ab)(1 + bc)(1 + ca) (1 ab)(1 bc)(1 ca) 2 : Do vÂȘy, Âș kžt thĂłc chĂčng minh, ta cŠn chÂż ra rÂŹng (1 + ab)(1 + bc)(1 + ca) 8(1 ab)(1 bc)(1 ca): 51
  • 52. °t x = ab; y = bc; z = ca. Khi Ăą x; y; z 0; x + y + z = 1 v  ta cŠn chĂčng minh (1 + x)(1 + y)(1 + z) 8(1 x)(1 y)(1 z); tĂ·ĂŹng Ă·ĂŹng 9xyz 7(xy + yz + zx) 2: Theo mĂ«t kžt quÂŁ Âą ֖c chĂčng minh Ă° b i 2.35 , ta cĂą x2 + y2 + z2 + 9xyz x + y + z 2(xy + yz + zx); tĂž Ăą sĂ» döng giÂŁ thižt x + y + z = 1 Âș suy ra 9xyz 4(xy + yz + zx) 1. CĂŠng vi»c cuĂši cĂČng l  chĂčng minh 4(xy + yz + zx) 1 7(xy + yz + zx) 2; hay xy + yz + zx 1 3 . Tuy nhi¶n „y l  mĂ«t ÂĄnh giÂĄ Ăłng vÂŒ xy + yz + zx (x + y + z)2 3 = 1 3 ; do vÂȘy b§t ÂŻng thĂčc ban Šu ֖c chĂčng minh xong. B i toÂĄn kžt thĂłc.2 2.25 Cho a; b; c l  cÂĄc sĂš thĂŒc dĂ·ĂŹng thay ĂȘi b§t kÂŒ. ChĂčng minh rÂŹng: 1 + a3 + b3 + 3 p 1 + b3 + c3 + 3 p 1 + a3 + c3 3 p 27 + 2(a + b + c)3 3 p LĂ­i giÂŁi. p döng b§t ÂŻng thĂčc Holder, ta cĂą (1 + a3 + b3)[27 + (a + b + c)3 + (a + b + c)3]2 [9 + a(a + b + c)2 + b(a + b + c)2]3; tĂž Ăą ta suy ra 1 + a3 + b3: 3 p [27 + 2(a + b + c)3]2 9 + (a + b)(a + b + c)2: 3 p Thižt lÂȘp hai b§t ÂŻng thĂčc tĂ·ĂŹng tĂŒ v  cĂ«ng l€i, ta ֖c 3 p [27 + 2(a + b + c)3]2( 3 p 1 + a3 + b3 + 3 p 1 + b3 + c3 + 3 p 1 + a3 + c3) 27 + 2(a + b + c)3; tĂž Ăą ta thu ֖c b§t ÂŻng thĂčc cŠn chĂčng minh. B i toÂĄn kžt thĂłc.2 NhÂȘn xÂČt. B§t ÂŻng thĂčc tr¶n l  h» quÂŁ trĂŒc tižp cĂ”a b§t ÂŻng thĂčc Minkowsky mĂ° rĂ«ng: 3 p a3 + b3 + c3 + 3 p d3 + e3 + f3 + 3 p g3 + h3 + k3 3 p (a + d + g)3 + (b + e + h)3 + (c + f + k)3: CÂĄch chĂčng minh tĂ·ĂŹng tĂŒ nhĂ· lĂ­i giÂŁi cĂ”a b i toÂĄn tr¶n. 2.26 Cho a; b; c l  cÂĄc sĂš thĂŒc khĂŠng „m thay ĂȘi b§t kÂŒ. ChĂčng minh rÂŹng: bc (a + b)(a + c) + ca (b + c)(b + a) + ab (c + a)(c + b) 2(a2 + b2 + c2) + ab + bc + ca 2(a2 + b2 + c2) + 2(ab + bc + ca) 52
  • 53. LĂ­i giÂŁi. B§t ÂŻng thĂčc cŠn chĂčng minh tĂ·ĂŹng Ă·ĂŹng vĂźi 1 bc (a + b)(a + c) ca (b + c)(b + a) ab (c + a)(c + b) 1 2(a2 + b2 + c2) + ab + bc + ca 2(a2 + b2 + c2) + 2(ab + bc + ca) : M°t khÂĄc, Âș ĂŸ rÂŹng ta cĂą cÂĄc ÂŻng thĂčc sau: 1 bc (a + b)(a + c) ca (b + c)(b + a) ab (c + a)(c + b) = 2abc (a + b)(b + c)(c + a) ; 1 2(a2 + b2 + c2) + ab + bc + ca 2(a2 + b2 + c2) + 2(ab + bc + ca) = ab + bc + ca (a + b)2 + (b + c)2 + (c + a)2 ; do Ăą ta cŠn chĂčng minh 2abc (a + b)(b + c)(c + a) ab + bc + ca (a + b)2 + (b + c)2 + (c + a)2 ; hay 2(a + b) (c + a)(c + b) + 2(b + c) (a + b)(a + c) + 2(c + a) (b + c)(b + a) 1 a + 1 b + 1 c : Âș ĂŸ rÂŹng 1 c 2(a + b) (c + a)(c + b) = (c a)(c b) c(c + a)(c + b) ; do Ăą b§t ÂŻng thĂčc cŠn chĂčng minh tĂ·ĂŹng Ă·ĂŹng vĂźi (a b)(a c) a(a + b)(a + c) + (b a)(b c) b(b + a)(b + c) + (c a)(c b) c(c + a)(c + b) 0: Tuy nhi¶n ÂĄnh giÂĄ n y Ăłng theo b§t ÂŻng thĂčc Vornicu - Schur, do vÂȘy b§t ÂŻng thĂčc ban Šu ֖c chĂčng minh xong. B i toÂĄn kžt thĂłc.2 2.27 Cho a; b; c l  cÂĄc sĂš thĂŒc dĂ·ĂŹng thoÂŁ mÂąn abc = 1. ChĂčng minh rÂŹng: 4 p 2a2 + bc + 4 p 2b2 + ac + 4 p 2c2 + ab ab + bc + ca 4 p 3 : q p a + p b + p c LĂ­i giÂŁi. °t x = 1 a ; y = 1 b ; z = 1 c . Khi Ăą x; y; z 0 v  xyz = 1. çng thĂ­i ta cĂŽng cĂą 4 p r 2a2 + bc = 4 2 x2 + 1 yz r = 4 2yz + x2 x ; v  ab + bc + ca = x + y + z. Theo Ăą, b§t ÂŻng thĂčc cŠn chĂčng minh trĂ° th nh r X 4 2yz + x2 x (x + y + z) 4 p 3 s 1 p x + 1 p y + 1 p z ; hay r X 4 2yz + x2 x !4 (x + y + z)4 3 1 p x + 1 p y + 1 p z 2 : 53
  • 54. p döng b§t ÂŻng thĂčc Holder, ta cĂą 3(2yz + x2 + 2zx + y2 + 2xy + z2) 1 p x + 1 p y + 1 p z 2 X 4 r 2yz + x2 x !4 : NhĂ· vÂȘy, Âș kžt thĂłc chĂčng minh, ta cŠn chÂż ra rÂŹng 3(2yz + x2 + 2zx + y2 + 2xy + z2) (x + y + z)4 3 ; hay x + y + z 3. Tuy nhi¶n „y l  mĂ«t ÂĄnh giÂĄ Ăłng vÂŒ theo b§t ÂŻng thĂčc AM-GM x + y + z 3 3 p xyz = 3; do vÂȘy b§t ÂŻng thĂčc ban Šu ֖c chĂčng minh xong. B i toÂĄn kžt thĂłc.2 2.28 Cho a; b; c l  cÂĄc sĂš thĂŒc khĂŠng „m ĂŠi mĂ«t ph„n bi»t. ChĂčng minh rÂŹng: (ab + bc + ca) 1 (a b)2 + 1 (b c)2 + 1 (c a)2 4 LĂ­i giÂŁi. B§t ÂŻng thĂčc ban Šu mang tÂœnh Ăši xĂčng giĂșa cÂĄc bižn, do Ăą khĂŠng m§t tÂœnh tĂȘng quÂĄt, ta giÂŁ sĂ» a b c 0. Khi Ăą ta °t a b = x; b c = y. TĂž „y ta suy ra x; y 0 v  ab + bc + ca ab = (c + y)(c + x + y) y(x + y): çng thĂ­i, cĂŽng tĂž phÂČp °t tr¶n, ta cĂą 1 (a b)2 + 1 (b c)2 + 1 (c a)2 = 1 x2 + 1 y2 + 1 (x + y)2 : NhĂ· vÂȘy, ta Ă·a b i toÂĄn v· vi»c chĂčng minh y(x + y) 1 x2 + 1 y2 + 1 (x + y)2 4; hay y(x + y) x2 + x y + y x + y 3: °t t = x y . Khi Ăą t 0 v  ta cŠn chĂčng minh t + 1 t2 + t + 1 t + 1 3: Sau khi bižn ĂȘi tĂ·ĂŹng Ă·ĂŹng, ta thu ֖c mĂ«t ÂĄnh giÂĄ hiÂșn nhi¶n Ăłng (t2 t 1)2 0; do vÂȘy b§t ÂŻng thĂčc ban Šu ֖c chĂčng minh xong. B i toÂĄn kžt thĂłc.2 54
  • 55. 2.29 Cho a; b; c l  cÂĄc sĂš thĂŒc dĂ·ĂŹng thĂ€a mÂąn abc = 1. ChĂčng minh rÂŹng: a b + b c + c a + 3 ab + bc + ca + a + b + c LĂ­i giÂŁi. Do abc = 1 n¶n tçn t€i cÂĄc sĂš thĂŒc dĂ·ĂŹng x; y; z sao cho a = x y ; b = y z ; c = z x : Khi Ăą b§t ÂŻng thĂčc cŠn chĂčng minh trĂ° th nh xz y2 + xy z2 + yz x2 + 3 x z + y x + z y + x y + y z + z x ; tĂ·ĂŹng Ă·ĂŹng x3y3 + y3z3 + z3x3 + 3x2y2z2 xyz[xy(x + y) + yz(y + z) + zx(z + x)]: Tuy nhi¶n „y l  mĂ«t h» quÂŁ trĂŒc tižp cĂ”a b§t ÂŻng thĂčc Schur bÂȘc 3: m3 + n3 + p3 + 3mnp mn(m + n) + np(n + p) + pm(p + m); Ă° „y m = xy,n = yz v  p = zx. Do vÂȘy b§t ÂŻng thĂčc ban Šu ֖c chĂčng minh xong. B i toÂĄn kžt thĂłc.2 2.30 Cho a; b; c 0. ChĂčng minh rÂŹng: P a + b ab + c2 P 1 a LĂ­i giÂŁi. B§t ÂŻng thĂčc Âą cho tĂ·ĂŹng Ă·ĂŹng vĂźi: P 1 a P a + b ab + c2 = (a c)(b c) abc + c3 + (b a)(c a) abc + a3 + (a b)(c b) abc + b3 0 °t 1 abc + a3 = x; 1 abc + b3 = y; 1 abc + c3 = z: B§t ÂŻng thĂčc ֖c Ă·a v· d€ng Vornicu Schur: x(a c)(b c) + y(b a)(c a) + z(a b)(c b) 0() GiÂŁ sĂ» a b c, thž thÂŒ abc + c3 abc + b3. 1 Suy ra abc + c3 1 abc + b3 hay z y M°c khÂĄc, theo i·u giÂŁ sĂ» thÂŒ b c, do Ăą a b a c. Kžt hñp vĂźi z y 0, suy ra z(a c) y(a b). Vižt l€i b§t ÂŻng thĂčc () nhĂ· sau: x(a b)(b c) + (b c)[z(a c) y(a b)] 0 B§t ÂŻng thĂčc n y Ăłng theo cÂĄc i·u giÂŁ sĂ». PhÂČp chĂčng minh ho n t§t. D§u bÂŹng xÂŁy ra khi a = b = c.2 2.31 Cho x; y; z l  cÂĄc sĂš thĂŒc thĂ€a mÂąn x2 + y2 + z2 = 1. TÂŒm max cĂ”a biÂșu thĂčc: P = x3 + y3 + z3 3xyz LĂ­i giÂŁi. Ta cĂą: P = x3 + y3 + z3 3xyz = (x + y + z)(x2 + y2 + z2 xy yz zx). Suy ra P2 = (x + y + z)2(x2 + y2 + z2 xy yz zx)2 p döng b§t ÂŻng thĂčc AM-GM cho 3 sĂš khĂŠng „m: 55
  • 56. P2 = (x + y + z)2(x2 + y2 + z2 xy yz zx)2 = (x + y + z)2(x2 + y2 + z2 xy yz zx)(x2 + y2 + z2 xy yz zx) (x + y + z)2 + (x2 + y2 + z2 xy yz zx) + (x2 + y2 + z2 xy yz zx) 3 3 = (x2 + y2 + z2)3 = 1 (theo giÂŁ thižt) . Suy ra P 1. VÂȘy maxP = 1 , (x+y +z)2 = x2 +y2 +z2 xy yz zx , x = 1; y = z = 0 v  cÂĄc hoÂĄn vĂ .2 2.32 Cho a; b; c 0. ChĂčng minh rÂŹng: a2 b + c + b2 a + c + c2 a + b a2 a + b + b2 b + c + c2 c + a LĂ­i giÂŁi. CÂĄch 1 Ta cĂą: P a2 b2 a + b = a b + b c + c a = 0 Suy ra P 2a2 a + b = P a2 + b2 a + b Khi Ăą ta cŠn chĂčng minh: P 2c2 a + b P a2 + b2 a + b B§t ÂŻng thĂčc n y tĂ·ĂŹng Ă·ĂŹng vĂźi: P 2c2 a2 b2 a + b 0 hay P c2 a + b a2 a + b + a2 b + c c2 b + c 0 hay P (c a)2(c + a) (a + b)(b + c) 0 (Ăłng) PhÂČp chĂčng minh ho n t§t. ÂŻng thĂčc xÂŁy ra khi v  chÂż khi a = b = c CÂĄch 2 B§t ÂŻng thĂčc Âą cho tĂ·ĂŹng Ă·ĂŹng vĂźi: P a2 1 b + c 1 a + c 0 hay (a2(a2 c2) + b2(b2 a2) + c2(c2 b2) 0 hay 1 2 [(a2 b2)2 + (b2 c2)2 + (c2 a2)2] 0 (Ăłng) . PhÂČp chĂčng minh ho n t§t. ÂŻng thĂčc xÂŁy ra khi v  chÂż khi a = b = c.2 2.34 Cho x; y; z 0. ChĂčng minh rÂŹng: a2 + bc (b + c)2 + b2 + ac (a + c)2 + c2 + ab (a + b 2 3 2 LĂ­i giÂŁi. B§t ÂŻng thĂčc Âą cho tĂ·ĂŹng Ă·ĂŹng vĂźi: P a2 + bc (b + c)2 1 2 0 hay 56
  • 57. P 2a2 b2 c2 (b + c)2 0 GiÂŁ sĂ» a b c. Khi Ăą ta cĂą hai dÂąy cĂČng chi·u: 8 : 2a2 b2 c2 2b2 a2 c2 2c2 a2 b2 1 (b + c)2 1 (a + c)2 1 (a + b)2 p döng b§t ÂŻng thĂčc Chebychep cho hai dÂąy tr¶n: P (2a2 b2 c2): 1 (b + c)2 [ P (2a2 b2 c2)] : P 1 (b + c)2 = 0: P 1 (b + c)2 = 0 PhÂČp chĂčng minh ho n t§t. ÂŻng thĂčc xÂŁy ra khi v  chÂż khi a = b = c.2 2.35 Cho a; b 0. ChĂčng minh rÂŹng: (a2 + b2)(a + b)2 + (ab + 1)2 2(a + b)2 LĂ­i giÂŁi. Âș ĂŸ rÂŹng a2 + b2 = (a + b)2 2ab. °t a2 + b2 = x; ab = y B§t ÂŻng thĂčc cŠn chĂčng minh tĂ·ĂŹng Ă·ĂŹng vĂźi: x(x + 2y) + (y + 1)2 2(x + 2y) Khai triÂșn v  rĂłt gĂ„n, ta ֖c: x2 + y2 + 1 2x 2y + 2xy 0 hay (x + y 1)2 0 (Ăłng) PhÂČp chr Ăčng minh ho n t§t. ÂŻng thĂčc xÂŁy ra khi v  chÂż khi a2 + b2 + ab = 1 (chÂŻng h€n a = b = 1 3 ).2 2.36 Cho a; b; c 2 R. ChĂčng minh rÂŹng: a3 b3 (a b)3 + b3 c3 (b c)3 + c3 a3 (c a)3 9 4 LĂ­i giÂŁi. B§t ÂŻng thĂčc cŠn chĂčng minh tĂ·ĂŹng Ă·ĂŹng vĂźi: P a2 + b2 + ab (a b)2 9 4 NhÂȘn th§y rÂŹng: a2 + ab + b2 (a b)2 = 1 4 (a + b)2 + 3 4 (a b)2 (a b)2 3 4 (a b)2 (a b)2 = 3 4 Thöc hi»n tĂ·ĂŹng tĂŒ cho hai b§t ÂŻng thĂčc cĂĄn l€i. PhÂČp chĂčng minh ho n t§t. ÂŻng thĂčc xÂŁy ra khi v  chÂż khi a + b + c = 0.2 2.37 Cho a; b 0. ChĂčng minh rÂŹng: 1 a2 + 1 b2 + 4 a2 + b2 32(a2 + b2) (a + b)4 LĂ­i giÂŁi. SĂ» döng b§t ÂŻng thĂčc AM-GM cho hai sĂš dĂ·ĂŹng: 1 a2 + 1 b2 + 4 a2 + b2 = a2 + b2 a2b2 + 4 a2 + b2 r a2 + b2 2 a2b2 : 4 a2 + b2 = 4 ab 57
  • 58. Ta sÂł chĂčng minh: 4 ab 32(a2 + b2) (a + b)4 hay 8ab(a2 + b2) (a + b)4 p döng b§t ÂŻng thĂčc 4xy (x + y)2: 8ab(a2 + b2) = 4:2ab:a2 + b2) (a2 + b2 + 2ab)2 = (a + b)4 PhÂČp chĂčng minh ho n t§t. ÂŻng thĂčc xÂŁy ra khi v  chÂż khi a = b.2 2.38 Cho a; b 0 thĂ€a mÂąn a2 + b2 + c2 + abc = 4. ChĂčng minh rÂŹng: a + b + c 3 LĂ­i giÂŁi. CÂĄch 1: Theo nguy¶n lÂœ Dirichlet, trong ba sĂš a; b; c ­t sÂł cĂą hai sĂš cĂČng phÂœa vĂźi 1 tr¶n tröc sĂš. G¿£ sĂ» hai sĂš Ăą l  a v  b. Thž thÂŒ: (a 1)(b 1) 0 hay ab a + b 1 . M°t khÂĄc, theo giÂŁ thižt v  b§t ÂŻng thĂčc AM-GM cho hai sĂš dĂ·ĂŹng: 4 c2 = a2 + b2 + abc 2ab + abc = ab(2 + c) hay (2 c)(2 + c) ab(2 + c) hay 2 c ab Kžt hñp vĂźi b§t ÂŻng thĂčc ab a + b 1 (chĂčng minh tr¶n), suy ra: 2 c ab a + b 1 hay a + b + c 3 . PhÂČp chĂčng minh ho n t§t. ÂŻng thĂčc xÂŁy ra khi v  chÂż khi a = b = c = 1 CÂĄch 2: °t a = 2x p (x + y) (x + z) ; b = 2y p (y + z) (y + x) ; c = 2z p (z + y) (z + x) Suy ra: a + b + c = P p p 2x y + z (x + y) (y + z) (z + x) VÂŒ thž b§t ÂŻng thĂčc a + b + c 3 sÂł tĂ·ĂŹng Ă·ĂŹng vĂźi: P 2x p (x + y)(y + z)(z + x) p y + z 3 „y chÂœnh l  b§t ÂŻng thĂčc Schur vĂźi cÂĄc bižn p y + z; p y + x; p z + x. PhÂČp chĂčng minh ho n t§t. ÂŻng thĂčc xÂŁy ra khi v  chÂż khi a = b = c = 1.2 CÂĄch 3: GiÂŁ sĂ» tçn t€i mĂ«t sĂš (cho sĂš Ăą l  a) trong ba sĂš a; b; c lĂźn hĂŹn 2. Khi Ăą, vÂŒ a; b; c dĂ·ĂŹng n¶n: a2 + b2 + c2 + abc = 4 4 + b2 + c2 + abc 4 (vĂŠ lÂœ!) Do Ăą a; b; c 2 (0; 2] TĂž giÂŁ thižt suy ra: a2 + abc + b2c2 4 = 4 + b2c2 4 b2 c2 58
  • 59. hay a + bc 2 2 = (4 b2)(4 c2) 4 Do b; c 2 n¶n suy ra: a + b + c = r (4 b2) (4 c2) 4 bc 2 + b + c p döng b§t ÂŻng thĂčc AM-GM, ta cĂą: r (4 b2) (4 c2) 4 bc 2 + b + c 1 2 (4 b2 + 4 c2) bc 2 + b + c = 3 b + c 2 2 1 3 . PhÂČp chĂčng minh ho n t§t. ÂŻng thĂčc xÂŁy ra khi v  chÂż khi a = b = c = 1.2 NhÂȘn xÂČt: B§t ÂŻng thĂčc a + b + c 3 cĂŽng Ăłng vĂźi i·u ki»n a2 + b2 + c2 + 3 2 abc = 9 2 . 2.39 Cho x; y; z 0. ChĂčng minh rÂŹng: 1 x + 1 y + 1 z 36 9 + x2y2 + y2z2 + z2x2 LĂ­i giÂŁi. °t xy = a; yz = b; xz = c, b§t ÂŻng thĂčc trĂ° th nh: p abc (a + b + c)(a2 + b2 + c2 + 9) 36 p döng b§t ÂŻng thĂčc AM-GM: a + b + c 3 3 p p (abc)4 abc = 3 12 q 3 3 p (abc)2:3:3:3 = 12 12 a2 + b2 + c2 + 9 3 3 p (abc)2 + 9 = 3 3 p (abc)2 + 3 + 3 + 3 4 4 p (abc)2 Nh„n vž theo vž hai b§t ÂŻng thĂčc tr¶n: (a + b + c)(a2 + b2 + c2 + 9) 3 3 p p (abc)2 = 36: abc:12 12 p abc . PhÂČp chĂčng minh ho n t§t. ÂŻng thĂčc xÂŁy ra khi v  chÂż khi a = b = c = 1.2 2.40 Cho a; b; c 0 thĂ€a mÂąn a2 + b2 + c2 = 3. ChĂčng minh rÂŹng: 4 a2 + b2 + 1 4 b2 + c2 + 1 4 c2 + a2 + 1 3(a + b + c)2 LĂ­i giÂŁi. p döng b§t ÂŻng thĂčc Holder: 4 a2 + b2 + 1 4 b2 + c2 + 1 4 c2 + a2 + 1 r 3 64 (a2 + b2)(b2 + c2)(c2 + a2) + 1 3 p döng b§t ÂŻng thĂčc AM-GM v  b§t ÂŻng thĂčc 3(x2 + y2 + z2) (x + y + z)2: r 3 64 (a2 + b2)(b2 + c2)(c2 + a2) + 1 3 12 2(a2 + b2 + c2) + 1 3 = 27 = 9(a2 + b2 + c2) 3(a + b + c)2 PhÂČp chĂčng minh ho n t§t. ÂŻng thĂčc xÂŁy ra khi v  chÂż khi a = b = c = r 1 3 .2 3.3 B i 3.1 žn b i 3.40 3.1 Cho a; b; c; x; y; z 0 thĂ€a mÂąn x + y + z = 1. ChĂčng minh rÂŹng: ax + by + cz + 2 p (xy + yz + zx)(ab + bc + ca) a + b + c LĂ­i giÂŁi. 59
  • 60. p döng b§t ÂŻng thĂčc Cauchy-Schwarz: ax + by + cz + 2 p (xy + yz + zx)(ab + bc + ca) pP a2: P x2 + p 2 P xy:2 P ab p ( P a2 + 2 P P ab)( x2 + 2 P xy) = a + b + c (do x + y + z = 1) PhÂČp chĂčng minh ho n t§t. ÂŻng thĂčc xÂŁy ra khi v  chÂż khi a x = b y = c z = 1 a + b + c = a + b + c x + y + z = a + b + c hay a + b + c = 1. 2 3.2 Cho a; b; c 0 thĂ€a mÂąn a3 + b3 + c3 = 3. TÂŒm giÂĄ trĂ  lĂźn nh§t cĂ”a: P = a4b4 + b4c4 + c4a4 LĂ­i giÂŁi. Ta chĂčng minh giÂĄ trĂ  lĂźn nh§t cĂ”a biÂșu thĂčc l  3. °t a3 = x; b3 = y; c3 = z, suy ra x + y + z = 3. p döng AM-GM 3a4b4 a3b3(a3 + b3 + 1) Khi Ăą, ta chÂż cŠn chĂčng minh: xy(x + y + 1) + yz(y + z + 1) + zx(z + x + 1) 9 Ă·a v· d€ng çng bÂȘc, ta cŠn chĂčng minh 3 P xy(x + y) + (x + y + z)(xy + yz + zx) (x + y + z)3 Sau khi khai triÂșn, b§t ÂŻng thĂčc trĂ° th nh: x3 + y3 + z3 + 3xyz P xy(x + y) Ăłng theo b§t ÂŻng thĂčc Schur. PhÂČp chĂčng minh ho n t§t. ÂŻng thĂčc xÂŁy ra khi v  chÂż khi a = b = c = 1.2 3.3 Cho a; b; c 0 thĂ€a mÂąn a2 + b2 + c2 = 1. TÂŒm giÂĄ trĂ  nhĂ€ nh§t cĂ”a: P = a2 b + c + b2 c + a + c2 a + b LĂ­i giÂŁi. CÂĄch 1: GiÂŁ sĂ» a b c. Ta sÂł cĂą hai dÂąy cĂČng chi·u: 8 a2 b2 c2 1 b + c : 1 a + c 1 a + b p döng lŠn l֖t b§t cÂĄc b§t ÂŻng thĂčc Chebychep, giÂŁ thižt a2 + b2 + c2 = 1 v  b§t ÂŻng thĂčc P 1 x P9 x , ta cĂą: P = a2 b + c + b2 c + a + c2 a + b 1 3 (a2 + b2 + c2) 1 b + c + 1 c + a + 1 a + b 3 2 (a + b + c) L€i theo b§t ÂŻng thĂčc: 3 = 3(a2 + b2 + c2) (a + b + c)2 Suy ra: a + b + c p 3 . Do Ăą: P 3 2(a + b + c) p 3 2 . VÂȘy minP = p 3 2 , a = b = c = 1 p 3 . 60